Está en la página 1de 144

www.upscportal.

com
Email us at igp@upscportal.com
www.upscportal.com
BROCHURE OF STUDY KI T
PAPER 2
IAS PRE 2013
at +918800734161, 011-65023618
Contents of The Brochure
1. Study Kit Details of Paper - II
Our Objectives
Our Strategy
Implementation Aspects
Contents of the Kit
Price of the Kit
Payment Options
Contact Details
2. Contents Details of Study Kit Paper - II
(I) Contents of Comprehension & English Language Comprehension
Contents of Comprehension
Contents of English Language Comprehension
(II) Contents of Interpersonal & Communication Skills, Decision Making & Problem Solving
Contents of Interpersonal & Communication Skills
Contents of Decision Making & Problem Solving
(III) Contents of General Mental Ability, Logical Reasoning& Analytical Ability
Contents of General Mental Ability
Contents of Logical Reasoning & Analytical Ability
(IV) Contents of Basic Numeracy
Contents of Basic Numeracy
(V) Contents of Data Interpretation & Data Sufficiency
Contents of Data Interpretation & Data Sufficiency
3. Sample Study Materials of Our Study Kit
(I) Comprehension
Comprehension
(II) English Language & Comprehension
Sentence
Sentence Correction
English Language Comprehension
(III) Interpersonal & Communication Skills
Interpersonal & Communication Skills
(IV) Decision Making and Problem Solving
Decision Making and Problem Solving
(V) General Mental Ability
Blood Relations
Sitting Arrangements
Arithmetical Reasoning
Number, Ranking and Time Sequence Test
(VI) Logical Reasoning & Analytical Ability
Syllogism
Statement and Courses of Action
Assertion and Reason
Situation Reaction Tests
Statement and Assumptions
(VII) Basic Numarcy
Number System
Simplification
Average
Percentage
Time and Distance
Probability
Set Theory, Venn Diagrams, Functions & Relations
Sequences & Series
(VIII) Data Interpretation & Data Sufficiency
Introduction to Data Interpretation
Pie Charts
Mix Diagrams
Data Sufficiency

Click Here to Order:
http://www.upscportal.com/civilservices/study-kit/ias-pre-csat-paper-2
For Any Help Call Our Expert: at +918800734161, 011-65023618
Dear Candidates,
The Complete Study Kit in Hard copy of paper II for civil service preliminary examination is now
available.
Contents of the Kit:
Booklet No Subjects Pages
Contents along
with practice set
after each
chapters
1
Comprehension & English Language With
Practice Set
171 1000+ Practice Set
2
Interpersonal Skill & Decision Making With
Practice Set
71 240 Practice Set
3
General Mental Ability, Logical Reasoning &
Analytical Ability With Practice Set
290 2200+ Practice Set
4 Basic Numeracy With Practice Set 432 3000+ Practice Set
5 Data Interpretation With Practice Set 151 1000+ Practice Set
Total
Booklets - 5

Total Pages:
1100+
Total Questions:
7400+

Click Here for How To Purchase
http://www.upscportal.com/civilservices/study-kit/how-to-purchase

For Any Guidance Call our CSAT Paper - 2 Expert at +91 9911157134, 011- 65023618
Price of the Kit:
1. The price of our Study Kit for Paper II is Rs. 2500, including courier charges.
Our Objectives:
Firstly to cover 100% civil service preliminary examination (IAS) syllabus.
Secondly to compile all the required study materials in a single place, So to save the precious time of
the aspirants.
Our Strategy:
Content of every section of the syllabus is developed after a thorough research of last year Question
Papers.
Every section is covered with practice set.
Click Here to Order:
http://www.upscportal.com/civilservices/study-kit/ias-pre-csat-paper-2
For Any Help Call Our Expert: at +918800734161, 011-65023618
Implementation Aspects:
You will Get 5 booklets of (i). Comprehension & English Language Comprehension, (ii)
Interpersonal Skill & Communication Skill and Decision Making & Problem Solving (iii) General
Mental Ability, Logical Reasoning and Analytical Ability (iv) Basic Numeracy (v) Data
Interpretation and Data Sufficiency.
Study Kit will be delivered to your postal Address after payment confirmation.
After dispatching your kit we will provide you a DTDC courier tracking details.
For any help we will provide Telephonic & Email Support to the candidates.
Click Here for Payment Option
http://www.upscportal.com/civilservices/study-kit/payment-
options
After Your Payment Click the Given Link for further Process:
http://www.upscportal.com/civilservices/study-kit/submit-
payment-details
Thank You
Course Director
Click Here to Order:
http://www.upscportal.com/civilservices/study-kit/ias-pre-csat-paper-2
For Any Help Call Our Expert: at +918800734161, 011-65023618

Contents of Study Kit Paper -2
Contents of Comprehension & English Language Comprehension
Contents of Comprehension
Comprehension
Contents of English Language Comprehension
Narration (Direct and Indirect)
Sentence
Common Error
One-Word Substitution
Synonyms and Antonyms
Idioms and Phrases
Sentence Correction
Sentence Arrangement
Analogy
Foreign Words and Phrases
English Language Comprehension
Contents of Interpersonal & Communication Skills, Decision
Making & Problem Solving
Contents of Interpersonal & Communication Skills
Interpersonal & Communication Skills
Contents of Decision Making & Problem Solving
Decision Making and Problem Solving
Contents of General Mental Ability, Logical Reasoning &
Analytical Ability
Contents of General Mental Ability
Analogy
Classification
Series
Coding-Decoding
Click Here to Order:
http://www.upscportal.com/civilservices/study-kit/ias-pre-csat-paper-2
For Any Help Call Our Expert: at +918800734161, 011-65023618
Blood Relations
Direction Sense Test
Logical Venn Diagrams
Alphabet Test
Sitting Arrangements
Mathematical Operations
Arithmetical Reasoning
Inserting the Missing Character
Number, Ranking and Time Sequence Test
Eligibility Test
Contents of Logical Reasoning & Analytical Ability
Syllogism
Statement and Arguments
Statement and Assumptions
Statement and and Courses of Action
Statement and Conclusions
Deriving Conclusion
Assertion and Reason
Punch lines
Situation Reaction Tests
Cause and Effect
Analytical Reasoning
Contents of Basic Numeracy
Contents of Basic Numeracy
Number System
Fractions
Indices and Surds
Square Root & Cube Root
Simplification
HCF & LCM
Orders of Magnitude
Unitary Method
Average
Percentage
Profit and Loss
Ratio & Proportion
Partnership
Alligation or Mixure
Time and Work
Time and Distance
Simple Interest
Compound Interest
Click Here to Order:
http://www.upscportal.com/civilservices/study-kit/ias-pre-csat-paper-2
For Any Help Call Our Expert: at +918800734161, 011-65023618
Area of Plane Figures
Volume and Surface Area of Solid Figures
Clocks and Calendar
Concepts of Geometry
Coordinate Geometry
Combinatorics
Probability
Basic Algebra
Set Theory, Venn Diagrams, Functions & Relations
Statistics
Sequences & Series
Contents of Data Interpretation & Data Sufficiency
Contents of Data Interpretation & Data Sufficiency
Introduction to Data Interpretation
Approaches to Data Interpretation
Table Chart
Pie Charts
Bar Charts
Line Graphs
Mix Diagrams
Data Sufficiency


Sample Chapter
of
Comprehension
WWW.UPSCPORTAL.COM For Any Help Call Our Course Director:
at +91 8800734161, 011- 65023618
Click Here to Order
http://www.upscportal.com/civilservices/study-kit/ias-pre-csat-paper-2
Di recti ons (Q.111): Read the fol l owi ng passage
careful l y and answer the questi ons gi ven bel ow
i t. Certai n words are pri nted i n bol d i n the passage
to hel p you l ocate them whi l e answeri ng some of
the questi ons.
What i s i mmedi atel y needed today i s the
establ i shement of a Wrol d Government or an
I nternational Federation of mankind. I t is the utmost
necessity of the world today, and all those persons who
wish to see all human beings happy and prosperous
naturally feel it keenly.
Of course, at ti mes we feel that many of the
problem of our political, social, linguistic and cultural
life would come to an end if there were one Govenment
all ovet the world. Travellers, businessmen, seekers of
knowledge and teachers of righteousness know very well
that great impediments and obstructions are faced by
them when they pass from one country to another,
exchange goods, get information, and make an efforts
to spread their good gospel among their fellow-men. I n
the past, religious sects divided one set of people against
another, colour of skin or shape of the body set one
against the other.
But today when philosophical light has exploded
the darkness that was created by religious differences,
and when sci enti fi c knowl edge has fl asi fi ed the
superstitions, they have enabled human beings of all
religious views and of all races and colours to come in
frequent contact with one another . I t is the governments
of various countries that keep poeple of one country
apart from, those of another. They create artificial
barriers, unnatural distinctions, unhealthy isolation,
unnecessary fears and dangers in the minds of common
men who by their nature want to live in friendship with
their fellow-men. But all these evils would cease to exist
if there were one Government all over the world.
1. What divides people of a country against another?
(a) Different religions
(b) Different language
(c) Di fferent soci al and pol i ti cal systems of
different people
(d) Government of various countries
2. What is the urgent heed of the world today?
(a) The establ i shment of an i nternati onal
economic order.
(b) The establishment of a world government.
(c) The creation of a cultural international social
order.
(d) The raising of an international spiritual army.
3. What will the world Government be expected to
do?
(a) it will arrange for interplanetary contacts
(b) it will end all wars for all time to come
(c) it will bring about a moral regeneration of
mankind
(d) it will kill the evil spirit in man
4. Choose the word which is SI MI LAR in meaning
as the word "ri ghteousness" as used i n the
passage.
(a) rectitude (b) religiosity
(c) requirement (d) scrupulousness
EXERCISE 1
Comprehension
WWW.UPSCPORTAL.COM For Any Help Call Our Course Director:
at +91 8800734161, 011- 65023618
Click Here to Order
http://www.upscportal.com/civilservices/study-kit/ias-pre-csat-paper-2
5. Which of the following problems has not been
mentioned in the passage as likely to be solved
with the establishment of world Government?
(a) Social Problems (b) Political Problems
(c) Cultural Problems (d) Economic Problems
6. Choose the word which is most OPPOSITE in
meaning of the word 'implediments' as used in
the passage.
(a) handicaps (b) furtherance
(c) providence (d) hindrances
7. The most appropriate title of the above passage
may be ...........
(a) The evils of the world order.
(b) The man can make his destiny.
(c) The need of world Government.
(d) The role of Religion in the Modern Times.
8. What was the factor, according to the passage,
that set one man against another?
(a) Material prosperity of certain people in the
midst of grinding poverty.
(b) Superior physical strength of some persons.
(c) Colour of skin or shape of the body.
(d) Some peopl e bei ng educated and other
illiterate.
9. The theory of racial superiority stands falsified
today by ........
(a) knowledge derived from scientific advances.
(b) the ascendancy of people who were here to fore
considered of inferior racial stock.
(c) the achievements of the so called backward
countries in every field of life.
(d) the precedence given to the physical powers
of different races.
10. I n the part religious sects .............
(a) united the people with one another.
(b) I nterfered in political affairs.
(c) did a good job by way of spreading message of
love and peace.
(d) divided one set of people from another.
Di recti ons (Q. 12-18): Read the fol l owi ng passage
careful l y and answer the questi on gi ven bel ow i t.
Certai n words are pri nted i n bol d to hel p you to
l ocate them whi l e answer i ng some of the
questi ons.
The window offered a view of the house opposite.
The two families did not speak to each other because of
a property dispute. One day, Ruchira's textbooks lay
untouched as the young gi rl 's gaze was on the
happenings in the house opposite. There were two new
faces in the neighbouring household that of an elderly
widow and a girls aged sixteen. Some times the elderly
lady would sit by the window, doing the young girl's
hair. On other days she was absent.
The new young neighbour's daily routine could be
seen through the window she cleaned the rice paddy;
split nuts, put the cushions in the sun to air them. I n
the afternoons while the men were all at world some of
the women slept and others played cards. The girl sat
on the terrace and read. Sometimes she wrote. One day
there was hindrance. She was writing when the elderly
woman snatched the unfinished letter from her hands.
Thereafter the girl was not to be seen on the terrace.
Sometimes during the day sounds came from the house
indicating that a massive argument was going on inside.
A few days passed. One evening Ruchira noticed
the girl standing on the terrace in tears. The evening
prayer was in progress. As she did daily, the girl bowed
several times in prayer. Then she went downstairs. That
night Ruchira wrote a letter. She went out and posted
it that very instant. But as she lay in bed that night,
she prayed fervently that her offer of friendship wouldn't
reach its destination. Ruchira then left for Madhupur
and returned when it was time for college to start. She
found the house opposite in darkness, locked. They had
left.
When she stepped into her room she found the desk
piled with letters one had a local stamp on it with her
name and address in unfamiliar handwriting. She
quickly read it. They continued to write to each other
for the next twenty years.
11. Why di d Ruchi ra wri te a l etter to her new
neighbour?
(a) She wanted to offer her, her help.
(b) She wanted to be friends with her.
(c) To apol ogi ze for her fami l y's behavi our
towards her family.
(d) To encourage her to continue learning to read
and write.
12. Which of the following can be said about Ruchira?
A. She used to spy on her neighbours because
she didn't trust them.
B. She was at home because she was studying.
C. She did not speak to her neighbours because
they did not own property.
(a) None (b) Only B
(c) Both A & B (d) Only C
WWW.UPSCPORTAL.COM For Any Help Call Our Course Director:
at +91 8800734161, 011- 65023618
Click Here to Order
http://www.upscportal.com/civilservices/study-kit/ias-pre-csat-paper-2
13. How did the new young neighbour spend her
days?
(a) She was busy writing letters to Ruchira.
(b) She used to daydream about her past
experiences.
(c) She would attend to the needs of the widow.
(d) She spent her time learning to read and write.
14. Why was the young neighbour prevented from
sitting on the terrace?
(a) She used to while away her time instead of
working
(b) The old woman could no longer keep an eye
on her.
(c) She had not finished writing the letter she was
asked to.
(d) She had been writing a letter which she wasn't
supposed to.
15. What was the major argument in the house about?
(a) There were too many people living there,
which resulted in arguments.
(b) The young girl was insisting on attending
college.
(c) The young girl had been wasting her time
instead of working.
(d) The old woman did not guard the young girl
closely.
16. Which of the following is TRUE in the context of
the passage?
(a) The young girl was very devout and prayed
everyday.
(b) Only two letters were exchanged between the
two girls.
(c) The new young neighbour was a servant.
(d) The afternoon was a time to relax for everyone.
17. Why did the young girl wish that the letter would
not reach its destination?
A. She was going away and would not be able to
see if her neighbour was glad to receive it.
B. She was afraid that it would lead to a quarrel
between the two families.
C. She was afraid that her neighbour would be
angry when she received her letter.
(a) None
(b) Only A
(c) Only C
(d) Both B & C
Di r ecti ons (Q.18-20): Choose the word whi ch i s
most nearl y the SAME i n meani ng as the worl d
pri nted i n bol d as used i n the passage.
18. Hindrance
(a) handicapped (b) delay
(c) interruption (d) difficult
19. Offered
(a) forward (b) willing
(c) volunteered (d) provided
20. Choose the word which is most OPPOSI TE in
meaning of the word piled as used in the passage.
(a) low (b) empty
(c) blank (d) nothing
Di recti ons (Q. 21-27): Read the fol l owi ng passage
careful l y and answer the questi ons gi ven bel ow
i t. Certai n words are pri nted i n bol d to hel p you to
l ocate them whi l e answer i ng some of the
questi ons.
The yearly festival was close at hand. The store
room was packed with silk fabrics, gold ornaments, clay
bowls full of sweet curd and platefuls of sweetmeats.
The orders had been placed with shops well in advance.
The mother was sending out gifts to everyone.
The eldest son, a government servant, lived with
his wife and children in far off lands. The second son
had left home at an early age. As a merchant he travelled
all over the world. The other sons had split up over petty
squabbles, and they now lived in homes of their own.
The relatives were spread all across the world. They
rarely visited. The youngest son, left in the company of
a servant, was soon bored left her and stood at the door
all day long, waiting and watching. His mother, thrilled
and excited, loaded the presents on trays and plates,
covered them with colourful kerchiefs, and sent them
off with maids and servants. The neighbours looked on.
The day came to an end. All the presents had been
sent off.
The child came back into the house and dejectedly
said to his mother, "Maa, you gave present to everyone,
but you didn't give me anything !"
His mother laughed, "I have given all the gifts
away to everyone, now see what's left for you." She
kissed him on the forehead.
The child said in a tearful voice, "Don't I get a gift?"
"You'll get it when you go far away."
"But when I am close to you, don't I get something
from your own hands?"
His mother reached out her arms and drew him to
her "This is all I have in my own hands. I t is the most
precious of all."
WWW.UPSCPORTAL.COM For Any Help Call Our Course Director:
at +91 8800734161, 011- 65023618
Click Here to Order
http://www.upscportal.com/civilservices/study-kit/ias-pre-csat-paper-2
21. Why did the woman's second son travel?
(a) He was restless by nature.
(b) He did not want to stay at home.
(c) He was rich and could afford to travel.
(d) His job prevented him from taking leave
22. Why did the woman's eldest son not attend the
festival?
(a) He was not on good terms with his youngest
brother who lived at home.
(b) He had quarrelled with his mother.
(c) His wife did not allow him to return home.
(d) His job prevented him from taking leave
23. How did the woman prepare for the fesitval?
A. She bought expensive gifts for her children
and neighbours.
B. She ordered her servents to prepare sweets
and food well in advance.
C. She made sure that her youngest child was
looked after so that he wouldn't be bored.
(a) None (b) Only A
(c) Only B (d) Both A & B
24. What did the youngest child do while his mother
was busy?
(a) He wai ted for a chance to steal some
sweetmeats
(b) He pestered his mother to give him a present.
(c) He stood at the door with the servants.
(a) Only A (b) Only B
(c) Both A & C (d) Only C
25. Which of the following can be said about the
woman?
(a) She was a widow who had brought up her
childern single handedly.
(b) She was not a good mother since her childern
had left home at an early age.
(c) She enjoyed sending her family gifts at festival
time.
(d) She gave expensive presents to show that she
was wealthy.
26. What did the boy receive from his mother?
(a) She taught him the value of patience
(b) She encouraged him to grow up and li ve
independently like his brother.
(c) She showed him the importance of giving
expensive gifts.
(d) She gave him a hug to express her love.
27. Which of the following is TRUE in the context of
the passage?
(a) The woman usually ignored her youngest son.
(b) The woman's eldest son lived abroad.
(c) The memers of the woman's family did not care
about her.
(d) The woman made all the preparations herself
since she did not want to burden the servants.
Di recti ons (Q. 28 30): Choose the word whi ch i s
most nearl y the SAME i n meani ng as the word
pri nted i n bol d as used i n the passage.
28. Left
(a) gone (b) quit
(c) remaining (d) disappeared
29. Packed
(a) filled (b) squeezed
(c) crowd (d) collected
30. Choose the word which is most OPPOSI TE in
meaning of the word dejectedly as used in the
passage.
(a) calmly (b) happily
(c) willingly (d) fortunately
Di recti on (31-36): Read the fol l owi ng two passages
careful l y and answer the questi ons gi ven bel ow
them. Certai n words/expressi ons are gi ven i n bol d
i n P assage I to hel p you l ocate them whi l e
answeri ng some of the questi ons.
Ghanshyam Das Birla was a great architect of
I ndia's industrial growth. He started his career in
Kolkata at the beginning of 20th century. He set up
many industries. He entered the field of business during
the days of the First World War and established himself
after the war years. First, he established a cotton mill
in Sabzi Mandi, Delhi, followed by Keshoram Cotton
Mills and Birla J ute Mills around 1920. The Keshoram
Mills were set up with the efforts of Andrew Yule. I n
1919, with an investment of Rs 50 lakhs, Birla Brothers
Limited was formed and thereafter a mill was set up in
Gwalior.
Mr. Birla realised that political freedom from British
rule was imperative for the industrial growth of I ndia.
I n 1920, he came into contact with Gandhiji and became
his disciple. I n the decade of the 30's he set up sugar
paper mills. From 1943 to 1946, with the stock exchange
gaining ground, Birla Brothers ventured into the
areas of cars, cotton, machinery and man-made fabrics.
United Commercial Bank was set up during this period.
Prior to this, he had established Ruby, Asiatic I nsurance
Co. and I nland Air Service.
After independence, the Birlas expanded their
business and started production in many fields. Near
WWW.UPSCPORTAL.COM For Any Help Call Our Course Director:
at +91 8800734161, 011- 65023618
Click Here to Order
http://www.upscportal.com/civilservices/study-kit/ias-pre-csat-paper-2
Mirzapur, he, in collaboration with Caesar, an American
friend, set up an aluminium plant Hindalco in record
time. He bought the Century Mill from Sir Chunnilal
V.Mehta, the cousin of Sir Purshottam Das Thakur. He
also bought tea estates and started cement and fertiliser
factories. He established a new style of management.
I n his birth place Pilani, and at many other places he
started many educational institutions. To his credit go
many temples, planetariums and hospitals. I n 1983, he
died while in London, but not before seeing his business
flourish as one of the topmost establishments in I ndia.
During the decades of 70's and 80's, Birla Brothers was
among the topmost I ndustrial Houses in I ndia.
Anna Saheb Karve's life was an simple and clean
as that of an ancient ascetic (Rishi Muni). He was liberal
in his views and sympathetic in his attitude. He was
deeply moved by the troubles and tribulations of women
flok.
What Anna Saheb did for the upliftment of women
in the last decade of the nineteenth century was beyond
the imagination of the people. He created an awakening
among women through his writing in Kesari. He was a
staunch supporter of widow remarriage. On March 11,
1893, he set an example for society by taking the bold
step of marrying a widow. The couple dedicated their
lives to social work and reform. He established an
orphanage in Pune.
Anna Saheb realised the importance of education
for women. He spared no effort in starting the Bharatiya
Mahila Vidyapeeth, the first university for women. I n
1921, he travelled to Europe and America and met
famous people like Albert Einstein and formulated his
opinions regarding work. I n 1958, he was awarded the
'Bharat Ratna' i n recogni ti on of hi s servi ces i n
educational and social reforms.
31. Ghanshyam Das Birla was better known to I ndia
as
(a) a freedom fighter against British rule
(b) an architect of temples in various cities
(c) a pioneer of I ndia's industrial growth
(d) an architect of the textile industry in I ndia
32. Ghanshyam Das Birla's first industrial venture
was
(a) a textile related industry
(b) a cotton mill, Gwalior
(c) Keshoram Cotton Mills, Delhi
(d) car manufacturing company
33. What was Andrew Yule's role in Ghanshyam Das
Birla's career?
(a) He helped Ghanshyam Das Birla in setting
up cotton mills in Sabzi Mandi, Delhi.
(b) He invested Rs. 50 lakhs in Keshoram Cotton
Mills
(c) He extended a lot of help to Ghanshyam Das
Birla in his second venture.
(d) He helped Ghanshyam Das Birla in the field
of business before the First World War.
34. What was Ghanshyam Das Birla's view about the
British rule in I ndia?
(a) The British rule was very atrocious.
(b) The British rule was counter productive to
I ndia's industrial growth.
(c) The British rule was not a hindrance to I ndia's
industrial growth.
(d) The industrial growth of I ndia had nothing to
do with gaining freedom from the British.
35. Which of the following can be inferred from the
passage ?
A. Ghanshyam Das Birla could set up a large
number of diverse industries due to British rule
in I ndia.
B. Ghanshyam Das Bi rl a, wi th hi s fami l y
members, started various industries due to
flourishing of the Stock Exchange.
C. Ghanshyam Das Bi rla was i mpressed by
Gandhiji's philosopy.
(a) Both A and B (b) Both A and C
(c) All the three (d) Only B and C
36. Which of the following statements is FALSE in
the context of the passage?
A. Ghanshyam Das Birla unfortunately died
before realising the progress his business had
made.
B. Hindalco was set up in a very short time span.
C. Ghanshyam Das Birla's management style
was different from the traditional one.
(a) Only A (b) Only B
(c) Only C (d) Both A and B
Di recti ons (Q. 37-38): Choose the word/group of
wor ds whi ch i s/ar e most near l y the SAME i n
meani ng to the word/group of words gi ven i n bol d
as used i n the passage.
37. Imperative
(a) trial (b) dispensable
(c) inadequate (d) unavoidable
38. Gaining Ground
(a) obtaining land (b) making advances
(c) losing heavily (d) grounding due to losses
WWW.UPSCPORTAL.COM For Any Help Call Our Course Director:
at +91 8800734161, 011- 65023618
Click Here to Order
http://www.upscportal.com/civilservices/study-kit/ias-pre-csat-paper-2
Di r ecti ons (Q.39-45): Choose the word whi ch i s
most OP POSI TE i n the meani ng of the wor d
pri nted i n bol d as used i n the passage.
39. Expanded
(a) increased (b) extended
(c) banned (d) curtailed
40. Flourish
(a) flower (b) tarnish
(c) gain (d) deteriorate
41. Which of the following makes Anna Saheb Karve
comparable to an ascetic?
(a) His emphasis on the importance of education.
(b) His efforts in founding Bharatiya Mahila
Vidyapeeth.
(c) His liberal views and deep sympathy and
sensitivity for women and simple and clean
life.
(d) His winning the 'Bharat Ratna' Award.
42. What did Anna Saheb Karve do to prove that he
practised what he preached?
(a) He wrote articl es in 'Kesari' for creati ng
awareness among women
(b) He established an orphanage in pune.
(c) He expressed very strong views in support of
widow remarriage.
(d) He married a widow.
43. What was Karve's ideology in starting Bhartiya
Mahila Vidyapeeth?
A. Women must have greater access to and
opportunities for education.
B. Wi dows and forsaken women should get
opportunities for being self reliant.
C. Upl i fti ng of women who had remai ned
underprivileged for years.
(a) All the three (b) Both A and C
(c) Both B and C (d) Only C
44. Anna Saheb Karve was felicitated with the
'Bharat Ratna' award for
A. His recognising that educational reform alone
was the solution to all the prolems that women
faced.
B. His contribution to the education of women
and their social transformation.
C. His intellectual discussions with world famous
scientists and social reformers.
(a) Only A (b) Only B
(c) Only C (d) Only A and B
45. Which of the following statements is definitely.
TRUE in the context of the passage?
A. Before the last decade of 19th century, window
remarriages were socially unacceptable.
B. Anna Saheb Karve thought that educating
women was one of the most effective ways of
their upliftment.
C. Anna Saheb Karve's wife actively joined him
in his social service.
(a) Both A and B (b) Both B and C
(c) Both A and C (d) All of these
Di recti on (Q. 4655): I n the fol l owi ng passage,
ther e ar e bl anks, each of whi ch has been
numbered. These numbers are pri nted bel ow the
passage and agai nst each, four wor ds ar e
suggested. One of whi ch f i l l s the bl ank
appropri atel y. Fi nd out appropri ate word i n each
case.
The Reserve Bank has taken a bold (46) in the
development of money, the government securities and
the foreign exchange markets in (47) of their critical
role in overall growth and development of the economy
and (48) in the transmission mechanism of monetary
policy. The approach has been one of simultaneous
movements on several fronts, graduated and callibrated,
with an (49) on institutional and infrastructural
devel opment and i mprovements i n market
microstructure. The pace of reforms was contingent (50)
putting in place appropriate systems and procedures,
technologies and market practices. I nitiatives taken by
the Reserve Bank have brought about a (51)
transformation of various segments of the financial
market. These developments by improving the depth
and liquidity in domestic financial markets have (52) to
better price discovery of interest rates and exchange
rates, which, in turn, hae led to greater (53) in resource
allocation in the economy. The increase in size and
depth of financial market has (54) the way for (55) use
of indirect instruments.
46. (a) interest (b) participation
(c) step (d) role
47. (a) point (b) tune
(c) view (d) pursuit
48. (a) decisively (b) reluctantly
(c) visibly (d) particularly
49. (a) equilibrium (b) emphasis
(c) appeasement (d) overload
50. (a) by (b) for
(c) against (d) upon
WWW.UPSCPORTAL.COM For Any Help Call Our Course Director:
at +91 8800734161, 011- 65023618
Click Here to Order
http://www.upscportal.com/civilservices/study-kit/ias-pre-csat-paper-2
ANSWERS
1. (d) 2. (d) 3. (c) 4. (a) 5. (d) 6. (b) 7. (c) 8. (c) 9. (a) 10. (d)
11. (b) 12. (b) 13. (d) 14. (d) 15. (c) 16. (a) 17. (b) 18. (c) 19. (d) 20. (b)
21. (d) 22. (d) 23. (a) 24. (a) 25. (c) 26. (d) 27. (a) 28. (c) 29. (a) 30. (b)
31. (c) 32. (a) 33. (c) 34. (b) 35. (d) 36. (a) 37. (d) 38. (b) 39. (d) 40. (d)
41. (c) 42. (d) 43. (b) 44. (b) 45. (d) 46. (c) 47. (d) 48. (d) 49. (b) 50. (d)

Sample Chapter
of
English Language
&
Comprehension

WWW.UPSCPORTAL.COM
For Any Help Call Our Course Director:
at +91 8800734161, 011- 65023618
Click Here to Order
http://www.upscportal.com/civilservices/study-kit/ias-pre-csat-paper-2
Language is the major means by which we communicate and interact with others. When we speak or write,
we use words. These words are generally used in groups e.g.: A bad workman quarrels with his tools.
A group of words arrange din a manner which makes a complete sense is called a Sentence. Based on meaning
and sense, the sentence can be classified as :
1. Declarative or assertive 2. I mperative
3. I nterrogative 4. Exclamatory
Parts of Speech
Words are classified into different kinds or groups called Parts of Speech according to their use and function
in a sentence. They are eight in number-Noun, Pronoun, Adjective, Verb, Adverb, Preposition, Conjunction and
I nterjection.
Noun
The Noun is a word used as the name of a person, place or thing. The word thing includes (i) all objects that
we can see, touch, hear, smell or taste, and (ii) something we can think of but cannot perceive through our senses.
There are five different kinds of noun.
They are :
1. Proper Noun 2. Common Noun
3. Collective Noun 4. Material Noun
5. Abstract Noun.
Kinds of Noun
The following chart will make you familiar with the different kinds of noun :
Noun Nature of Noun Examples
Proper Noun It denotes a particular person, place or thing. Chander, Kolkata, India, the Ganges, the Gita, etc.
Common Noun It is the name given to any and Table, glass, town, king, book, river, country, etc.
every person or thing of the same class or kind.
Collective Noun It denotes a group or collection of Army, class, host, jury, mob, crowd, team, parlia
similar individuals or things considered ment, committee, family, fleet etc.
as one complete whole.
Material Noun It denotes the matter or substance of Wood, clay, rubber, iron, silver, gold, cloth, etc.
Abstract Noun It expresses quality, state or action. Truth, love, soul, mind, greatness, life, poverty,
manhood, pleasure, pain, honesty, etc.
Sentence
CHAPTER 2
WWW.UPSCPORTAL.COM
For Any Help Call Our Course Director:
at +91 8800734161, 011- 65023618
Click Here to Order
http://www.upscportal.com/civilservices/study-kit/ias-pre-csat-paper-2
Pronoun
The repetition of a noun in a sentence or a set of sentences is really boring. So, Grammar prescribes that
instead of repeating the noun, we may use a word (for that noun) called pronoun. This leads us to a precise
definition of Pronoun.
The Pronoun is a word that we use instead of a noun.
Many people commit grammatical mistakes because they lack thorough knowledge regarding the use of
pronouns.
The following facts can be stated on the basis of the above definition:
(a) A pronoun must itself be something equivalent to a noun.
(b) As a rule, the pronoun should not be mentioned until the noun has been mentioned.
(c) A pronoun must be of the same number, gender and person as the noun it stands for.
Pronouns have numerous subclasses. Though there are several features that pronouns have in common with
nouns, yet there are some features which distinguish them from nouns. They are as follows:
(a) They do not admit determiners;
(b) They often have an objective case;
(c) They often have person distinction;
(d) They often have overt gender contrast;
(e) Singular and plural forms are often not morphologically related.
Kinds of Prnouns
I. Personal Pronouns : A pronoun which is used instead of the name of a person is known as a Personal
Pronoun.
Exampl es I , my, mine, me, we, our, ours, us.(First Person)
Thou, thine, thy, thee.
You, your, yours. (Second Person)
He, his, him, she, her, hers, it, its,
they, their, theirs, them. (Third Person)
II. Reflexive or Emphatic Pronouns : When self is added to my, your, him, her, it and selves to our, your,
them, we get Compound Personal Pronouns. When the action done by the subject turns back (reflects) upon
the subject, self is added to the pronoun and becomes a Reflexive or Emphatic pronoun.
Exampl es 1. I hurt myself
2. He hurt himself.
3. They hurt themselves.
III. Demonstrative Pronouns: The pronouns which are used to point out the objects which they refer to are
called Demonstrative Pronouns.
Exampl es 1. This is a gift from my father.
2. These are merely excuses.
3. Mumbai mangoes are better than those of Bangalore.
4. That is the fort of Allahabad.
IV. Indefinite Pronouns : All pronouns which refer to persons or things in a general way and do not refer to
any particular person or thing are called I ndefinite Pronouns.
Exampl es 1. Somebody has stolen my watch.
2. Few escaped unhurt.
3. Did you ask anybody to come? 4. Nobody was there to welcome her.
WWW.UPSCPORTAL.COM
For Any Help Call Our Course Director:
at +91 8800734161, 011- 65023618
Click Here to Order
http://www.upscportal.com/civilservices/study-kit/ias-pre-csat-paper-2
V. Distributive Pronouns: Each, either, neither are called Distributive Pronouns because they refer to
persons or things-one at a time. For this reason, they are always singular and thus followed by the verb in
singular.
Exampl es 1. Each of these men received a reward.
2. Either of you can go.
3. Neither of the accusations is true.
VI. Relative Pronouns: A Relative Pronoun refers or relates to some noun going before, which is called its
Antecedent.
Exampl es 1. I met Hari who used to live here.
2. I have found the pen which I had lost.
3. Here is the book that you lent me.
VII. Interrogative Pronouns: These pronouns are used to ask questions.
Exampl es 1. Whose book is this?
2. What will all the neighbours say?
3. What do you prefer, tea or coffee?
Note: I nterrogative pronouns can also be used in asking indirect questions. Consider the following examples:
1. I asked who was speaking.
2. Tell me what you have done.
3. Say which you would like best.
Adjective
An Adjective is a word that qualifies or adds something to the meaning of a noun.
1. Madan is a good boy. (Boy of what kind?)
2. He gave me ten bananas. (How many bananas?)
3. We do not like that girl. (Which girl?)
I n sentence 1, good tells what kind of boy Madan is.
I n sentence 2, ten shows how many bananas he gave me.
I n sentence 3, that points out which girl is meant.
Some Facts About Adjectives and their Uses
Adjectives can be used in two ways-attributively and predicatively. When an adjective is placed just before the noun, its
use is Attributive, but when it is used after the noun or pronoun, the use is called Predicative.
Examples
(i) This is a sweet mango. (Attributive)
(ii) This mango is sweet. (Predicative)
(iii) Fame is the perfume of heroic deeds. (Attributive)
(iv) The judge declared him guilty. (Predicative)
(v) It is a beautiful painting. (Attributive)
(vi) The painting is beautiful. (Predicative)
(vii) Sita is a pretty girl. (Attributive)
(viii) My daughter is pretty. (Predicative)
(ix) Each boy of the class was punished yesterday.
(x) There were only two poets. Each poet recited his poem.
(xi) Every man is expected to do his duty. (xii) He comes to me every day.
(xiii) Every man in this world dies. (xiv) India wants peace in every corner of
the world.
WWW.UPSCPORTAL.COM
For Any Help Call Our Course Director:
at +91 8800734161, 011- 65023618
Click Here to Order
http://www.upscportal.com/civilservices/study-kit/ias-pre-csat-paper-2
Di recti ons: Each of the fol l owi ng questi on i s i n the
form of a sentence wi th four underl i ned porti ons
marked a, b, c and d, respecti vel y. Choose the
marked porti on that carri es errors. I f no error
requi red then your answer i s 5.
1. A tapestry consists of a foundation weave,
a
called the wrap, which across are passed
b
several coloured threads, called wefts,
c
forming decorative patterns.
d
2. The fossil remains of much extinct
a
mammals have been found in the tar pits
b c
or Rancho Brea in Los Angles.
d
3. Chemical engineering is based on the
a
principles of physics, chemists, and
b c
mathematics.
d
4. The salesman refused to show the family
a
around the showroom, till they told him
b c
what kind of a car they were looking for.
d
5. Despite the manager's comprehensive and
a
enthusiastic explanation of the scheme, we
b
were completely disinterested in the matter.
d
6. I like both further and farther as I have
a
never been been able to tell which is which
b c
and why one is further or farther than the other.
d
7. Some women acknowledge the fact that
a
their principal goal in life is to dress well,
b c
but such a principal is not guaranteed to
d
lead to spiritual salvation.
8. Political pundits have been able to correctly
a b
prophesy results of Election '98, but one
c
wonders whether the outcome will hold
good for long.
d
9. The doctor advised his young patient to
a
EXERCISE
Sentence Correction
CHAPTER 7
WWW.UPSCPORTAL.COM
For Any Help Call Our Course Director:
at +91 8800734161, 011- 65023618
Click Here to Order
http://www.upscportal.com/civilservices/study-kit/ias-pre-csat-paper-2
take two two capsful of the bitter medicine,
b
but the latter stuck to the single-cup ritual
c d
with no adverse effect.
10. Radhika and Preeti were excited about the
a
school party, as they were alumni of the
b
school, and looked forward to meeting
c d
their old friends.
11. Practising progressive muscular relaxation,
a
like a contracting and relaxing the muscles
b
of the different areas of the body strengthen
c d
the neural connections too.
12. Business magazines have been consistent
a
in saying that unless I ndian banks merged,
b c
they will lose market share.
d
13. Wrapped as they were in layer of soft cotton
a b
wool, neither of the two crystal goblets were
c
broken when the lady dropped the crate.
d
14. Although my brother had been anxiously
a b
looking forward to the prize-giving
c
ceremony for weeks, a bad-hair day prevented him
taking part in it.
d
15. There reason for I refusing the gift was that
a b
it was meant as a bribe to lure me away
c d
from the case.
16. Charles Correa, the father of modern I ndia
a
architecture, designed buildings that have
b c
earned him an international reputation and
d
influe need an entire generation of artists and
architects.
17. I n order to make bus journeys safer,
a b
smoother and more enjoyable a number of
c
operators have hired consultants to suggest
changes.
d
18. I n the middle half of the nineteen hundreds,
a
the political values taught in the public and
b
private schools did not change substantially
c
from those celebrated in the first fifty years of the
republic.
d
19. After all, the informed voter's evaluation
a
of a party depends on the extent of precise
b
information he has of its policies so that he should
gauge the relationship between
c
those policies and his conception of his own
d
welfare.
20. While the BJ P seeks a modification of the
a
World Trade Organisation rules in concert
b
with other developing nations, the Congress-I
would prefer to work with
c
industry to smoothen the transition to an era of
free trade.
d
21. The adaptation of mammals to almost all
a
possible modes of life parallels the reptiles
b
in the Mesozoic time, and except for greater
c
intelligence, the mammals do not seem to have
done much better than corresponding
d
reptilian forms.
22. Sancho ran as fast as he could walk to help
WWW.UPSCPORTAL.COM
For Any Help Call Our Course Director:
at +91 8800734161, 011- 65023618
Click Here to Order
http://www.upscportal.com/civilservices/study-kit/ias-pre-csat-paper-2
a b
his master whom he found lying on the
c
ground and not able to stir, such a blow he
d
and Rozinante had received.
23. As Caesar loved me, I wept for him, as he
a b c
was fortunate I rejoice at it.
d
24. The people of Orleans, when they first saw
a b
her in their city thought she was an angel.
c d
25. Chemical compounds with barium, cobalt
a
and strontium are responsible to many of
b c
the vivid colours in fireworks.
d
26. Plants, which make up 90 percentage of
a b
visible living organisms, get their food
c d
energy from sunlight.
27. I n the company of human beings, parrots
a
demonstrate a remarkable talent for
b
mimicry, for which they never use in the forest.
c
28. Conducting without a baton or score, the
a
leader waved his and around in order that
b c
he would have knocked the music desk off
the podium if there had been a desk, which there
wasn't
d
29. Another outstanding characteristic of water
a
is that water has a heat capacity which is
b
the hi ghest i n al l l i qui ds and sol i ds except
ammonia
30. Like a number of her white friends in
a
government, Bethune assumed that the
b
preservation of democracy and black people's full
integration into the benefits
c
and the responsiblities of American life wereinextricably tied.
d
ANSWERS
1. (b) 2. (a) 3. (c) 4. (c) 5. (c) 6. (a) 7. (d) 8. (b) 9. (b) 10. (d)
11. (d) 12. (c) 13. (d) 14. (d) 15. (a) 16. (c) 17. (c) 18. (a) 19. (c) 20. (a)
21. (b) 22.(b) 23. (d) 24. (c) 25. (c) 26. (d) 27. (d) 28. (c) 29. (b) 30. (d)
Difinition of Comprehension
Comprehension is the ability to read and understand unfamiliar text matter, to answer questions based on
that. You will be presented with passages drawn from a variety of subject areas, including humanities, the
sciences, latest happenings in society etc. The questions will ask you to analyze what is stated in the passage and
would have to identify underlying assumptions and implications. Although nothing is still clear but most probably
the passage length in CSAT will vary from 300-800 words. The passage may be followed by 3-10 questions to
check the thorough understanding of the contents. The questions will be objective type with each question followed
by four answer options out of which the most appropriate one is to be chosen on the basis of the information given
in the passage. No outside information or data should be used while answering the questions and answers should
simply be marked keeping in mind the authors viewpoints on the topic.
The passages could be written in the following styles
1. Analytical: The analytical way of writing usually presents the pros and cons of the issue at hand. I t
discusses both sides of the issue and the author may after analysis, present his final viewpoint in the end.
The questions are usually about the main idea and the authors viewpoint.
2. Discursive: A discursive style of writing is where the author discusses various aspects of certain issues in
brief and superficially. Hence, the questions that follow are simple and basic questions on theme and the
facts presented therein.
3. Argumentative: The argumentative style of writing is where the author supports his viewpoint on an
issue by presenting facts and data which support his viewpoint. Unlike the argumentative style, he does not
present both sides of the issue. The questions that follow are mostly about the authors tone and whether he
would agree or disagree with certain points on the issues as discussed in the passage.
4. Descriptive/Narrative: When the passage describes a certain event or phenomenon chances are it uses
the descriptive style of writing. I n simple words descriptive style of writing uses story telling format. I t is
lucid and easy to understand and is usually followed by easy questions on the main theme or the facts stated
in the passage.
Suggestions
1. Use only the information given or implied in a passage. Do not consider outside information, even if it seems
more accurate than the given information.
2. You are looking for the best answer, so be sure to read all the choices.
3. I f you dont know the answer, try to eliminate some choices and then take an educated guess. 4. Because
English Language
Comprehension
CHAPTER 11
WWW.UPSCPORTAL.COM
For Any Help Call Our Course Director:
at +91 8800734161, 011- 65023618
Click Here to Order
http://www.upscportal.com/civilservices/study-kit/ias-pre-csat-paper-2
Di recti on : Read the fol l owi ng passages careful l y
and answer the questi ons that fol l ow
Passage 1
Nobel prize winner J oseph E. Stiglitz, described
globalisation as a double-edged sword. For those willing
to seize the opportunities and manage globalisation on
thei r own terms, i t has provi ded the basi s of
unprecedented growth. Taking full advantage of
gl obal i sati on I ndi a has managed a hi stori cal l y
unprecedented growth rate for more than a decade and
half. Following the Washington consensus in the last
quarter of 20th century, international institutions
i ncl udi ng the World Bank and the I nternati onal
Monetary Fund mounted a sustained campaign to push
liberalisation of national economies and privatisation
of their public sector. Empirical studies have amply
demonstrated that the benefits of the globalisation have
not been shared by all the countries. Even in the same
country, the benefits arising out of globalisation have
not filtered to the various strata of the population and
disparities have widened. Thus, there are transparent
inequalities amongst the countries as also within the
same oountry. I ndia is no exception to the latter. I n
most of the poorer countries in Africa growth rates have
not registered any improvements and the number of
people below poverty line has in some cases doubled.
Moreover there is reluctance of developed countries in
removing the trade distorting subsidies in agriculture
and giving duty-free market access to the least developed
countries with very limited exportable products.
The issue is how has I ndia reaped benefits of
globalisation ? To assume that economic, fiscal, trade
and allied policies initiated by the government created
an envi ronment whi ch faci l i tated economi c
advancement on these fronts wi l l amount to
manipulation of ground realities. The foremost factor
which engineered the growth in I ndia was emergence
of a self-reliant middle class equipped with strong
knowledge base with technical qualifications. They
pursued innovative businesses requiring managerial
and technical ski ll s i n the upcoming sectors l ike
information technology and other allied fields. I nstead
of the traditional industry-led growth path followed by
the west and other developing countries I ndia opted for
services-led growth which had visible, tangible results.
I n the manufacturing sector, technological innovation,
low-cost production, ability to quickly adapt to changes,
establishing world class R & D facilities etc., greatly
helped in successfully meeting global competition.
The automobile industry is a classic example. Strong
presence of Non-Resi dent I ndi ans (NRI s) i n the
developed countries occupying senior management
positions in several multinational corporations built
confi dence i n I ndian manageri al competence and
leadership. This promoted MNCs networking with I ndia
(as also in China) with its expanding domestic market
which in the present circumstances remained the only
viable option for their sustained growth in future. The
success stories of businesses controlled by NRI s in the
western markets established I ndias reputation as
dependabl e and di sci pl i ned busi nessmen. Wel l
established democratic political framework, large young
population ingrained with absorption capacity of new
technologies have all created a responsive realisation
that I ndia is marching ahead. The initiative largely of
its private sector in expanding connectivity by improved
telecommunications, low-cost air transportation and vast
press, TV and other medi a penetrati on i gni ted
awareness amongst all sections of its people of a bright
future and thus radically changed their peroeption,
thinking and actions.
Furthermore the enti re worl d took note of
unexpl oi ted potenti al s of I ndi a i n becomi ng a
competitive centre of excellence and cost efficiency.
1. According to the author, which of the following
was primarily responsible for I ndias growth ?
(a) Well developed and implemented economic,
fiscal and trade policies
(b) Detailed microlevel analysis of the economy
(c) A workforce with managerial rather than
technical qualifications
(d) None of the above
Di recti ons (Q. Nos. 2 to 3) Choose the word whi ch
i s most nearl y the same i n meani ng as the word
gi ven i n bol d as used i n the passage.
2. mounted
(a) increased (b) grew
(c) organised (d) climbed
you may refer to the passage, dont try to memorize everything in the passage. 5. Read the passage focusing
on the main point or purpose and the structure of the passage.
EXERCISE 1
WWW.UPSCPORTAL.COM
For Any Help Call Our Course Director:
at +91 8800734161, 011- 65023618
Click Here to Order
http://www.upscportal.com/civilservices/study-kit/ias-pre-csat-paper-2
3. tangible
(a) valuable (b) complex
(c) touch (d) concrete
Di recti ons (Q. No. 4): Choose the word whi ch i s
most opposi te i n meani ng of the word gi ven i n bol d
as used i n the passage.
4. viable
(a) indirect (b) impractical
(c) unsatisfied (d) unpleasant
Passage 2
I t is difficult to imagine the extraordinary number
of control s on I ndi an i ndustry before 1991.
Entrepreneurs needed permission to invest and could
be penalized for exceeding production capacity. Even
with the given investment capacity they had, entering
certain areas was prohibited as these were reserved for
the public sector. I f they had to import anything, they
required licences. To get these licences was tough, they
had to persuade a bureaucrat that the item was required
but even so permission was unavailable if somebody
was already producing it in I ndia. The impact of the
reforms was not instantaneously and permanently
wonderful. I n I ndias case it began to show after about
a year-and-a-half. After 1993, there came three years
of rapid industrial growth of about 8% or so. But, in the
second half of the 90s, there was a tapering of industrial
growth and investment. After 1997 and the East Asian
crisis there was global slowdown which had an impact
on the I ndian industry. But, in the last few years there
has been a tremendous upturn. With the rise of
investment industrial growth has reached double digits
or close.
However, even during the period when industrial
growth was not that rapid, there is a lot of evidence
that positive results of the reforms were seen. There
were companies that didnt look at all internally but
instead performed remarkably in the highly competitive
global market. For instance, the software sectors
performance was outstanding in an almost totally global
market. Reliance built a world-class refinery. Tatas
developed an indigenously designed car. The success of
the software sector has created much higher expectations
from and much hi gher confi dence i n what I ndi an
i ndustry can do. On the governments si de i ts a
vindication that liberalization of both domestic and
external policies, including the increased inflow of
Foreign Direct I nvestment, has created an environment
in which industry can do well, has done well and is
preparing to do even better. What they need is not sops,
but good quality infrastructure. For the 11th Plan an
industrial growth rate of around 12% is projected. I t
will have methods of developing infrastructure, which
will close the deficit. This can be done through increased
investment in public sector for those infrastructure
areas, which cannot attract private investment, and
through efforts to improve private participation in
different ways of public-private participation.
I n the early stages of reforms, the liberalization of
trade policies and a shift to a market-determi ned
exchange rate had the effect of removing constraints
on agriculture in terms of depressed prices. The removal
of protection on industry helped to produce a more level
playing field, because the earlier system was extremely
unfair to agriculture. The lesson to be learnt from the
reforms process is to persevere in reforming the strategic
parts of the economy, which will lead to even higher
growth rate. I ndia has to do better than its current
average growth rate of 8% and ensure that benefits
from this higher growth go beyond industry and urban
areas and extend to agriculture.
5. Which of the following is NOT true in the context
of the passage?
A. I ndia experienced a sl owdown i n growth
during the late 1990s because initially the
economic growth rate was too rapid.
B. There were a few stringent regulations on
I ndian industries which kept economic growth
below 12%.
C. Companies did not get import licenses for goods
if they were being indigenously produced.
(a) Only A (b) Both Band C
(c) Both A and B (d) All A, B and C
Di recti ons (Q. No. 6) Choose the word whi ch i s
most nearl y the same i n meani ng as the word gi ven
i n bol d as used i n the passage.
6. extraordinary
(a) unusual (b) wonderful
(c) rare (d) beautiful
Di recti ons (Q. No. 7) Choose the word whi ch i s
most opposi te i n meani ng of the word gi ven i n bol d
as used i n the passage.
7. attract
(a) ugly (b) offend
(c) push (d) sick
WWW.UPSCPORTAL.COM
For Any Help Call Our Course Director:
at +91 8800734161, 011- 65023618
Click Here to Order
http://www.upscportal.com/civilservices/study-kit/ias-pre-csat-paper-2
Passage 3
The wakeup call that China represents to I ndia is
not limited to its showpiece urban centres or that New
Delhi hopes I ndia will experience the benefits that the
Olympic games gave brought to Beijing. More pertinent
is the comparison of the agricultural sectors of the two
countries. Why and how has China managed to outstrip
I ndia in agriculture when 25 years ago the two countries
were on par on most parameters? Both have
traditionally been agrarian economies and over half
their populations continue to depend on the land for
their livelihood. With large populations and histories of
famine I ndia and China share concern on issues such
as food security, however, while I ndias agricultural
sector is projected to grow by about 2.5 percent this year-
a slide from the previous years growth. Chinas has
been steadily growing at between 4 percent and 5
percent over the l ast fi fteen years. The wi dest
divergence between I ndia and China is in the profitable
horticultural sector with the production of fruits and
vegetables in China leaping from 60 million tonnes in
1980 compared to I ndias 55 million tonnes at the same
time, to 450 million tonnes in 2003 ahead of I ndias
correspondi ng 135 mi l li on tonnes. Chinas added
advantage lies in the more diversified composition of its
agricultural sector with animal husbandry and fisheries
which account for close to 45 percent of growth compared
to 30 percent for I ndia.
According to the latest report by Economic Advisory
council the traditional excuses for I ndias substandard
performance in the farm sector are inadequate since
I ndia is placed favourably when compared to China in
terms of quantity of arable land, average farm size, farm
mechani zation etc. The reasons for Chi na havi ng
outperformed I ndi a are threefol d : technol ogi cal
improvements accruing from research and development
(Chi na has over 1000 R & D centers devoted to
agriculture), investment in rural infrastructure and an
increasingly liberalised agricultural policy moving away
from self-sufficiency to leveraging the competitive
advantage with a focus on effi ciency as much as
equity. I nvestment in rural infrastructure, roads,
storage facilities, marketing facilities are also crucial but
government support in I ndia has been mainly been
through subsidies , not investment. There has been
much debate about subsidies and their utility; the
opposing view being that subsidies are against the
market reforms and distort the market as well as reduce
resource efficiency. I n contrast to the 2046 applications
for the registration of new plant varieties in China over
the past few years data reveals that despite I ndia having
the largest number of agricultural scientists in the world
I ndi as current research track record i s abysmal,
equivalent to what number of field crop varieties fell by
50 percent between 1997 and 2001 despite the fact that
there was sharp and sustained increase in funding. One
reason is that majority Oof the budget is eaten up by
staff salaries with only 3 percent being allotted for
research. I n contrast, most agricultural research centres
in China must use Central Government funding purely
for research. Funds relating to sal aries and other
administrative incidentals must be generated by the
centres themselves. The centres and scientists are thus,
encouraged to engage in joint ventures with private
sector companies to form commercial signoffs from their
research. I n fact research staff are now being hired on
a contact basis with pay based on performance and
salaries raised proportionately for those who perform
well. I ndia needs to learn from Chinas example and
adopt a pragmatic approach if it has to meet its targets
of the Eleventh Five Year Plan.
8. Which of the following is/are area/s in which
China has not outdone I ndia?
A. Development of urban infrastructure.
B. Activities allied to agriculture like animal
husbandry.
C. Successful bids for international sporting
events
(a) None (b) Only B
(c) Only A (d) Both A and C
Di recti ons (Q. Nos. 9 to 11) Choose the word whi ch
i s most nearl y the same i n meani ng as the word
pri nted i n bol d as used i n the passage.
9. concern
(a) importance (b) regret
(c) business (d) anxiety
10. devoted
(a) surrendered (b) dedicated
(c) established (d) staunoh
11. pertinent
(a) intense (b) sizeable
(c) practical (d) significant
Passage 4
Though it is common place to say we live in a
gl obal i sed worl d, l ess wel l understood i s that
globalisation is taking place in stages. I n the first stage
WWW.UPSCPORTAL.COM
For Any Help Call Our Course Director:
at +91 8800734161, 011- 65023618
Click Here to Order
http://www.upscportal.com/civilservices/study-kit/ias-pre-csat-paper-2
as flows of capital and goods were liberated the benefits
of globalisation, eg, technological advancements, flowed
primarily to the developed world. As we enter the current
newer age of mobility, people have begun to move across
borders in great numbers in pursuit of economic security
and a better life for themselves and to keep their families
out of poverty. At the turn of the 20th Century the
United Nations estimates that approximately three
percent of the worlds population is on the move, a similar
scale to that witnessed in earlier eras. Growing economic
inequality together with natural and man made crises
prompts emigration. But this mobility has the potential
to chip away at the vast inequalities that characterise
our time and accelerate progress throughout the
devel opi ng worl d. To take an exampl e, l ast year
migrants sent home $264 billion triple all international
aid combined. The freer movement of people helps to
underwrite health care, education and grass roots
entrepreneurship across the developing world.
I t .hel ps to oi l the gl obal economy. When
industrialised nations need to recruit nurses or software
programmers developing countries are often the source.
Yet rather than look at how these gains can be harnessed
to reduce poverty governments have been slow to adapt.
The result is burgeoni ng ill egal i mmigration, and
trafficking, social tension and intolerance, loss of faith
in the government and the empowerment of criminal
networks. Until a few years ago, migrants were paying
exorbitant fees to send money home losing about 20%
in transaction costs. Today though migrants move freely
and easily thanks to easy access to low cost transport
and the internet, affordable and rapidly advancing
telephony and satellite television which keep them in
constant touch with home. Banks easily and wirelessly
transmit hard won and sometimes meagre salaries
instantly to their families. The flow of people until now
mostly benefited richer countries and generated worries
about brain drain and the violation of migrant rights
in poorer ones. Global fora represent a step in the effort
to harness the power of mi grati on to advance
development and increase our knowledge of how to
make the migration equation work for all. Rather than
focus on the negative consequences and recriminations
of exploitation by developing countries such summits
provide an opportunity for all nations to come together
and address these issues in a comprehensive, logical and
rational way so that the benefits of migration are fully
realised in both developing and industrialised countries.
Countries may examine how dual citizenship laws can
ease the way for migrants to play a bigger role in
development by bringing their capital, knowledge and
networks back home. Such efforts will usher in the third
stage of globalisation where everyone can share in the
worlds prosperity.
12. Which of the following is TRUE in the context of
the passage?
(a) The world is currently in the final stage of
globalisation
(b) The aim of summits on migration is to curb
migration and prevent brain drain
(c) Governments have not fully exploited the
potential benefits of migration
(d) Currently reforms are needed as I nternational
remittances are marginal
Di recti ons (Q. Nos. 13 to 14) Choose the wor d
whi ch i s most nearl y the same i n meani ng as the
word pri nted i n bol d as used i n the passage.
13. harnessed
(a) mobilised (b) control
(c) pushed (d) pull
14. accelerate
(a) further (b) quick
(c) open (d) rapid
Passage 5
Though the last twenty-five years have seen China
dazzle the world with its excellent economic performance
it has shied away from playing the kind of active role in
international affairs that would seem commensurate
with its economic weight. This is because traditionally
Chinas politics have been defined by the need for
economic development above all else. I n the past Chinas
authorities have tended to downplay the countrys
international clout, choosi ng to stress instead i ts
devel opi ng country status and l i mi ted mi l i tary
capabilities. Such modest rhetoric was intended to allay
the fears that Chi nas rise was causi ng across its
immediate neighbourhood. That Bei jing is final ly
acknowledging its status as a major player in the
international system is evidenced by the fact that the
presi dent has formal l y devel oped a theory of
international relations; the concept of harmonious world.
The concept, encompassi ng broad noti ons of
multilateralism, prosperity for all through common
development and tolerance for diversity has left world
opinion perplexed. These are commendable objectives
but the theory is short on specifics regarding the means
to achieve them.
WWW.UPSCPORTAL.COM
For Any Help Call Our Course Director:
at +91 8800734161, 011- 65023618
Click Here to Order
http://www.upscportal.com/civilservices/study-kit/ias-pre-csat-paper-2
Di r ecti ons (Q. Nos. 16 to 18) Choose the wor d
whi ch i s most si mi l ar i n meani ng to the word
pri nted i n bol d as used i n the passage.
16. sustained
(a) sporadic (b) confirmed
(c) steady - (d) supported
17. potential
(a) promise (b) talent
(c) opportunity (d) possible
18. encompassing
(a) surrounding (b) incorporating
(c) enveloping (d) accepting
Di recti ons (Q. No. 19) Choose the word whi ch i s
most opposi te i n meani ng to the word pri nted i n
bol d as used i n the passage.
19. commendable
(a) valuable (b) unreliable
(c) undeserved (d) unworthy
Passage 6
Thinking of what education might look like in the
next decade, one quickly realizes that the trends in
technology are leaving a large number of our students
behind. Today is an age of exponential change. New
and ever-improving technologies are popping up every
day and in every corner of society.
Educating the best and the brightest in this brave
new world will take a new and improved educational
paradigm. Allowing our educational tools to age in the
corner of the classroom will be the mistake that may
cost us our future. Throwing away masses of children
to inequitable access will ensure that we languish at
the bottom of the global pool of employable workers for
decades to come.
Technology will shape the way we educate students
in the next decade. A user is not simply a person who
uses. For the student, being a user should involve using
the latest technology in a free and autonomous manner.
This new-found freedom will allow the student to become
an active participant in his/her education instead of a
passive passenger. I n our current technological society,
being a user also means being tracked. Tracking a
student means having the ability to target education
towards weaknesses and strengths. The abi lity to
accurately customize to the individual has been the holy
grail of educational philosophy for many years. This
golden age of technological development may soon
enable this dream to become a reality.
Chinas recent willingness to be a more active
player internationally stems from complex factors. The
countrys economic strength-having acquired the largest
foreign exchange reserves in the world is undeniable
and reports favour it to be the largest economy in the
next quarter of a century. For sustained double digit
economic growth China thus has no choice but to become
more active i nternati onally. Moreover as a major
proportion of the oil and other natural resources that
China needs to feed its growing economy are imported
Beijing has to aggressively woo the countries rich in
energy resources, whi ch al so represent emerging
markets for Chi nese products. To ensure a stable
securi ty envi ronment withi n the region and thus
facilitate economic growth China played an active role
i n faci l i tati ng negoti ati ons wi th North Korea.
Destabilization of a potential flashpoint like the Korean
peninsula would lead to a flood of refugees crossing the
border, i nterrupti ng careful pl ans of economi c
rejuvenation of Chinas North-East. Chinas growing
influence has caused a shift in the geopolitical status
quo and its influence is beginning to replace that of the
United States and European powers in Africa. Chinas
new diplomacy though has had its share of critics who
have expressed thei r unease at Chi nas mil i tary
modernization programme and its willingness to deal
wi th regi mes wi del y condemned as corrupt and
oppressive. Despite this when the Africa was in need of
aid and infrastructure or the US needed hel p i n
negotiating with Korea they turned to China. By taking
a lead in a variety of international and regional forums,
i ni ti ati ng bi l ateral and mi l i tary exchanges and
dispensing aid and technical assistance in parts of the
world where traditional powers are cautious to tread
China has signalled that its days of sitting on the
sidelines, content to let others shape world affairs are
emphatically over.
15. Which of the following is TRUE in the context of
the passage ?
(a) Chi nas current pol i ti cal standi ng
i nternati onal l y i s disproportionate to its
financial strength
(b) China is a reluctant participant in military
dialogues
(c) The harmonious world theory is the only
utilitarian remedy to the current challenges
facing the world
(d) The US has recognized and acknowledged
Chinas growing international reputation
WWW.UPSCPORTAL.COM
For Any Help Call Our Course Director:
at +91 8800734161, 011- 65023618
Click Here to Order
http://www.upscportal.com/civilservices/study-kit/ias-pre-csat-paper-2
ANSWERS
1. (d) 2. (c) 3. (d) 4. (b) 5. (a) 6. (a) 7. (b) 8. (a) 9. (a) 10. (b)
11. (d) 12. (c) 13. (a) 14. (d) 15. (b) 16. (c) 17. (d) 18. (b) 19. (d) 20. (d)
21. (a) 22. (d) 23. (b) 24. (d) 25. (c) 26. (c) 27. (b) 28. (b) 29. (b) 30. (a)
31. (b) 32. (c) 33. (a) 34. (d) 35. (b) 36. (c) 37. (d) 38. (b) 39. (a) 40. (b)
41. (d) 42. (c) 43. (c) 44. (b) 45. (b) 46. (d) 47. (b) 48. (b) 49. (d) 50. (c)
51. (a) 52. (d) 53. (d) 54. (d)
WWW.UPSCPORTAL.COM
For Any Help Call Our Course Director:
at +91 8800734161, 011- 65023618
Click Here to Order
http://www.upscportal.com/civilservices/study-kit/ias-pre-csat-paper-2

Sample Chapters
of
Interpersonal &
Communication
Skills

WWW.UPSCPORTAL.COM
For Any Help Call Our Course Director:
at +91 8800734161, 011- 65023618
Click Here to Order
http://www.upscportal.com/civilservices/study-kit/ias-pre-csat-paper-2
Meaning of Communication
Transmission of meaning from one person to another or to many people is communication, whether verbally
or non-verbally.Communication from one person to another is commonly depicted as a simple triangle consisting
of the context, the sender, the massage, and the receiver.I n the work area Communication skills have great
importance just as they have in all areas of life.Communication skills such as being able to express one or to
understand the others correctly are required for success and satisfaction at least in elementary level for most of
the professions. Even though an organization performs its tasks more efficient than expected, one should know
that this success will not continue so long if it lacks the same efficacy in communication. The success of communication
depends on several factors. Transmission of message by the process of encoding and decoding the message, which
may result in short-term perception, is not adequate for the success of communication. Communication should be
in such a way that will give opportunity for the respondent or receiver to take decision with regard to the message
he perceived.
Meaning of Interpersonal Communication
The concept of I nter Personal Communication (I PC) skills was introduced primarily in 1950s. Ability to work
well with people, and involve your acceptance of others, without prejudice is called I PC skills. Ability to respond to
staffs needs positively, fostering a non-discriminatory work environment where staff can develop to their full
personal potentials, and delegating authority I nterpersonal communication competence consists of a set of skills,
knowledge about communication, and self-evaluation is I PC skills. The skills involved in good interpersonal
communication include
1. Demonstrating caring, concern and commitment
2. Listening and Understanding
3. Problem solving and Motivating.
You can demonstrate that you care by expressing your understanding of the feelings and concerns of the
other person and by letting them know that you want to help them. You can reflect the other persons emotions
back to them with facial expressions that show you are concerned. You can also provide verbal feedback to them
to show acknowledgement and recognition of their fears and concerns.Listening and understanding involve more
than simply being present while someone is speaking. Active listening means genuinely hearing the other persons
words. Often, we think we are listening, but we actually do not pay close attention or do not really hear what the
other person is trying to say.
Everybody has their own & unique Ability and strengths. I nspite of how bad you might think of yourself,
you are special, unique and one of a kind.Trust in your abilities,recognise your strengths and succeed in life. On
Interpersonal &
Communication Skills
WWW.UPSCPORTAL.COM
For Any Help Call Our Course Director:
at +91 8800734161, 011- 65023618
Click Here to Order
http://www.upscportal.com/civilservices/study-kit/ias-pre-csat-paper-2
the flip side, do recognise that even the weakest individual is better than you or me in some way or another. I t
may be intelligence or kindness or generosity or even grace.
Emerson once said: Every man is my superior in some way. I n that case, I learn of him.
Almost every man you meet feels himself superior to you in some way, and a sure way to win a persons heart
is to let him or her realize that in some subtle way, you recognize his importance in his little world, and recognize
it sincerely.
Salient features of IPC skills
v Think positively, and enter the mindset to work well with others and maintain good relationships. Do not
criticise others or yourself.
v Be sensitive to others, this includes not gossiping.
v Be patient.
v Be cheerful and try to make others smile.
v Treat others and their experience with respect.
v Learn to listen, experts recommend listening 80% of the time and only talking 20%.
v I f youre not naturally confident or happy, fake it until you generally possess the desired characteristics.
v Praise and compliment people when they deserve it,
v When someone is telling a story, dont interrupt or try to upstage them with a story of your own.
v Treat your team members and colleagues as friends and not as strangers or subordinates.
v When youre unhappy, try your best to act happy anyway. You will end up feeling better and so will the
people around you, your mood is contagious.
v Smile - even when you dont feel like smiling.
v Look for solutions.
v When someone compliments you, dont disagree or boast about it simply say thank-you with a smile and
move on.
v Learn to appreciate, be helpful and not demotivate your team members. Work as a team, not as an individual.
This will achieve better results.
Behaviour Psychology
Deal with people as though they are your client or boss and convey the message you want in a proper
manner. (This includes rhythm of voice to make them comfortable with you.)
Beatrice Vincent once said: The people with whom you work reflect your own attitude. I f you are suspicious,
unfriendly and condescending, you will find these unlovely traits echoed all about you. But if you are on your
best behaviour, you will bring out the best in the persons with whom you are going to spend most of your working
hours.
Robbins and Hunsaker (2003) reviewed a large number of studies and synthesized the I PC skills that
surfaced on most lists Most of- these skills. belong to three categories - leadership, the process of communication
and motivation .I nterpersonal skills under leadership relate to leadership style, handling conflicts, running meetings,
team building and promoting change. The process of communication includes sending messages, listening and
providing feedback Similarly, motivating is broken down into goal setting, clarifying expectations, persuading
and empowering. Other interpersonal skill include negotiating. The dimensions and components of above model
are shown in the following table:
Concept Dimension Component
Interpersonal communication skills (IPC) 1. Process of communication 1. Effective communication
2. Communication styles
3. Building relationships
WWW.UPSCPORTAL.COM
For Any Help Call Our Course Director:
at +91 8800734161, 011- 65023618
Click Here to Order
http://www.upscportal.com/civilservices/study-kit/ias-pre-csat-paper-2
2. Motivation 1. Goal setting
2. Clarifying expectations
3. Self development
4. Stress management
5. Emotinal intelligence
3. Leadership 1. Leadership style
2. Values and ethics
3. Handling conflicts
4. Promoting change
Steps to Develop Your IPC Skills?
1. Networking
I n additionto strong communication Skills and Personal Skills, Networking uses the Background skills of
network building and motivating others. involves working with others ina group towards a common goal. This
requires cooperating with others, being responsive to others ideas, taking a collaborative approach to learning,
and taking a responsibility for developing and acheving group goals.The ability to actively seek, identify and
create effective contacts with others, and to maintain those contacts for mutual benefit.Teamwork uses the
Background skills of acollaboration, mentoring, decisionmaking and delegation.
2. Leadership
The process of successfully influencing the activities of a group towards the achievement of a common goal is
called as leadership.As well as requiring strong communication Skills and Personal Skills, leadership uses the
Background skills of mentoring, decision making, delegation and motivating others.A leader has the ability of
influence others through qualities such as personal charisma, expertise, command of language, and the creation
of mutual respect.
1. Leadership
Mentoring
Decision Making
Delegation
Motivating others
2. Networking
Self confidence
Network building
Effective
communication
3. Teamwork
Mentoring
Groupwork
Delegation
Collaboration
Interpersonal Skills
WWW.UPSCPORTAL.COM
For Any Help Call Our Course Director:
at +91 8800734161, 011- 65023618
Click Here to Order
http://www.upscportal.com/civilservices/study-kit/ias-pre-csat-paper-2
2. Background skills
(i) Decision making
v I dentifying appropriate evidence and weighing up that evidence to make a choice for example, gathering
and assessing information to find the best way to perform am experiment).
v Taking responsibility for a decision and its outcomes (for example, choosing a topic for a group
presentation from a number of suggestions).
(ii) Mentoring
v Being a trusted advisor and helper with experience in a particular field. Actively supporting and
guiding someone to develop knowledge and experience, or to achieve career or personal goals (for
example, a third-year student mentoring a first year student, helping to adjust tothe university
experience).
v A mentoring relationship may be formal or informal, but must involve trust, mutual respect, and
commitment as both prties work together to achieve a goal (for example, mentoring a younger member
of a team to achieve better performance in the lead-up to a sporting event).
(iii) Group work
v any activity inwhich sutdents work together;
v any activity which has been specifically designed so that students work impairs or groups, and may be
assessed as a group (referred to as formal group work); or
v when students come together naturally to help each other with their work (referred to as informal
group work).
v peer group activity in lab classes, tutorials etc.
(iv) Group work
v any activity inwhich sutdents work together;
v any activity which has been specifically designed so that students work impairs or groups, and may be
assessed as a group (referred to as formal group work); or
v when students come together naturally to help each other with their work (referred to as informal
group work).
v peer group activity in lab classes, tutorials etc.
(iv) Motivating others
v Generating enthusiasm and energy by being positive, focussing on finding solutions and maintaining
a positive attitude even when things are not going well (for example, when something goes wrong,
asking What can we try now instead of saying, That should have worked better.).
v Encouraging others to come up with solutions, listening carefully to their ideas and offering constructive
feedback (for example, gathering suggestions for a group project, and giving each persons ideas fair
discussion).
v Being prepared to support others in taking agreed, calculates risks, and not blaming others when
things go wrong (for example, one group members portion of a presentation receives a poor mark-
make sure that this student isnt blamed by the group, and focus on learning from the mistakes).
(v) Delegation
v Taking responsibility for determining when to ask someone else to make a decision or carry out a task
(for example, figuring out what is a fair distribution of the workload in a group project, and sharing
responsibility with others).
v Distributing responsibility and authority in a group by giving someone else the discretiontomake
decisions that you have the authority to make (for example, as the chosen leader of a lab experiment
team, you could assign tasks and decisions to different group members).
WWW.UPSCPORTAL.COM
For Any Help Call Our Course Director:
at +91 8800734161, 011- 65023618
Click Here to Order
http://www.upscportal.com/civilservices/study-kit/ias-pre-csat-paper-2
(vi) Collaboration
v Working cooperatively and productively with other team members to contribute to the outcomes of the
teams work (for example, dividing the workload and sharing the results of your own work with others
in the group, or assisting members of the group who are having difficulty completing their tasks).
(vii) Network building
v Creating contacts with other people and maintaining those contacts (for example, meeting someone at
a seminar with similar interests, and swapping email addresses with them).
v Acquiring and maintaining information about people who might be useful contacts for specific purposes
(for example, seeking out people established in an industry you hope to work with one day).
v Using a contact in an ethical manner to help each of you meet specific goals, (for example, collaborating
on projects of importance to both of you).
3. Networking
The ability to actively seek, identify and create effective contacts with others, and to maintain those contacts
for mutual benefit. I n additionto strong communication Skills and Personal Skills, Networking uses the Background
skills of network building and motivating others. involves working with others ina group towards a common goal.
This requires cooperating with others, being responsive to others ideas, taking a collaborative approach to learning,
and taking a responsibility for developing and acheving group goals. Teamwork uses the Background skills of
acollaboration, mentoring, decisionmaking and delegation.
Types of Leadership
Management experts have undergone a revolution in how they define leadership and what their attitudes
are toward it,in the past several decades. They have gone from a very classical autocratic approach to a very
creative, participative approach. Somewhere along the line, it was determined that not everything old was bad
and not everything new was good. Rather, different styles were needed for different situations and each leader
needed to know when to exhibit a particular approach.
Four of the most basic types ofleadership are:
A. Democratic
B. Laissez-faire
C. Autocratic
D. Bureaucratic
Although good leaders use all the styles, with one of them normally dominant, bad leaders tend to stick with
one style.
A. Democratic Leadership Type
The democratic leadership type encourages employees to be a part of the decision making therefore it is also
called the participative type. The democratic manager keeps his or her employees informed about everything that
affects their work and shares decision making and problem solving responsibilities. This style requires the leader
to be a coach who has the final say, but gathers information from staff members before making a decision.
Democratic leadership can produce high quality and high quantity work for long periods of time. Many
employees like the trust they receive and respond with cooperation, team spirit, and high morale. Typically the
democratic leader :
1. Develops plans to help employees evaluate their own performance
2. Recognizes and encourages achievement.
3. Encourages employees to grow on the job and be promoted
4. Allows employees to establish goals
WWW.UPSCPORTAL.COM
For Any Help Call Our Course Director:
at +91 8800734161, 011- 65023618
Click Here to Order
http://www.upscportal.com/civilservices/study-kit/ias-pre-csat-paper-2
The democratic type is not always appropriate,like the other type. I t is most successful when used with
highly skilled or experienced employees or when implementing operational changes or resolving individual or
group problems. The democratic leadership type is most effective when :
1. The leader wants employees to share in decision-making and problem-solving duties.
2. The leader wants to keep employees informed about matters that affect them.
3. The leader wants to provide opportunities for employees to develop a high sense of personal growth and job
satisfaction.
4. Changes must be made or problems solved that affect employees or groups of employees.
5. There is a large or complex problem that requires lots of input to solve.
Democratic leadership should not be used when
1. The manager feels threatened by this type of leadership.
2. There is not enough time to get everyones input.
3. Employee safety is a critical concern.
4. I ts easier and more cost-effective for the manager to make the decision.
B.Laissez-Faire Leadership Type
Laissez-Faire type is one, in which the manager provides little or no direction and gives employees as much
freedom as possible. All authority or power is given to the employees and they must determine goals, make
decisions, and resolve problems on their own.This leadership type is also known as the hands-off type.
This is an effective type to use when :
1. Employees are highly skilled, experienced, and educated.
2. Employees have pride in their work and the drive to do it successfully on their own.
3. Outside experts, such as staff specialists or consultants are being used
4. Employees are experienced & trustworthy.
This style should not be used when
1. Managers are unable to thank employees for their good work.
2. I t makes employees feel insecure at the unavailability of a manager.
3. The manager doesnt understand his or her responsibilities and is hoping the employees can cover for him or
her.
4. The manager cannot provide regular feedback to let employees know how well they are doing.
C. Autocratic Leadership Type
Autocratic Leadership type is one in which the manager retains as much power and decision-making authority
as possible. The manager does not consult employees, nor are they allowed to give any input.This is often considered
the classical approach. Employees are expected to obey orders without receiving any explanations. The motivation
environment is produced by creating a structured set of rewards and punishments.
This leadership type has been greatly criticized during the past 30 years. Some studies say that organizations
with many autocratic leaders have higher turnover and absenteeism than other organizations.These studies say
that autocratic leaders:
1. Rely on threats and punishment to influence employees.
2. Do not trust employees.
3. Do not allow for employee input.
Yet, autocratic leadership is not all bad. Sometimes it is the most effective type to use. These situations can
include:
WWW.UPSCPORTAL.COM
For Any Help Call Our Course Director:
at +91 8800734161, 011- 65023618
Click Here to Order
http://www.upscportal.com/civilservices/study-kit/ias-pre-csat-paper-2
1. A managers power is challenged by an employee.
2. New, untrained employees who do not know which tasks to perform or which procedures to follow.
3. Work needs to be coordinated with another department or organization.
4. Effective supervision can be provided only through detailed orders and instructions.
5. Employees do not respond to any other leadership type.
6. There are high-volume production needs on a daily basis.
The autocratic leadership type should not be used when
1. Employees begin depending on their manager to make all their decisions.
2. Employees become tense, fearful, or resentful.
3. There is low employee morale,absenteeism and work stoppage.
4. Employees expect to have their opinions heard.
D.Bureaucratic Leadership Type
Where the manager manages by the book and everything must be done according to procedure or policy is
Bureaucratic leadership.I f it isnt covered by the book,the manager refers to the next level above him or her. This
manager is really more of a policeofficer than a leader. He or she enforces the rules.
This type can be effective when :
1. Employees are performing routine tasks over and over.
2. Employees are performing tasks that require handling cash.
3. Employees need to understand certain standards or procedures.
4. Safety or securitytraining is being conducted.
5. Employees are working with dangerous or delicate equipment that requires a definite set of procedures to
operate.
This style is ineffective when
1. Work habits form that are hard to break, especially if they are no longer useful.
2. Employees lose their interest in their jobs and in their fellow workers.
3. Employees do only what is expected of them and no more.
Varying Leadership Types
While the proper leadership type depends on the situation, there are three other factors that also influence
which leadership style to use.
1. The company, the traditions, values, philosophy, and concerns of the company will influence how a manager
acts.
2. The managers personal background.Employees are individuals with different personalities and backgrounds.
3. The employees being supervised. The leadership type,managers may use will vary, depending upon the
individual employee and what he or she will respond best to.
VALUES AND ETHICS
Values mostly remain predictable, they are consistent, even in volatile times. A clear set of values helps tell
people what their lives are for and what is worth working for. Values tell people what is good and important and
values bind society. Social scientists believe that without values, a society could disintegrate, a risk often present
in I ndia. Religious heads believe that without values, human life is meaningless and all the worldly pleasures will
not lead to any satisfaction. Yes, a lack of good values is why scams happen, nepotism exists and the government
doesnt care about its people.To any society and human being core values are vital.

Sample Chapters
of
Decision Making
&
Problem Solving

WWW.UPSCPORTAL.COM
For Any Help Call Our Course Director:
at +91 8800734161, 011- 65023618
Click Here to Order
http://www.upscportal.com/civilservices/study-kit/ias-pre-csat-paper-2
Matters at Stake
Good problem solving and decision making can avert tragedy and help the people recover from the event
more quickly. Conversely, poor decision making? or the absence of decisions? potentially can result in injury or
death to victims or responders. (Clearly, in our case study, if the District Magistrate makes a poor decision-or
simply doesnt make any decision before its too late-the consequences could be disastrous for the people of Murariganj
and Dumdama.
But the repercussions dont stop there. Poor decisions in the early stages of an event can make the responders
job more difficult and more dangerous. I n addition, they can give rise to much more critical or complex decisions
later on? not to say the consequences on the lives of the local people. Good decision-making skills are one of your
most critical assets as an Administrator, and CSAT will test on this account before you step to the next stage for
the selection to top administrative jobs in I ndia. I n this book , we shall familiarize you with what CSAT will
actually attempt to test in you.We take here a case study to understand this aspect of making decisions in emergency.
Making Decisions in Emergency
Case Study* - Floods Fury and Koshis Wrath in North Bihar
Instructions: Read the following case study. As you read, try to identify what decisions must be made by the District
Magistrate or other emergency management officials. Mind your ideas as you read the situation.
Background: Heavy rains have hit, and the entire area of the Koshi belt in the North Bihar bordering Nepal is inundated
due to change of course of Koshi. The district town of Sakri-located in the North Biharsits high and well away from the
fury of Koshi, so the fear of floods is riot much of concern of the people in the Town. As the DM of Sakri district, last year
you got a new chute(gutter, passageway) built to increase the capacity of the Reservoir in Sakri to 35 Crore litres of
water. Two other subdivisional towns downstream-Murariganj and Dumdama are less than 15 minutes of drive from Sakri.
Each Town has about 10,000 residents on both the sides of National Highway 77.
Events Chronology
Late afternoon Rains begins, and the Meteorological Office at Patna predicts it will be a strong, pachhuwa (Westerli(e),
which will produce heavy rains.
8:12 p.m. A flash flood watch is issued by the Meteorological Office.
8:30 p.m. Heavy rains begin.
10:15 p.m. The Executive Engineer of Sakri stations a Junior Engineer on the dam to watch for, and report any problems.
The Junior Engineer sees water pouring a good 2 feet over the chute. (It was later estimated that the reservoir was
holding 55 Crore litres of water during and after the storm!)
11:30 p.m. Six inches of rain have fallen over the last 3 hours.
Decision Making and
Problem Solving
WWW.UPSCPORTAL.COM
For Any Help Call Our Course Director:
at +91 8800734161, 011- 65023618
Click Here to Order
http://www.upscportal.com/civilservices/study-kit/ias-pre-csat-paper-2
1:15 a.m. The Junior Engineer sees a section of dirt break away.
1:30 a.m. When water recedes below the top of the dam, Junior Engineer and other workers discover that water has
eaten around the drainage and is gradually carving away the side of the earthen dam. A first attempt at closing the hole
with sandbags fails when the force of the water carries the bags right through.
1:45 a.m. As the DM of Sakri , you are now in meeting with the Executive Engineer of Sakri, the PWD Engineer, the SP,
the Chief Medical Officer of Sakri, and the District Disaster Cell Manager to discuss the situation.
* The names of some of places and other recognized symbols could be fictitious and resemblances to events already
occurred or occurring to the situations described here are only coincidental. This is just an academic reconstruction.
Decision Points
What decision points did we identify in the above situation? I t is to be noted that the central problem is that
if the dam breaks, the people in the downstream towns will be flooded. Numerous decisions must be made to
address this problem. Some of the key decisions that must be made are as follows, and we probably identified
others as well.
1. Should residents in the two downstream subdivisional towns of Murariganj and Dumdama be evacuated?
2. I f the decision is to evacuate, when should the evacuation take place?
3. Who will notify the populace of the evacuation, and how?
4. What additional resources will be necessary to accomplish the evacuation and provide any shelter needed?
5. What is the most effective way to keep the dam from collapsing, and what resources will be needed to
accomplish that?
DECISION MAKING PROCESS
Whether making ordinary day-to-day decisions or critical, time-sensitive decisions during an emergency,
using a standard problem-solving model will help ensure that your decisions are rational and logical.
Problem Solving VS Decision Making
Lets first clarify what we mean by problem solving and decision making and how they go together:
Problem solving is a set of activities designed to analyze a situation systematically and generate, implement, and
evaluate solutions.
Decision makings a mechanism for making choices at each step of the problem-solving process.
Decision making is part of problem solving, and decision making occurs at the every step of the problem-solving
process.
Problem-Solving Model
There are many different decision-making/problem-solving models that a decision making person can use.
Following is a five-step model which may be used in an emergency situation.
Step 1 I dentifying the problem
Step 2 Exploring the alternatives
Step 3 Selecting the best alternative
Step 4 I mplementing the solution
Step 5 Evaluating the solution situation
When using this model, each step may be completed quickly, but every step must be considered. I t is not
necessary to document each step, but it is important to think through every step.
WWW.UPSCPORTAL.COM
For Any Help Call Our Course Director:
at +91 8800734161, 011- 65023618
Click Here to Order
http://www.upscportal.com/civilservices/study-kit/ias-pre-csat-paper-2
Step 1 Identifying the Problem
Problem identification is undoubtedly the most important? and the most difficult? step in an situation. Whether
all subsequent steps are productive or not, will be based on how you define and assess the problem at hand.
What i s a Probl em?
Situation or condition of people or the organization that will exist in the future, and that is considered
undesirable by members of the organization is called a Problem.
Probl em or Sol uti on?
We must distinguish between a problem and its solution in carrying out Step 1. Defining problems in terms
of their solutions is the most common error in problem solving. Sometimes people think that they are articulating
problems when actually they are stating a potential solution.
Marki ng Out the Probl em Parameters
v I dentifying the problem also involves analyzing the situation to determine the complete extent of the problem.
Problem parameters include :
v What is happening (and is not happening)?
v Who is/are involved?
v What the stakes are?
Checklist for Identifying, Defining, and Analyzing Problems
1. Is this a new problem? (Answer in Yes or No)
2. Is the problem clearly and precisely stated? (Answer in Yes or No)
3. What assumptions are We making about the problem? Are they true? (Answer in Yes or No)
4. What would happen if nothing were done about this problem? (Answer in Yes or No)
5. Can the problem be restated in other terms? If yes, how? (Answer in Yes or No)
6. What data are Known that bear on the problem?
7. Is the information accurate? (Answer in Yes or No)
8. Are there any precedents or rules about other procedures that apply to the problem?
If so, what precedents or rules apply? (Answer in Yes or No)
9. What additional facts are needed to analyze the problem? ( Make a List)
10. Is it possible to interpret the facts differently? How would that affect the problems solution? (Answer in Yes or No)
11. Do I have to make this decision, or does someone else? If this decision is someone
elses to make, whose is it? (Answer in Yes or No)
Step 2 Exploring the Alternatives
To explore alternative solutions to the problem identified in Step 1 is the second step in the decision-making
process. This step really consists of two parts
v Generating alternatives
v Evaluating alternatives
Methods of Generati ng Al ternati ves
So, what process should we use to generate the alternatives for the case study? There are three ways to
generate alternatives.
v Brainstorming can be done individually or in a group. Brainstorming requires an environment in which the
participants (individuals or group members) are free to think out loud. Participants blurt out as many
ideas as possible within a specified time period.
WWW.UPSCPORTAL.COM
For Any Help Call Our Course Director:
at +91 8800734161, 011- 65023618
Click Here to Order
http://www.upscportal.com/civilservices/study-kit/ias-pre-csat-paper-2
Surveys economically tap the ideas of a large group of respondents. Surveys present respondents with the
problem and a series of alternative solutions.
Discussion groups should consist of those who are directly involved in decision making. I n generating
alternatives, the group members should
1. Focus on the problem, not on the personalities of the people involved in the decision-making process.
(But be sensitive to the impact of personalities on the process.)
2. Avoid initial judgments (as in brainstorming).
3. Be comprehensive.
Criteria for Evaluating Alternatives:We must have some means of evaluating them,after we have
generated alternative solutions. We should lists criteria by which we can evaluate alternatives. Another part of
evaluation is identifying contingencies-what could go wrong. Think in terms of Murphys Law (I f anything can
go wrong, it will.) and identify what could get in the way of solving the problem you are facing.
Criteria for Evaluating Alternatives
Questions to Ask
1. Identify Constraints, Do any of the following factors serve as a limitation on this solution?
Human resources (limited ability of relevant people to understand or initiate certain actions)
Social (restrictions imposed by organized groups with special interests)
Political (legal restrictions or ordinances)
Economic (cost or capital restrictions)
Technical (limited equipment or technology)
Time (requirements that a solution be found within a prescribed time period, thereby eliminating consideration
of long-range solutions)
2. Determine Appropriateness Does this solution fit the circumstances?
3. Verify Adequacy Will this option make enough of a difference to be worth doing?
4. Evaluate Effectiveness Will this option meet the objective?
5. Evaluate Efficiency What is the cost/benefit ratio of this option?
6. Determine Side Effects What are the ramifications of this option?
Step 3 Selecting the Best Alternative
To select one of the alternatives explored in Step 2 for implementation is third step in the problem-solving
model. After you have evaluated each alternative, one should stand out as coming closest to solving the problem
with the most advantages and fewest disadvantages.
I mplementing the solution may not be easy, however. There may be repercussions, and you should complete
a reality check to identify and evaluate the possible consequences of implementing the solution. Carefully consider
how the solution will be implemented before selecting an alternative.
Selecting an alternative is a critical step in the problem-solving process. When selecting an alternative, you
will encounter factors that affect your decision making. These factors may include:
Ethical factors.
Financial factors.
Political factors.
Environmental considerations.
Safety factors.
All of these factors may not be readily recognizable. Be alert for these potential limits on the solutions that
you can implement, as you examine the situation and apply the problem-solving model, .
WWW.UPSCPORTAL.COM
For Any Help Call Our Course Director:
at +91 8800734161, 011- 65023618
Click Here to Order
http://www.upscportal.com/civilservices/study-kit/ias-pre-csat-paper-2
Step 4 Implementing the Solution
The fourth step involves five subparts.
1. Develop an action plan I mplementation requires a series of steps to :
Articulate who has to do what, with what resources, by what time, and toward what goal.
I dentify who must know about the decision.
2. I dentify needed resources Resources include people, information (data), and things. Ask yourself:
Are there any special requirements?
What resources do I need?
How long will it take?
Where will I get them?
What can others offer?
3. Determine objectives,Objectives are measurable targets that are:
Used to monitor progress and establish priorities.
Based on analysis of the situation and contingencies.
4. Build a plan Your plan should state :
Who ...
Will do what (and with whom) ...
How
By when
Where
Remember: Communicate the plan to all parties involved!
5. I mplement the plan Use the action plan to put the decision in place.
The correct answers are
Presents the best of the feasible solutions.
Describes the activities required to implement the solution.
Describes the resources needed to implement the solution.
I ncludes a timetable for implementing the solution.
Addresses foreseeable side effects.
I dentifies who will take the action.
Remember that the plan must be disseminated to all who have responsibility for any part of implementing it.
Step 5 Evaluating the Solution Situation
Evaluation involves two parts :
Monitoring progress. Ask :
1. Has the situation changed?
2. Are more (or fewer) resources required?
3. I s a different alternative solution required?
Monitoring the success and results of a decision is an ongoing process that is critical to fine tuning a course of
action.
Evaluating the results. Use the following checklist to help you evaluate the decision.
WWW.UPSCPORTAL.COM
For Any Help Call Our Course Director:
at +91 8800734161, 011- 65023618
Click Here to Order
http://www.upscportal.com/civilservices/study-kit/ias-pre-csat-paper-2
Checklists For Evaluating the Results
1. Does the decision and action plan make use of existing channels of communication to generate feedback? 3. Will
the feedback test the effectiveness of the decision?
2. Is the solution achieving its purpose?
3. Will the feedback be sufficient to reflect changing circumstances and conditions that might occasion the need to
modify the plan?
4. How will you know if the proposed decision has worked? Is it measurable? If yes, how?
5. Is timely information generated so that it can be supplied to operational, administrative, and policy units in the
jurisdiction?
For effective decision making, one needs to identify the various styles and attributes of Decision Making. One
must find ones psychological type, and the ways to relate it to ones personal preferences as well as factors and
styles that have an impact on ones ability to make rational and effective decisions. Also required is that you first
accept whether the situation requires the individual decision or group decision.
Carl J ungs Psychological Type
TYPE AND COMPONENTS OF DECISION MAKING
The psychiatrist Carl J ung categorized different personality patterns as psychological types. Based on his
observation of peoples behavior, J ung concluded that people have inherent differences in how they use their
minds and where they derive and focus their energy.
J ung identified two mental processes:
1. Taking in information (or perceiving).
2. Organizing information and drawing conclusions (or judging).
J ung also identified two different ways in which people do each of these mental activities
1. People take in information either through their senses or by intuition.
2. People organize information either by thinking or by feeling.
People also differ in where they derive and focus their energy. They are either externally oriented (extroverts),
energized by people and activity; or internally oriented (introverts), energized by ideas and thoughts. These
different ways of organizing and relating to the world obviously can be combined in different ways, thus creating
different psychological types. For example, one person could be an extrovert who relies on thinking more than
feeling and sensing over intuiting, while another individual could be an introvert who is intuitive and feeling-
oriented.
Using Type Inventories to Determine Preferences
One kind of personality test called a type inventory can help you to determine your preferred or dominant
ways of functioning, including your preferred decision-making style. Such tests ask you to answer a series of
questions, and, based on your responses, assign you a type that is an aggregate of your preferences.
1. Extroversion vs. introversion (where energy is derived and focuse(d)
2. Sensing vs. intuition (how information is obtaine(d)
3. Thinking vs. feeling (how decisions are mad(e)
4. J udging vs. perceiving (how the test taker is oriented toward the external worl(d)
Your TypeWhat Does It Mean for Decision Making?
The following are some generalizations about each dominant function:
1. Feeling: The bias is towards integrity. Decisions should consider peoples values and needs. Ask, How does
this affect those involved?
WWW.UPSCPORTAL.COM
For Any Help Call Our Course Director:
at +91 8800734161, 011- 65023618
Click Here to Order
http://www.upscportal.com/civilservices/study-kit/ias-pre-csat-paper-2
2. Sensing: The bias is toward stability. Decisions have to make sense based on past experience. Ask, What
are the facts, costs, and benefits?
3. Thinking: The bias is toward effectiveness. Decisions must be objective and logical. Ask, What are the pros
and cons, causes and effects?
4. Intuition: The bias is toward innovation. Decisions should creatively make use of new opportunities and
insights. Asks, What are the patterns and possible interpretations?
What Does It Mean to Flex?
To flex means to ask yourself the questions of the other three functions, as well as the questions of your
dominant function that naturally occur to you.A key to good decision making is that it uses both sensing and
intuition to gather all the pertinent information, and both thinking and feeling to weigh all the factors involved.
When we rely only on our dominant function, we tend to miss things and make poorer decisions. Although this
process may feel awkward at first, it will lead to decisions that are more sound.
Who Decides?
As you read through the four types, note that the amount of control that the leader has over the decision
drops from total to almost none.I n addition to the four dominant functions explained above, there are also four
styles of decision making based on who makes the decision.Yet, the leader retains ultimate responsibility.
As an emergency manager There are other times, however, when you must make a command decision alone.
Several factors affect whether a decision should be made by an individual or a group. Use the questions in J ob Aid
3-1, on the following page, to determine whether the circumstances call for an individual or group decision.
Individual Decision Making
You work often in situations that require a high degree of coordination in individual decision making. These
cases call for a group decision-making process. The leader must make the decision alone, and input from others is
limited to collecting relevant information.
Group Decision Making
I n this case, the leaden and others work together until they reach a consensus decision. Each group members
opinion and point of view is considered. As a result of helping to make the decision, group members buy into the
final decision and commit to supporting its implementation.
Successful Group Decision Making
Group decision making requires good leadership to be successful. There are special conditions necessary for
group decision making, such as adequate time. There are also particular pitfalls unique to group decision making,
such as groupthink.
Decision Making Through Consultation
I n consultation, the leader shares the issue with one or more people-seeking ideas, opinions, and suggestions
and then makes a decision. The leader considers the input of others, but the final decision may or may not be
influenced by it.
Delegating the Decision
When delegating a decision, the leader sets the parameters, then allows one on more others to make the final
decision. Although the leaden does not make the decision, he on she supports it.

Sample Chapters
of
General Mental
Ability

WWW.UPSCPORTAL.COM
For Any Help Call Our Course Director:
at +91 8800734161, 011- 65023618
Click Here to Order
http://www.upscportal.com/civilservices/study-kit/ias-pre-csat-paper-2
What is Blood Relations ?
Blood relation logical problems mainly deal with the hierarchical structure of a family i.e., grand-parents,
parents and children etc. Different relationships between the family members of different generations will be
given. To solve the questions related to blood relations, the entire family tree has to be drawn by putting the
various relationships.
I n such problems, the aptitude of candidate is shown by the knowledge of the various blood relations. The
typical relationships that are commonly used in blood relation problems are summarized as follows
v Fathers or Mothers son =Brother
v Fathers or Mothers daughter =Sister
v Fathers or Mothers brother =Uncle
v Fathers or Mothers sister =Aunt
v Fathers or Mothers father =Grandfather
v Fathers or Mothers mother =Grandmother
v Brothers or Sisters Son =Nephew
v Brothers or Sisters Daughter =Niece
v Uncles or Aunts son or daughter =Cousin
v Sisters Husband =Brother-in-law
v Brothers wife =Sister-in-law
v Sons wife =Daughter-in-law
v Daughters Husband =Son-in-law
v Husbands or wifes sister =Sister-in-law
v Husbands or wifes Brother =Brother-in-law
To develop a blood relation tree, some standard symbols are used in this chapter to indicate the relationships
among the family members. I t is not compulsory to follow them, You can design your own symbols to draw the
family tree quickly and accurately.
v P is male =P
v P is female =P
v Gender of P is not known =P
v P and Q are married to each other P Q
v P and Q are sibling =P Q
Blood Relations
CHAPTER 5
WWW.UPSCPORTAL.COM
For Any Help Call Our Course Director:
at +91 8800734161, 011- 65023618
Click Here to Order
http://www.upscportal.com/civilservices/study-kit/ias-pre-csat-paper-2
v P and Q are Rs children =R
v P is the Uncle/Aunt of Q =
R
P Q
v P is the only child of q =
Q
P
To draw a family tree, first of all identify the males and the females and then try to put each member at the
appropriate position in the tree.
Example 1: Pointing to Lalit in the photograph, Rajan said, His mother has only one grandchild whose
mother is my sister.How is Rajan related to Lalit ?
(a) Brother (b) Brother-in-law (c) Father-in-law (d) Data inadequate
Sol uti on: (b)
Sister
Rajan
Brother-in-law
wife
Grandchild
Mother Lalit
Mother
Example 2: Deepak said to Nitin, The boy playing with the football is the younger of the two brothers of the
daughter of my fathers wife. How is the boy playing football related to Deepak?
(a) Cousin (b) Brother (c) Son (d) Brother-in-law
Sol uti on: (b)
Father = Wife
Deepak
B
r
o
t
h
e
r
Daughter
Nitin
Example 3: B is the brother of A, S is the sister of B, E is the brother of D, D is the daughter of A, F is the
father of S. Then, the uncle of E is
(a) A (b) B (c) F (d) D
Sol uti on: (b)
Father
Sister Brother
Uncle
Daughter
S B
E D
A
Brother
E and D are children of A and B is the brother of A.
So, B is the uncle of E.
WWW.UPSCPORTAL.COM
For Any Help Call Our Course Director:
at +91 8800734161, 011- 65023618
Click Here to Order
http://www.upscportal.com/civilservices/study-kit/ias-pre-csat-paper-2
Example 4: Pointing to a man. On the stage Rita said He is the brother of daughter of the wife of my
husband. How is the man of stage related to Rita?
(a) Son (b) Husband
(c) Cousin (d) Nephew
Sol uti on: (a)
Husband
Wife
Rita
Brother Daughter
So, that person is the son of Rita.
Example 5: I f P * Q indicates P is the father of Q; P Q indicates `P is the sister of Q; P o Q indicates
P is the brother of Q; P Q indicates P is the mother of Q; which of the following would represent P is brother-
in-law of Q ?
(a) PoRQ (b) P *RQ
(c) PRQ (d) None of these
Sol uti on. (d) I n order to represent the relationship of, brother-in-law, we need the symbol for either
husband or wife, which is not given in the question. Hence, the required relationship cannot be represented by
any of the combinations given. So, the answer is (d).
Example 6: I ntroducing a boy, a girl said, He is the only son of my mothers mother. How is the girl related
to that boy ?
(a) Aunt (b) Niece
(c) Sister (d) Mother
Sol uti on: (b)
Mother
Girl
O
n
l
y

s
o
n
Niece
Boy
Mother
Di recti ons (Q. Nos. 7 to 8) Read the fol l owi ng i nforamati on and answer the questi ons gi ven bel ow. P +
Q mean P i s the daughter of Q, P Q means P i s the son of Q and P Q means P i s the wi fe of Q.
Example 7: I f A B D, which of the following is true ?
(a) D is wife of B (b) A is daughter of B
(c) B is father of A (d) D is father of A
Sol uti on: (d)
B D
A
S
o
n
Example 8: I f C D P O, which of the following is not true?
(a) P is mother of D (b) D is daughter of P
(c) C is wife of D (d) 0 is husband of P
WWW.UPSCPORTAL.COM
For Any Help Call Our Course Director:
at +91 8800734161, 011- 65023618
Click Here to Order
http://www.upscportal.com/civilservices/study-kit/ias-pre-csat-paper-2
Sol uti on: (b)
D
S
o
n
C
P O
Example 9: R is the brother of G. Q is the sister of R. 0 is the brother of N. N is the daughter of G. L is the
father of Q, who is the uncle of O ?
(a) R (b) L
(c) G (d) Q
Sol uti on. (a) O is the brother of N who is the daughter of G. So, 0 is the son of G. R is the brother of G. So,
R is the uncle of O. Hence, the answer is (a) R.
L
R G Q(Sister or R) (Brother)
N (Sister) (Brother) O
Example 10: I f R * S means R is the wife of S, RT S means R is the father of S, R #S means R is the daughter
of S and R -S means R is the brother of S, if P 0 #H #D Q, then how is D related to P?
(a) Father (b) Brother
(c) Mother-in-law (d) Father-in-law
Sol uti on. (d) P 0 #H #D Q means, P is the father of 0, 0 is the daughter of H, that means H is the
mother of 0. (So P is the husband of H) and H is the daughter of D and D is the brother of Q, this means D is the
father of H. So, D is the father-in-law of P.
Example 11: I f Y says that his mother is the only daughter of Xs mother, How is X related to Y?
(a) Uncle (b) Brother
(c) Cousin (d) Maternal uncle
Sol uti on. (d) Assume Ys mother be S, and Xs mother be T, As S is the only daughter of T. So, X is the son
of T. The family tree can be draw as following.
T
S X
Y
Brother
Mother
U
n
c
le
So, X is maternal uncle of Y.
Example 12: Pointing to Sagar in a photograph, Manjula said, His brothers father is the only son of my
grandfather. How is Manjula related to Sagar ?
(a) Aunt (b) Sister
(c) Mother (d) None of these
Sol uti on. (b) Only son of Manjulas grandfather. Manjulas father and Sagars brothers father-Sagars
father. So, Manjula is Sagars sister.
WWW.UPSCPORTAL.COM
For Any Help Call Our Course Director:
at +91 8800734161, 011- 65023618
Click Here to Order
http://www.upscportal.com/civilservices/study-kit/ias-pre-csat-paper-2
Grandfather
Father
Majula Saga Brother of Sagar
Hence, the answer is (b) Sister.
Example 13: Sia introduced Raghav as the son of the only daughter of the father of her uncle. How is
Raghav related to Sia ?
(a) Brother (b) Cousin
(c) Nephew (d) Cant be determined
Sol uti on. (d) Only daughter of uncles father-uncles sister. But it is not clear that uncles sister is Sias
mother.
Father
(Son) Uccle Only Daughter
Sia Son (Raghav)
Hence, the answer is (d) cannot be determined.
Example 14: I f P @Q means P is the brother of Q, P Q means P is the mother of Q and P Q means P is
the sister of Q, then which of the following would mean A is the uncle of B.
(a) AP@B (b) AB@P
(c) A@PB (d) APB
Sol uti on. (c) A is the uncle of B means A is the brother of the mother of B. According to the given expressions.
I t must be A @PAB, so the answer is (c).
Example 15: I ntroducing a woman, Nisha said, She is the daughter-in-law of the grandmother of my
fathers only son. How is the woman related to Nisha ?
(a) Grandmother (b) Sister-in-law
(c) Sister (d) Mother
Sol uti on. (d) My fathers only son-my brother, grandmother of my brother-my grandmother and daughter-
in-law of my grandmother-my mother.
Grand mother
Father
Nisha Son
Woman
(Daughter-in-law)
Hence, Nisha is the daughter of the woman. So, the answer is (d) Mother.
WWW.UPSCPORTAL.COM
For Any Help Call Our Course Director:
at +91 8800734161, 011- 65023618
Click Here to Order
http://www.upscportal.com/civilservices/study-kit/ias-pre-csat-paper-2
1. Pointing to a woman, Naman said, "She is the
daughter of the only child of my grandmother."
How is the woman related to Naman?
(a) Sister (b) Niece
(c) Cousin (d) Data inadequate
2. Pointing to a photograph, a person tells his friend,
"She is the grand daughter of the elder brother of
my father." How is the girl in the photograph
related to this man?
(a) Niece (b) Sister
(c) Aunt (d) Sister-in-law
3. A man said to a lady, "Your mother's husband's
sister is my aunt." How is the lady related to the
man?
(a) Daughter (b) Grand daughter
(c) Mother (d) Sister
4. I f Neena says, "Anita's father Raman is the only
son of my father-in-law Mahipal", then how is
Bi ndu, who is the sister of Anita, rel ated to
Mahipal?
(a) Niece (b) Daughter
(c) Wife (d) None of these
5. Pointing to the woman in the picture, Rajiv said,
"Her mother has onl y one grandchi ld whose
mother is my wife." How is the woman in the
picture related to Rajiv?
(a) Cousin (b) Wife
(c) Sister (d) Data inadequate
ANSWERS
1. (a) 2. (a) 3. (d) 4. (d) 5. (b)
EXPLANATIONS
1. Only child of Naman's grandmother Naman's
father/mother.
Daughter of Naman's father/mother Naman's
sister.
2. Brother of father Uncl e; Uncl e's grand
daughter Daughter of Uncle's son Daughter
of cousin Niece.
3. Lady's mother's husband Lady's father; Lady's
father's sister Lady's aunt.
So, Lady's aunt is man's aunt and thererfore lady
is man's sister.
4. Only son of Neena's father-in-law Mahipal
Neena's husband.
So, Raman is Neena's husband and Anita and
Bindu are his daughters.
Thus, Bindu is the grand daughter of Mahipal.
5. Rajiv's wife's child Rajiv's child.
The woman's mother is the grandmother of Rajiv's
child. So, the woman is Rajiv's wife.
BLOOD RELATIONS
Here, we deal with the questions having information regarding arrangement. We are giving some objects
and then we have to arrange them according to information provided. Objects are arranged either in row or in a
circle on the basis of such conditions. I n some questions, we will also deal with other arrangements like square/
pentagon/ hexagonal.
Different Types of Sitting Arrangement
There are two types of questions which are asked in various competitions
Type 1 Arrangement around a Closed Path
I n such questions, we are giving some clues regarding arrangement. We have apply these clues on a circle
and using these information, we have to find the solutions of questions associated with them. I n this arrangement,
we also deal square/pentagon/ hexagon, questions.
Before solving type 1 questions, following facts are necessary to know.
For Circular Arrangement
I n this arrangement some persons are sitting around a circle and they facing the centre.
Left Right
1. Left movement also called clockwise rotation.
2. Right movement also called anticlockwise rotation.
For Rectangular Arrangement
Right
Left A B
C
D
A B (Fornt with each other) C D (Front with wach other)
Sitting Arrangements
CHAPTER 9
WWW.UPSCPORTAL.COM
For Any Help Call Our Course Director:
at +91 8800734161, 011- 65023618
Click Here to Order
http://www.upscportal.com/civilservices/study-kit/ias-pre-csat-paper-2
For Hexagonal Arrangement
A
B
C
D
E
F
Right Left
A D (Front with each other) B E (Front with each other)
C F (Front with each other)
Di recti ons (Exampl es 1 to 5) Read the fol l owi ng i nformati on careful l y and answer the questi ons based
on i t:
P, Q, R, S, T, U, V and W are eight friends. They are sitting around a round table and taking coffee.
(i) P is sitting between U and V (ii) Q is sitting between W and T
(iii)R is sitting to the third left of V (iv)W is sitting to the third right of S
Example 1: What is the position of V with respect to S?
(a) immediate left (b) immediate right
(c) third to the left (d) fourth to the right
Example 2: How many persons are sitting between R and S?
(a) 2 (b) 3
(c) 4 (d) 5
Example 3: What is the position of T with respect to W?
(a) immediate left (b) immediate right
(c) third to the left (d) second to the left
Example 4: Which of the following is true?
(a) U is sitting immediate right of R (b) U is sitting immediate left of P
(c) There is only one friend between Wand T (d) All of the above
Example 5: Which of the following is false?
(a) There are two friends between P and W (b) There is no friend between Wand Q
(c) P is immediate right of V (d) S is to the second left of Q
Sol uti ons. Given figure shows the correct sitting arrangement of all eight friends sitting around the circle.
Q
T
S
V
P
U
R
W
1. (a) I t is clear from the diagram, V is immediately left of S.
2. (b) Clearly three members are sitting between R and S.
3. (d) Clearly, T is sitting second to the left of W.
4. (a) U is sitting immediately right of R (True)
(b) U is sitting immediately left of P (True) (c) There is only Q between W and T (True)
WWW.UPSCPORTAL.COM
For Any Help Call Our Course Director:
at +91 8800734161, 011- 65023618
Click Here to Order
http://www.upscportal.com/civilservices/study-kit/ias-pre-csat-paper-2
5. (a) There are two friends between P and W (True)
(b) There is no friend between W and Q (True)
(c) P is immediately right of V (False)
(d) S is to the second left of Q (True)
Di recti ons (Exampl es 6 to 7) Read the fol l owi ng i nformati on careful l y and answer the questi ons based
on i t.
Six persons M, N, 0, P, Q and R are playing cards sitting in a circle facing the centre. R is sitting between M
and N and Q is sitting between 0 and P. P is sitting immediate right of M.
Example 6: Who is sitting immediate left of 0?
(a) R (b) M
(c) Q (d) P
Example 7: Who are the neighbours of R?
(a) M, N (b) P, Q
(c) P, O (d) Q, O
Sol uti on. Given figure shows the correct arrangement of these six persons.
N
O
Q
P
M
R
6. (c) Clearly, Q is sitting immediately left of 0.
7. (a) Clearly, M and N are the neighbours of R.
Type 2 Arrangement in a Line or Others
Sometime, we deal with questions where arrangement is required to be done in a line or
others.
Before solving type 2 questions, following facts are necessary to know.
When A, B, C, D facing South direction and P, Q, R, S facing North direction in a line, then their position of
Right and Left will be
A B C D
P Q R S
Right
Left Right
Left
North
South
West East
When A, B in one line and P, Q in other line. then their diagonally opposite direction will be
WWW.UPSCPORTAL.COM
For Any Help Call Our Course Director:
at +91 8800734161, 011- 65023618
Click Here to Order
http://www.upscportal.com/civilservices/study-kit/ias-pre-csat-paper-2
A B
P Q
A is diagonally opposite to Q. B is diagonally opposite to P.
Di recti ons (Exampl e 8) Read the fol l owi ng i nformati on careful l y and answer the questi on based on i t.
I n a shop, the items were arranged in a shelf consisting of six rows. Biscuits are arranged above the tins of
chocolates but below the rows of packets of chips, cakes are at the bottom and the bottles of pepermints are below
the chocolates. The topmost row had the display of jam bottles.
Example 8: The bottles of peppermints are below the chocolates. The topmost row hat the display of jam
bottles. Where exactly are the bottles of pepermints? Mention the place from the top?
(a) 2
nd
(b) 3
rd
(c) 4
th
(d) 5
th
Sol uti on. (d) J ams bottles are at the top. Now,
Chocolates
Biscuits
Chips Chocolates
Pepermints
Cakes
Finally
Jam Bottles
Chips
Biscuits
Chocolates
Pepermints
Cakes
5 from top
th
Di recti ons (Exampl es 9 to 12) Study the fol l owi ng i nformati on careful l y and answer the questi ons
gi ven bel ow
Six friends A, B, C, D, E and F are sitting in a row facing towards North. C is sitting between A and E. E is not
at the end. B is sitting immediate right of E. F is not at the right end. D is sitting second to the left.
Example 9: How many persons are there to the right of D?
(a) One (b) Two
(c) Three (d) Four
Example 10: Which of the following pairs is sitting to the left of D?
(a) F (b) C
(c) E (d) A
Example 11: Who is immediate left of C?
(a) A (b) E
(c) Either E or A (d) Cannot be determined
WWW.UPSCPORTAL.COM
For Any Help Call Our Course Director:
at +91 8800734161, 011- 65023618
Click Here to Order
http://www.upscportal.com/civilservices/study-kit/ias-pre-csat-paper-2
Di recti ons (Q. Nos. 1 to 2) Read the i nformati on
gi ven bel ow and then answer the questi ons that
fol l ow
Eight friends A, B, C, D, E, F, G and H are sitting
in a circle facing the centre. B is sitting between G and
D. H is third to the left of B and second to the right of A.
C is sitting between A and G and B and E are not sitting
opposite to each other.
1. Who is third to the left of D?
(a) F (b) E
(c) A (d) None of these
2. Which of the following statements is not correct?
(a) D and A are sitting opposite to each other
(b) C is third to the right of D
(c) E is sitting between F and D
(d) A is sitting between C and F
Di recti ons (Q. Nos. 3 to 5) Study the fol l owi ng
i nformati on to answer the gi ven questi ons
(i) Eight friends A, B, C, D, E, F, G and H are seated
in a circle facing the centre.
(ii) D is between B and G and F is between A and H.
(iii) E is second to the right of A.
3. Which of the following is As position?
(a) Left of F (b) Right of F
(c) Between E and F (d) None of these
4. Whi ch of the fol l owi ng i nformati on are not
required to ascertain the position of C?
(a) (i) (b) Either (ii) or (iii)
(c) (iii) (d) All are required
5. Which of the following is Cs position?
(a) Between E and F (b) Between G and E
(c) Second to the left of B
(d) None of these
ANSWERS
1. (a) 2. (c) 3. (b) 4. (d) 5. (d)
EXPLANATIONS
Explanation (Q. Nos. 1 to 2) Given figure shows
the correct position of persons sitting around the
circular table.
1. F is third to the left of D.
2. E is sitting between H and D. Hence, alternative
is not correct.
Explanation (Q. Nos. 3 to 5) Given figure shows
the correct position of persons sitting around the
circle.
B or G
E
C
A
F
H
B or G
D
3. A is sitting to the right of F.
4. All information are necessary.
5. I t is clear from the diagram, C is sitting between
A and E.
Explanation (Q. Nos. 6 to 10)
Gi ven figure shows the correct
position of persons sitting around
the circle.
Example 12: Who is at the right end?
(a) A (b) B
(c) E (d) Cannot be determined
Sol uti on. The given figure, shows the correct arrangement of six friends.
F D A C E B
Sitting Arrangement
9. (d) There are four persons to the right D-A, C, E, B.
10. (a) Clearly, F is sitting to the left of D.
11. (a) I t is clear from the diagram that A is to the immediate left of C.
12. (b) I t is clear from the diagram that B is at the right end.
EXERCISE
B
E
F
C
G
D
WWW.UPSCPORTAL.COM
For Any Help Call Our Course Director:
at +91 8800734161, 011- 65023618
Click Here to Order
http://www.upscportal.com/civilservices/study-kit/ias-pre-csat-paper-2
WWW.UPSCPORTAL.COM
For Any Help Call Our Course Director:
at +91 8800734161, 011- 65023618
Click Here to Order
http://www.upscportal.com/civilservices/study-kit/ias-pre-csat-paper-2
First we should know some mathematical operations. They are add (+), subtraction (), multiply () and
division (), greater than (>), less than (<). This test is set up to test candidates skill in mathematical operations.
The questions involving these operations are set using artificial symbols. You are required to substitute the real
signs and solve the questions accordingly, to get the answer.
Different Type of Questions
There are three types of questions based on mathematical operations which are asked in various competitive
examinations. They are,
Problem-Solving by Substitution
I n such type of questions you have some substitutes for various mathematical symbols or numerals followed
by a question involving calculation of an expression or choosing the correct/incorrect equation.
Rule BODMAS
Brackets
Of
Division
Multiplication
Addition
Subtraction
While solving a mathematical operations proceed according to the BODMAS formula.
Example 1: I f + means minus means divided by means plus and means multiplied by then
which of the following will be the value of expression 7 3.5 2 4 +5 ?
(a) 4 (b) 5
(c) 11 (d) None of these
Sol uti on. (b) Using the proper notations in the given expression, we have
=7 3.5 2 4 +5 =7 +3.5 +2 4 5
=2 +2 4 5 =2 +8 5 =10 5 =5
Arithmetical Reasoning
CHAPTER 11
WWW.UPSCPORTAL.COM
For Any Help Call Our Course Director:
at +91 8800734161, 011- 65023618
Click Here to Order
http://www.upscportal.com/civilservices/study-kit/ias-pre-csat-paper-2
Example 2: I f means +, +means , means and means , then 6 4 5 +2 1 =?
(a) 10 (b) 11
(c) 12 (d) 15
Sol uti on. (d) Using the proper notations in the given expression, we have
6 4 5 +2 1 =6 +4 5 2 1
=6 +4 2.5 1
=6 +10 1
=16 1 =15
Example 3: I f P denotes multiplied by T denotes subtracted from, M denotes added to and B denotes
divided by, then
12 P 6 M 15 T 16 B 4
(a) 70 (b) 83
(c) 75 (d) 110
(e) None of these
Sol uti on. (b) 12 P 6 M 15 T 16 B 4
=12 6 +15 16 4 =12 6 +15 4
=72 +15 4 =87 4 =83
Interchanging of Signs and Numbers
This type of question certain signs or numbers interchanging with each other. The candidate is required to
change the given signs or change the given numbers with each other and select which of the equation is correct
of the given alternatives.
Example 4: I f signs +and and numbers 4 and 8 interchanges with each other, which one of the following
four equations would be correct?
(a) 4 8 +12 =0 (b) 8 4 12 =8
(c) 4 8 12 =16 (d) 8 4 12 =24
Sol uti on. (a) On interchanging signs +and and numbers 4 and 8 in equation (a)
8 +4 12 =0
12 12 =0
0 =0
Example 5: Which one of the four interchanges in signs and number would make the given equation correct?
6 4 +2 =16
(a) +and , 2 and 4 (b) +and , 4 and 6
(c) +and , 2 and 6 (d) None of the above
Sol uti on. (b) On interchanging signs +and and 4 and 6,
4 +6 2 =4 +12 =16
Example 6: I f 5 4=15, 7 8 =49 and 6 5 =24, then 8 4=?
(a) 24 (b) 26
(c) 28 (d) 30
Sol uti on. (a) As, 5 4 =5 (4 1) =5 3 =15
7 8 =7 (8 1) =7 7 =49
and 6 5 =6 (5 1) =6 4 =24
Similarly, 8 4 =8 (4 1) =8 3=24
WWW.UPSCPORTAL.COM
For Any Help Call Our Course Director:
at +91 8800734161, 011- 65023618
Click Here to Order
http://www.upscportal.com/civilservices/study-kit/ias-pre-csat-paper-2
Example 7: I f 64 52 =17, 48 56 =23 and 74 35 =19 then 84 37 =?
(a) 32 (b) 28
(c) 22 (d) 20
Sol uti on. (c) As,
64 52 (6 +4) +(5 +2) =17
48 56 (4 +8) +(5 +6) =23
and 74 35 (7 +4) +(3 +5) =19
Similarly, 84 37 (8 +4) +(3 +7) =22
Deriving the Appropriate Conclusions
I n this type of questions certain relations between different sets of elements is given (in terms of less than,
greater than, or equal to), using either the real symbols or substituted symbols. The candidate is required to
briefly read the given statements and then choose which of the conclusions is/are definitely true.
Di recti ons (Exampl es 8 to 10) I n the fol l owi ng questi ons, the symbol s , @, , % and * are used wi th the
fol l owi ng means as i l l ustrated bel ow :
P Q means P is not smaller than Q
P % Q means P is neither smaller than nor equal to Q
P Q means P is neither greater than nor equal to Q
P Q means P is not greater than Q
P @Q means P is neither greater than nor smaller than Q
Now in each of the following questions assuming the given statements to be true, find which of the three
conclusions I , I I , I I I and I V given below them is/are definitely true and give your answer accordingly.
Example 8: Statements D T, T @R, R M, M % K
Conclusions I . R @D I I . R % D
I I I . K T I V. M T
(a) Only either I or I I is true (b) Only I I I and I V are true
(c) Only either I or I I and I I I are true (d) Only either I or I I and I I I and I V are true
Example 9: Statements J @F, F N, N % H, H G
Conclusions I . G N I I . N J
I I I . F J I V. J G
(a) Only I and I I are true (b) Only I , I I and I I I are true
(c) Only I , I I I and I V are true (d) All I , I I , I I I and I V are true
Example 10: Statements R K, K % D, D @V, V M
Conclusions I . R D I I . V R
I I I . D @M I V. M % D
(a) None is true (b) Only I I I is true
(c) Only I V is true (d) Only either I I I or I V is true
Sol uti ons (810)
Finally
% >
<

a =
WWW.UPSCPORTAL.COM
For Any Help Call Our Course Director:
at +91 8800734161, 011- 65023618
Click Here to Order
http://www.upscportal.com/civilservices/study-kit/ias-pre-csat-paper-2
1. I f stands for division, + for multiplication,
for subtraction and for addition, then which
one of the following equations is correct?
(a) 4 5 +9 3 4 =15
(b) 4 5 9 +3 4 =11
(c) 4 5 9 3 4 =17
(d) 4 5 +9 3 +4 =18
2. I f stands for division, + for multiplication,
for subtraction and for addition. Which
one of the following equation s is correct?
(a) 6 +20 12 7 1 =38
(b) 6 20 +12 7 +1 =57
(c) 6 +20 12 7 1=62
(d) 6 20 12 +7 1 =70
3. I f + stands for di vi si on, stands for
multiplication stands for subtraction and
stands for addition.
Which one of the following is correct?
(a) 18 6 7 +5 2 =22
(b) 18 6 +7 5 2 =16
(c) 18 6 7 +5 2 =20
(d) 18 +6 7 5 2 =18
4. I f stands for addition, < for subtraction, +
for division, > for multiplication for equal
8. (a) Here, D T D T; T @ R T = R;
R M R M; M % K M >K
So, D T =R M >K
Now, R @D R =D (False); R % D R >D (False)
K T K <T (True) M T M T (True)
Hence, only either I or I I and I I I and I V are true.
9. (a) Here, J @F J =F; F N =F N
N % H N >H; H G =H G
So, J =F N >H G
Now, G N G <N (True); N J N J (True)
F J F <J (False): J G J G (False)
Hence, only I and I I are true.
10. (d)Here, R K R <K; K % D K >D
D @V D = V; V M =V M
So, R <K >D =V M
Now, R D R <D (False); V R S V<R (False)
D @M D ?? M (False); M % D M >D (False)
But either I I I or I V is true.
EXERCISE
to, for greater than and = for less than, then
state which of the following is true?
(a) 3 4 >2 9 +3 <3
(b) 5 3 <7 8 +4 1
(c) 5 >2 +2 =10 <4 8
(d) 3 2 <4 16 >2 +4
5. I f + stands for greater than, stands for
addition, + stands for division, stands for
equal to, > stands for ,multiplication= stands
for less than and, stands for minus then which
of the following alternatives is correct?
(a) 5 >2 <1 3 4 1
(b) 5 <2 1 +3 >4 1
(c) 5 >2 1 3 >4 <1
(d) 5 +2 1 =3 +4 >1
ANSWERS
1. (a) 2. (d) 3. (d) 4. (c) 5. (c)
EXPLANATIONS
1. Using the proper notations in (a), we get the
statement as
4 +5 9 3 4 =4 +5 3 4
WWW.UPSCPORTAL.COM
For Any Help Call Our Course Director:
at +91 8800734161, 011- 65023618
Click Here to Order
http://www.upscportal.com/civilservices/study-kit/ias-pre-csat-paper-2
=4 +15 4 =15
2. Using the proper notations in (d), we get the
statement as
6 20 +12 7 1 =6 20 +84
=90 20 =70
3. Using the proper notation i n (d), we get the
statement as
18 6 7 5 +2 =3 7 5 +2
=21 5 +2
=18
4. Using the proper notations in (c), we get the
statement as
5 2 2 <10 4 +8
5 1 <18 4
5 <14
Which is true.
WWW.UPSCPORTAL.COM
For Any Help Call Our Course Director:
at +91 8800734161, 011- 65023618
Click Here to Order
http://www.upscportal.com/civilservices/study-kit/ias-pre-csat-paper-2
1. I f the positions of the first and the sixth digits of
the number 2796543018 are i nterchanged,
similarly the positions of the second and the
seventh digits are interchanged and so on, which
of the following will be the third to the left of
seventh digit from the left end?
(a) 0 (b) 1
(c) 7 (d) 8
2. What will be the difference between the sum of
the odd digits and the sum of the even digits in
the number 857423?
(a) 0 (b) 1
(c) 2 (d) None of these
3. I f each of the odd digits in the number 54638 is
decreased by 1 and each of the even digits is
increased by 1, then which of the following will
be the sum of the digits of the new number?
(a) 25 (b) 26
(c) 28 (d) 29
4. How many times will you write even numerals if
you write all the numbers from 291 to 300?
(a) 11 (b) 13
(c) 15 (d) 17
5. A number is greater than 3 but less than 8. Also,
it is greater than 6 but less than 10. The number
is?
(a) 5 (b) 6
(c) 7 (d) 8
ANSWERS
1. (b) 2. (b) 3. (d) 4. (b) 5. (c)
EXPLANATIONS
1. The new number formed after rearrangement is
4 3 0 1 8 2 7 9 6 5.
2. Required difference =(5 +7 +3) (8 +4 +2) =15
14 =1.
3. The new number formed is 45729.
Required sum =(4 +5 +7 +2 +9) =27.
4. The numbers are 291, 292, 293, 294, 295, 296,
297, 298, 299 and 300.
5. According to first condition, the number is greater
than 3 but less than 8. Such numbers are 4, 5, 6, 7.
According to the second condition, the number is
greater than 6 but less than 10. Such numbers
are 7, 8, 9.
Cl early, the required number is the number
satisfying both the above conditions, i.e.,7.
NUMBER, RANKING AND TIME SEQUENCE TEST

Sample Chapters
of
Logical Reasoning
&
Analytical Ability

WWW.UPSCPORTAL.COM
For Any Help Call Our Course Director:
at +91 8800734161, 011- 65023618
Click Here to Order
http://www.upscportal.com/civilservices/study-kit/ias-pre-csat-paper-2
The word Syllogism is also referred to Logic. Syllogism is an important section of logical reasoning and
hence, a working knowledge of its rules is required on the part of the candidate. Hence, it can be expressed as the
Science of thought as expressed in language. The questions based on syllogism can be solved by using Venn
diagrams and some rules devised with the help of analytical ability.
With this unique characteristic, this test becomes an instrument of teaching the candidates to follow the rules
and work as per the instructions without an error. Here, only the basic concept and rules, which have a bearing
on reasoning faculty could alone help. There are some terminology which are used in syllogism.
Proposition
I t is also referred to as Premises. I t is a sentence which asserts that either a part of, or the whole of, one sets
of objects-the set identified by the subject term in the sentence expressing that sentence either is included in, or is
excluded from, another set-the set identified by the predicate term in that sentence.
Types of Proposition
Categorical Proposition There is relationship between the subject and the predicate without any condition.
Exampl e : I . All beams are logs.
I I . No rod is stick.
Hypothetical Proposition: There is relationship between subject and predicate which is asserted
conditionally.
Exampl e : I . I f it rains he will not come.
I I . I f he comes, I will accompany him.
Disjunctive Proposition I n a disjunctive proposition the assertion is of alteration.
Exampl e : I . Either he is brave or he is strong.
I I . Either he is happy or he cannot take revenge.
Parts of Proposition
I t consists of four parts.
1. Quantifier: I n quantifier the words, all, no and some are used as they express quantity. All and no are
universal quantifiers because they refer to every object in a certain set. And quantifier some is a particular
quantifier because it refers to at least one existing object in a certain set.
2. Subject: I t is the word about which something is said.
Syllogism
CHAPTER 1
WWW.UPSCPORTAL.COM
For Any Help Call Our Course Director:
at +91 8800734161, 011- 65023618
Click Here to Order
http://www.upscportal.com/civilservices/study-kit/ias-pre-csat-paper-2
3. Predicate: I t is the part of proposition which denotes which is affirmed or denied about the subject.
4. Copula: I t is the part of proposition which denotes the relation between the subject and predicate.
Exampl e : All boys are brilliant

Quantifier Subject Copula Predicate
Hence, the standard form of proposition is
Quantifier +Subject +Copula +Predicate
Four-fold classification of categorical proposition: On the basis of quality and quantity of proposition
we can classify them in four categories. To draw valid inferences it is necessary to have a clear
understanding of the A, E, I , O relationship as given in the table.
Symbol Proposition Quantity Quality
A All A are B Universal Affirmative
E No A is B Universal Negative
I Some A are B Particular Affirmative
O Some A are not B Particular Negative
Rules for Deriving the Conclusions from Two Given Premises
1. Universal affirmative or A-type proposition.
Dogs
Goats
Take an example : All goats are dogs This is A type proposition: We can see it by graphical representation of
the above proposition we observe that goats are distributed in dogs. Hence. we can conclude that in A type
proposition only subject is distributed.
2. Universal negative or E-type proposition.
Boy Girl
Take an example : No girl is boy I n this type of proposition both subject and predicate are denial of each
other. This can also be seen in the diagram representing boy Girl and girl. They have nothing in common.
Hence, both subject and predicate are distributed.
3. Particular affirmative or I-type proposition.
Mobile Telephone
Take an example : Some mobiles are telephones. I n this type of proposition subject and predicate have
something in common. This implies that in I -type neither subject nor Mobiles Telephones predicate is
distributed. We can see it graphically as given in figure.
WWW.UPSCPORTAL.COM
For Any Help Call Our Course Director:
at +91 8800734161, 011- 65023618
Click Here to Order
http://www.upscportal.com/civilservices/study-kit/ias-pre-csat-paper-2
4. Particular negative or O-type proposition.
Boys Students
Take an example : Some boys are not students. I n O-type propositions some of the category represented by
boys subject is not students, which means that a section of boys is denied with the entire category of students.
I t is, therefore, deduced that in O-type proposition only predicate is distributed. On account of different
logical approach required to be applied for drawing each type of inference, a clear understanding of this
difference becomes more important.
Rules for Mediate Inference
First introduced by Aristotle, a syllogism is a deductive argument in which conclusion has to be drawn from
two propositions referred to as premises.
Now consider an example.
Statement: I . Vinay is a boy. I I . All boys are honest.
Conclusion I . Vinay is honest.
First two sentences I and I I are called propositions and the sentence I is called conclusion. This conclusion is
drawn from above given two propositions.
Types of Questi ons Asked i n the Exami nati on
There are mainly two types of questions which may be asked under this
1. When premises are in specified form Here premise is in specified form. Here mainly two propositions are
given. Propositions may be particular to universal; universal to particular; particular to particular; universal
to universal.
2. When premises are in jumbled/mixed form Here at least three or more than three proposition are given.
Here pair of two propositions out of them follow as same as in specified form.
Type 1 Premises in Specified Forms
Case 1: The conclusion does not contain the middle term Middle term is the term common to both the premises
and is denoted by M. Hence, for such case, conclusion does not contain any common term belong to both premises.
Example 1
Statement: I . All men are girls.
I I . Some girls are students.
Conclusions I . All girls are men.
I I . Some girls are not students.
Sol uti on. Since, both the conclusions I and I I contain the middle term girls so neither of them can follow.
Venn di agram Representati on: All possible cases can be drawn by using Venn diagram.
Girls
men
Students
or,
Girls Men Students
WWW.UPSCPORTAL.COM
For Any Help Call Our Course Director:
at +91 8800734161, 011- 65023618
Click Here to Order
http://www.upscportal.com/civilservices/study-kit/ias-pre-csat-paper-2
By using both representation (a) and (b) it is clear all girls cannot be men as well as (a) shows some girls are
students, here no man is included but at the same time (b) shows some girls are students have some men are also
students as all men are girls. Hence, we cannot deduce conclusion I I .
So, neither of them can follow.
Example 2
Statement: I . All mangoes are chairs.
I I . Some chairs are tables.
Conclusions I . All mangoes are tables.
I I . Some tables are mangoes.
I I I . No mango is a table.
Solution. Here, the term chair is common to both the statement and hence, is the middle term. Statement
(I ) is A type proposition and in A-type proposition, only subject is distributed, hence, chair being the predicate in
the statement (I ) is not distributed in the second statement. Thus, none of the conclusions following statement
is a valid inference.
Venn di agram representati on: All possible cases can be drawn as
Chairs
Mangoes
Tables
or,
Chairs
Mangoes
Tables
(i) All mangoes are table-this inference is definitely false neither (a) nor (b) shows this conclusion.
(ii) Some tables are mangoes, this inference is uncertain or doubtful.
(iii) No mango is a table, this inference is also uncertain or doubtful. Though it can be concluded from the above
discussion that no valid inference can be drawn between mango and table.
Case 2: No term can be distributed in the conclusion unless it is distributed in the premises.
I f case 1 is compiled with by a pair of statement, it is confirmed that valid mediate inferences can be drawn
from such pair of statement. But every mediate inference drawn cannot be valid. Therefore, case 2 is applied to
check as to the conclusions drawn from a pair of statement in which middle term is distributed, is valid.
Example 3
Statement: I . Some boys are students.
I I . All students are teenagers.
Conclusions I . All teenagers are students.
I I . Some boys are teenagers.
Sol uti on. Statement I is an I -type proposition which distributes neither the subject nor the predicate.
Statement I I is an A type proposition which distributes the subject students. Conclusion I is an A-type proposition
which distributes the subject teenagers only.
Since. the term teenagers is distributed in conclusion I without being distributed in the premises. So, conclusion
I cannot follow. I n second conclusion, where it is asked that some boys are teenagers. But from statement I it is
clear that some students are not students. These students may not be teenagers.
Venn di agram representati on: All possible cases can be drawn as follows
Boys
Students
T
e
e
n
a
g
e
rs
.
WWW.UPSCPORTAL.COM
For Any Help Call Our Course Director:
at +91 8800734161, 011- 65023618
Click Here to Order
http://www.upscportal.com/civilservices/study-kit/ias-pre-csat-paper-2
We have given that all students are teenagers so, its reverse cannot be possible. Hence, conclusion I is false.
As we are also given that some boys are students and all students are teenagers. So, some boys whi ch are
students must be teenagers. Hence, conclusion I I follows.
Case 3: I f one premises is particular, conclusion is particular. Take an example which explains this case
Example 4
Statement: I . Some boys are thieves.
I I . All thieves are dacoits.
Conclusions I . Some boys are dacoits.
I I . All dacoits are boys.
Sol uti on. Since, one premise is particular, the conclusion must be particular. So, conclusion I I cannot follow.
Venn di agram representati on: All possible cases can be drawn as follows
Boys
Thieves
D
a
c
o
its
Here conclusion I follows but the conclusion I I cannot follow.
Case 4 I f the middle term is distributed twice, the conclusion cannot be universal Take an example which
explains such case.
Example 5
Statement: I . All Lotus are flowers.
I I . No Lily is a Lotus.
Conclusions I . No Lily is flowers.
I I . Some Lilies are flowers.
Sol uti on. Here, the first premise is an A proposition and so, the middle term Lotus forming the subject is
distributed.The second premise is an E proposition and so, the middle term Lotus forming the predicate is
distributed. Since, the middle term is distributed twice, so the conclusion cannot be universal.
Venn-di agram representati on: All possible cases can be drawn as follows
Flowers
Lotus Lily
Lotus
Lotus
Flowers Flowers
Lily
I t is clear from the given Venn-diagrams either conclusion I or I I must be followed.
Case 5 I f both the premises are affirmative, the conclusions must be affirmative. Take an example which
follows such case:
Example 6
Statement: I . All gardens are schools.
I I . All schools are colleges.
Conclusions I . All gardens are colleges.
I I . Some gardens are not colleges.
WWW.UPSCPORTAL.COM
For Any Help Call Our Course Director:
at +91 8800734161, 011- 65023618
Click Here to Order
http://www.upscportal.com/civilservices/study-kit/ias-pre-csat-paper-2
Sol uti on. Since, both the premises are affirmative, the conclusion must be affirmative, so conclusion I I
cannot follow.
Venn di agram representati on: All possible cases can be drawn as follows.
Gardens
Schools
Colleges
Now, taking conclusion I , it is clear that all gardens are also colleges. But taking conclusion I I , we cannot that
derive second conclusion is true. Hence, only the first conclusion must be true.
Case 6 No conclusion follows. There are three types of such cases.
(a) If both the premises are particular
Example 7
Statement: I . Some cups are spoons.
I I . Some spoons are sauccers.
Conclusions I . All cups are sauccers.
I I . Some sauccers are cups.
Sol uti on.Since both the premises are particular, so no definite conclusion follows.
Venn di agram representati on
Cups
Spoons
Sauccers
Cups Sacuccers Spoons
I t is clear from both given Venn-diagrams that no conclusion is followed.
(b) If both the premises are negative
Example 8
Statement: I . No flower is mango.
I I . No mango is cherry.
Conclusions I . No flower is cherry.
I I . Some cherries are mangoes.
Sol uti on. Since, both the premises are negative hence, neither conclusion follows.
Venn di agram representati on: I t is clear from both Venn-diagrams that neither conclusion follows.
(c) I f the major premise is particular and the minor premise is negative
Major premise is the predicate of the conclusion and minor premise is the subject of the conclusion.
Example 9
Statement: I . Some pubs are cows.
I I . No kitten are pubs.
WWW.UPSCPORTAL.COM
For Any Help Call Our Course Director:
at +91 8800734161, 011- 65023618
Click Here to Order
http://www.upscportal.com/civilservices/study-kit/ias-pre-csat-paper-2
Conclusions I . No pubs are kitten.
I I . Some cows are kitten.
Sol uti on. Here, the first premise containing the middle term Kitten as the subject is the major premise and
the second premise containing the middle term Kitten as the predicate is the minor premise. Since, the major
premise is particular and the minor premise is negative. So, no conclusion follows.
Venn di agram representati on: All possible cases are given
Puts
Cows
Kitten
Pubs Cows Kitten Pubs
Cows
Kitten
(a) (b) (c)
I t is clear from the Venn-diagram representation that conclusion I follows.
Complementary pair of conclusions
I n drawing mediate inferences from given statement, students are required to be more attentive in selecting
complementary pair of conclusion where neither of the conclusions is definitely true but a combination of both
makes a complementary pair. As we have already discussed in case number 1 that in the statement where middle
term is not distributed, no valid mediate inference can be drawn but there still exists a possibility that a
complementary pair of conclusions follows from the statement.
Example 10
Statements I . Some cameras are radios.
I I . Some statues are cameras.
Conclusions I . Some ratios are statues.
I I . No radio is statue.
Sol uti on. Either some radios are statues or No radio is statue follows, as I and E-type proposition form a
complementary pair.
Venn diagram representation We can draw all possible cases as given below
Radios Cameras
Statues
Radios Cameras Statues
Hence, using both diagrammatical representation we can conclude either some radios are statues or no radio
is statue. Hence, at least one of the conclusions must be true.
WWW.UPSCPORTAL.COM
For Any Help Call Our Course Director:
at +91 8800734161, 011- 65023618
Click Here to Order
http://www.upscportal.com/civilservices/study-kit/ias-pre-csat-paper-2
Special Cases
Facts
Combi nati ons (Concl usi on) Concl usi on
A +E Either I or I I follows
E +I Either I or I I follows
I +O Either I or I I follows
E +O Either I or I I follows
Example 11
Statements I . All vegetables are green.
I I . Some greens are fruits.
Conclusions I . Some fruits are vegetables.
I I . No fruit is vegetable.
Sol uti on. Here, conclusion I is particular affirmative and conclusion I I is universal negative proposition.
Hence, either conclusion I or conclusion I I follows.
Alternatives
Green
Vegetables
Fruit
or
Vegetables
Fruit
Green
Conclusion: I f we follow Venn-diagram (a) then we can say no fruit is vegetable. Conclusion I I but if we
that follow Venn diagram (b) then we can say some fruits are vegetables (conclusion I ).
Here, either Venn diagram (a) or Venn-diagram (b) is possible. Hence, conclusion I or conclusion I I must be
followed.
Minimal Possibilities
We can represent statements by keeping in mind our conclusions I t we follow that our two conclusions
belong to special case, then either one of them is true.
We can represent minimum possibilities as given directly in the statement, we dont need to think about
parameters.
Example 12
Statements I . Some fruits are vegetables.
I I . Some vegetables are junk food.
I I I . Some junk foods are snacks.
Conclusions I . Some junk foods are vegetables.
I I . Some junk foods are fruits.
Sol uti on. Minimal possibilities
Fruit Vegetables Junk foods Snacks
WWW.UPSCPORTAL.COM
For Any Help Call Our Course Director:
at +91 8800734161, 011- 65023618
Click Here to Order
http://www.upscportal.com/civilservices/study-kit/ias-pre-csat-paper-2
I t is clear from the above diagrams that only conclusion I follows.
Type 2 Mixed/jumbled Problems
I n such type of problem there is a constitution of mixed problems of universal and particular premises.
Di recti ons (Q. Nos. 13 to 16) I n each of the questi ons gi ven bel ow there are three statements fol l owed by
three concl usi ons numbered I , I I and I I I , you have to take the gi ven statements to be true even i f they
seem to be at vari ance from commonl y known facts. Read al l the concl usi ons and then deci de whi ch of
the gi ven concl usi ons l ogi cal l y fol l ows from the gi ven statements di sregardi ng commonl y known, facts.
Example 13
Statements All halls are tyres.
Some tyres are wheels.
All wheels are cars.
Conclusions I . Some cars are wheels.
I I . Some cars are tyres.
I I I . Some wheels are halls.
(a) None follows (b) Only I follows
(c) Only I and I I follow (d) Only I I I follows
Sol uti on. (c)
Conclusions
Tyres
Halls
Cars
Wheels
I . Some cars are wheels. I t can be seen from Venn-diagram.
I I . Also, some cars are tyres (as shown in Venn-diagram).
I I I . I t is not clear from the Venn-diagram. Hence, only I and I I follow.
Example 14
Statements Some pictures are frames.
Some frames are idols.
All idols are curtains.
Conclusions I . Some curtains are pictures.
I I . Some curtains are frames.
I I I . Some idols are frames.
(a) Only I and I I follow (b) Only I I and I I I follow
(c) Only I and I I I follow (d) All follow
Sol uti on. (b)
WWW.UPSCPORTAL.COM
For Any Help Call Our Course Director:
at +91 8800734161, 011- 65023618
Click Here to Order
http://www.upscportal.com/civilservices/study-kit/ias-pre-csat-paper-2
Conclusions
Pictures
frames
idols
Curtains
I . Does not follow.
I I . Follows (clear from the Venn-diagram)
I I I . Follows (clear from the Venn-diagram)
Hence, only I I and I I I follow.
Example 15
Statements Some ice are rings.
No ring is paint.
Some rings are gold.
Conclusions I . No gold is paint.
I I . No ice is gold.
I I I . Some rings are paints.
I V. All golds are rings.
(a) Only I and I I I follow (b) Only I and I I follow
(c) Only I I I and I V follow (d) None follows
Sol uti on. (d) Hence, none of the conclusions follows.
Ice
Rings
Gold
Paint
Example 16
Statements No candle is bell.
Some shoes are bells.
All tables are shoes.
Conclusions I . Some tables are bells.
I I . No table is bell.
I I I . Some shoes are candles.
I V. No shoes are candles.
(a) Only I and I V follow (b) Only I and I I follow
(c) Only I I I and I V follow (d) None of these
Sol uti on. (d)
Candles Bells
Table
Shoes
Sol uti on. (d) Here conclusion I is particular affirmative and conclusion I I is its universal negative. Hence,
either conclusion I or I I follows. Also conclusion I I I is particular affirmative and conclusion I V is its universal
negative. Hence, either I I I or conclusion I V follows. Finally, either conclusion I or I I follows and either conclusion
I I I or conclusion I V follows.
WWW.UPSCPORTAL.COM
For Any Help Call Our Course Director:
at +91 8800734161, 011- 65023618
Click Here to Order
http://www.upscportal.com/civilservices/study-kit/ias-pre-csat-paper-2
Di recti ons (Q. Nos. 1 to 5) I n each questi on gi ven
bel ow ar e two Statements fol l owed by two
concl usi ons numbered I and I I . You have to take
the two gi ven Statements to be true even i f they
seem to be at vari ance from commonl y known facts
and deci de whi ch of the gi ven concl usi ons l ogi cal l y
fol l ows fr om the the gi ven two Statements,
di sregardi ng commonl y known facts.
Gi ve answer as
(a) I f only conclusion I follows
(b) I f only conclusion I I follows
(c) I f neither I nor I I follows
(d) I f both I and I I follow
1. Statement: I . All jungles are tigers.
I I . Some tigers are horses.
Conclusions I . Some tigers are jungles.
I I . All horses are jungles.
2. Statement: I . All birds are tall.
I I . Some tall are hens.
Conclusions I . Some birds are hens.
I I . Some hens are tall.
3. Statement: I . All artists are smokers.
I I . Some smokers are drinkers.
Conclusions I . All smokers are artists.
I I . Some dri nkers are not
smokers.
4. Statement: I . Some hens are cows.
I I . All cows are horses.
Conclusions I . Some horses are hens.
I I . Some hens are horses.
5. Statement: I . All buses are cars.
I I . Some cars are roads.
Conclusions I . Some cars are buses.
I I . Some buses are roads.
ANSWERS
1. (a) 2. (b) 3. (b) 4. (d) 5. (a)
EXPLANATIONS
1.
Tigers
Jungles Horses
Hence, only I follows.
2.
Tall
Birds
Hens
Hence, from both Venn-diagram conclusion I I
follows.
3.
Smokers
Aritist
Drinkers
Hence, conclusion I follows.
4.
Hens Cows
Horses
Hence, both conclusions are true.
5.
Cars
Buses
Roads
Roads
Hence, only I follows.
EXERCISE
I n such types of questions a situation would be presented and some courses of action are suggested in the
context of those situations. These types of questions are designed to test candidates decision making ability. I n
other words a course of action is a step or administrative decision to be taken for improvement, follow up or
further action with regard to the problem on the basis of data provided.
To solve these questions, the candidates are advised to keep in mind the following important points:
1. Correct course of action should either lessen the problem or improve the situation created by the problem.
2. Simple problem must have simple course of action and not a complex one which may create more problems
than to solve or reduce it.
3. Course of action should be feasible and should relate with the practical aspect of life.
Type of Questions that could be Asked in CSAT
Type 1 Two Courses of Action Based
I n these type of questions a Statement is given followed by two courses of action numbered I and I I .
The candidate is required to grasp the Statement, analyse the problem or policy it mentions and then decide as
to which of the courses of action logically follow(s).
Di recti ons (Exampl es 1 to 2) I n each of the questi ons gi ven bel ow i s a Statement fol l owed by two courses
of acti on numbered I and I I . A course of acti on i s a step or admi ni strati ve deci si on to be taken for
i mprovement, fol l ow up or further acti on i n regard to the probl em, pol i cy etc. On the basi s of i nformati ons
gi ven i n the Statement you have to assume everythi ng i n the Statement to be true, then deci de whi ch of
the two gi ven courses of acti on l ogi cal l y fol l ow.
Give your answer as
(a) I f only I follows (b) I f only I I follows
(c) I f neither I nor I I follows (d) I f both I and I I follow
Example 1: Statement: A large number of engineering graduates in our country are not in a position to
have gainful employment at present and the number of such engineers are likely to grow in future.
Courses of Action:
I . The government should launch attractive employment generating schemes and encourage these graduates
to opt for such schemes to effectively use their expertise and knowledge.
I I . This happened due to poliferation of engineering colleges in the country and there by lowering the quality
of the engineering graduates. Those colleges, which are not equipped to impart quality education should be
closed down immediately.
Statement and
Courses of Action
CHAPTER 4
WWW.UPSCPORTAL.COM
For Any Help Call Our Course Director:
at +91 8800734161, 011- 65023618
Click Here to Order
http://www.upscportal.com/civilservices/study-kit/ias-pre-csat-paper-2
Sol uti on. (a) Only course of action I seems to be suitable for pursuing. The first line of course of action I I is
not a course of action and second line does not properly address the problem.
Example 2: Statement: There have been many instances of dacoity and looting in many passenger trains
this years.
Courses of Action:
I . The railway authority should immediately deploy one policemen in each compartment in all the
passenger trains.
I I . The passengers travelling by train should be given training on how to tackle the dacoits and looters.
Sol uti on. (c) Considering the insufficient number of security personnel the course of action I seems to be
untenable. Similarly, course of action I I is not practically feasible. Thus, neither course of action I nor I I follows.
Type 2 Three Courses of Action Based
I n such questions a Statement followed by three courses of action numbered I , I I and I I I . A course of action
is a step or administrative decision to be taken for improvement follow - up or further action in regard to the
problem, policy etc. On the basis of the information given in the Statement, you have to assume everything in
the Statement to be true, then decide which of the suggested courses of action logically follow(s) for pursuing.
Example 3: Statement Many peoples in the locality have fallen sick and admitted in the local hospital after
consuming sweets served during a community meal.
Courses of Action:
I . The police should immediately arrest all the peoples responsible for making the sweets.
I I . The people admitted in the local hospital should immediately be shifted to bigger hospitals.
I I I . The local food and drug authority should investigate to find out the cause of the sickness and take necessary
action.
(a) Only I follows (b) Only I and I I follow
(c) Only I I I follows (d) Only I I and I I I follow
Sol uti ons. (c) Only I I I follows. After finding the cause of the sickness necessary action should be taken.
Hence this authority should investigate to find out the cause of the sickness and take necessary action. As it is not
possible to shift local hospital of bigger hospital.
Example 4: Statement: Many management institutes in the city have enrolled a large number of students
for management courses which are not recognized either by the local university or by the department of technical
education.
Courses of Action:
I . All these management institutes should immediately derecognized by the university and the department of
technical education.
I I . All these management institutes should be asked to refund fees to all such students and enroll them only for
recognized courses.
I I I . All such students should be advised to switch over to the recognized courses in other institutes.
(a) Only I follows (b) Only I I follows
(c) Only I I I follows (d) Only I I and I I I follow
Sol uti on. (a) Such institutes should immediately be derecognized by the university. Hence I course of action
follows. I t is not possible to refund fees to all such students and enroll them only for recognized courses. All
students could not be advised to switch over to the recognized courses in other institutes.
Example 5: Statement: Higher disposal costs encourage those who produce waste to look for cheaper ways
to get rid of it.
Courses of Action:
I . The disposal costs should be made higher.
WWW.UPSCPORTAL.COM
For Any Help Call Our Course Director:
at +91 8800734161, 011- 65023618
Click Here to Order
http://www.upscportal.com/civilservices/study-kit/ias-pre-csat-paper-2
I I . The disposal costs should be brought down.
I I I . A committee should be set up study the details in this respect.
(a) Only I follows (b) Only I I follows
(c) Either I or I I follows (d) Only I I and I I I follow
Solution.
(e) Action I I and I I I are feasible and effective to combat the problem.
EXERCISE
TYPE 1 : TWO POSSIBLE COURSES OF ACTION
Di recti ons (Q. Nos. 1 to 5) I n each of the questi ons
bel ow i s gi ven a Statement fol l owed by two courses
of acti on. Cour se of acti on i s a step f or
admi ni str ati ve deci si on to be taken f or
i mprovement, fol l ow up or further acti on i n regard
to the probl em, pol i cy etc. On the basi s of the
i nfor mati on gi ven i n Statement, you have to
assume everythi ng i n the Statement to be true,
then deci de whi ch of the gi ven suggested courses
of acti on i s/are l ogi cal l y worth pursui ng.
Gi ve your answer as
(a) I f only I follows
(b) I f only lI follows
(c) I f neither I nor I I follows
(d) I f both I and I I follow
1. Statement: A recent study show that children
below five die in the cities of the developing
countries mainly from diarrhoea and parasitic
intestinal worms. Courses of Action
I . Government of the developing countries
should take adequate measure to improve
the hygienic condition in the cities.
I I . Chi l dren bel ow fi ve year i n the of the
developing countries need to kept under
constant medication.
2. Statement: Air export volumes have increase
substantially over the post decade causing be clogs
and difficulties for air cargo agents because of
increased demand for space and services. Courses
of Action
I . Airlines and air cargo agents should jointly
work out a solution a combat the problem.
I I . The reasons for the increase in the volume
of air export should we found out.
3. Statement: There has been significant drop in
the water level of all the lakes supplying water to
the city.
Courses of Action:
I . The water supply authority should ampose
a partial cut in supply to tackle the situation.
I I . The government shoul d appeal to al l
residents through mass media for minimal
use of water.
4. Statement: On an average, about twenty peoples
are run over by trains and dies everyday while
crossi ng the rail way track through the level
crossing.
Courses and Action
I . The railway authorities should be instructed
to close all the level crossings.
I I . Those who are found crossing the tracks
when gates are closed should we fi ned
heavily.
5. Statement: The state Government has decided
to declare Kala Azar as a notifiable disease under
the Epidemics Act, 1978. Family members or
neighbours of the patient are liable to be punished
in case they do not inform the state authorities.
Courses of Action:
I . Efforts shoul d be made to effecti vel y
implement the Act.
I I . The cases of puni shment shoul d we
propagated through mass media so that
more people become aware of the stern
action.
ANSWERS
1. (d) 2. (d) 3. (d) 4. (b) 5. (d)
EXPLANATIONS
1. I f the Government concentrates to improve the
WWW.UPSCPORTAL.COM
For Any Help Call Our Course Director:
at +91 8800734161, 011- 65023618
Click Here to Order
http://www.upscportal.com/civilservices/study-kit/ias-pre-csat-paper-2
hygienic condition in the cities, it may definitely
reduce the impact of problem and this course of
action appears to be feasible also. Hence this
course of action appears to be correct Medication
is another practical feasible step which would help
minimize the cases of death due to diarrhoea and
intestinal worms. Therefore, course of action I I also
follows.
2. The problem faced by airlines and cargo agents is
the non-availability of cargo space. Therefore,
right course of action is to combat the problem.
3. The situation can be tackled by periodic cuts in
supply and urging people to conserve water. So,
both the courses of action follow.
4. The accidents can clearly be prevented by barring
people from crossing the tracks when the gates
are closed. So, only follow (I I ).
5. When the Government takes such an action it is
necessary that people are made aware of the
consequence they would face if they dont obey
the direct loss. Hence I I follows, I is obvious.
WWW.UPSCPORTAL.COM
For Any Help Call Our Course Director:
at +91 8800734161, 011- 65023618
Click Here to Order
http://www.upscportal.com/civilservices/study-kit/ias-pre-csat-paper-2
According to the dictionary, Assertion refers to stating or claiming something forcefully and that of Reason
is fact. This test is meant to judge the candidates knowledge and with it the aim is to determine his ability to
reason out correctly.
Under, we shall deal with the combination of Assertion and Reason and relate with each other in order to find
their correct coordination. Here, four alternative comments on these are given and the correct one is to be chosen.
This is again an important feature of logical reasoning. Following illustrations will fully illustrate the type of such
questions
Di recti ons (Q. Nos. 1 to 3) I n each of the i l l ustrati ons questi ons, there are two statements l abel l ed as
Asserti on (A) and Reason (R). Mark your answer as
(a) if both A and R are true and R is the correct explanation of A
(b) if both A and R are true but R is not the correct explanation of A
(c) if A is true but R is false
(d) if A is false but R is true
Example 1
Assertion (A): Most of ancient civilisations grew near the rivers.
Reason (R): The main occupation of man was agriculture.
Sol uti on. The Assertion is correct because most of the ancient civilisations grew near the rivers as land over
there was fertile and water was easily available. But the Reason is not the correct explanation of the Assertion
Hence, option (b) is true.
Example 2
Assertion (A): Seeds should be treated with fungicide solution.
Reason (R): Seeds do not germinate, unless treated with fungicide solution.
Sol uti on: Seeds are treated with fungicide before sowing to avoid seed borne disease. But, it is not a true
fact that unless seeds are treated with fungicide solution, they cannot germinate. Hence, option (c) is true.
Example 3
Assertion (A): Appendix is a vestigial organ in the human body.
Reason (R): I t does not participate in digestion.
Assertion and Reason
CHAPTER 7
WWW.UPSCPORTAL.COM
For Any Help Call Our Course Director:
at +91 8800734161, 011- 65023618
Click Here to Order
http://www.upscportal.com/civilservices/study-kit/ias-pre-csat-paper-2
Sol uti on: We know that, appendix is a vestigial organ in the human body and does not participate in
digestion. Hence, both Assertion and Reason are correct and Reason fully explains Assertion. Hence, option
as (a) is our answer.
EXERCISE
Di r ecti ons (Q. Nos. 1 to 5) Choose the cor r ect
al ternati ve from the fol l owi ng opti ons as gi ven
bel ow for the Asserti on (A) and Reason (R) gi ven
i n each of the questi ons.
(a) Both A and R are true and R is the correct
explanation of A
(b) Both A and R are true but R is not the correct
explanation of A
(c) A is true but R is false
(d) A is false but R is true
1. Assertion (A): Alcohol rather than mercury is
used in a thermometer to measure a temperature
of 60C.
Reason (R): Alcohol has a lower freezing point
than mercury.
2. Assertion (A): Tides indicate the regular and
periodic rise and fall in sea level.
Reason (R): Tides are caused by the gravitational
pull of the moon and sea level.
3. Assertion (A): Graphite is slippery and used as
a lubricant.
Reason (R): Graphite has free electrons.
4. Assertion (A): When common salt is kept open,
it absorbs moisture from the air.
Reason (R): Common salt contains magnesium
chloride.
5. Assertion (A): The steam engine was invented
by J ames Watt.
Reason (R): There was a problem of taking out
water from flooded mines.
ANSWERS
1. (a) 2. (a) 3. (b) 4. (a) 5. (c)
EXPLANATIONS
1. We know that, mercury freezes at 39

C, while
alcohol freezes at a point far below -100

C. So,
alcohol is used to measure lower temperatures.
Similarly, mercury boils at 357

C, while alcohol
boils at 78

C. So, mercury is used to measure high


temperature.
2. Reason is the main fact and given Assertion fully
describes it.
3. I n graphite, each carbon atom is linked to three
other carbon atoms, while one electron in the
carbon atom is delocalised. So, graphite has free
electrons.
4. We know that, common salt contains magnesium
chloride, hence absorbs moisture from the air.
5. Assertion is correct but Reason is false.
WWW.UPSCPORTAL.COM
For Any Help Call Our Course Director:
at +91 8800734161, 011- 65023618
Click Here to Order
http://www.upscportal.com/civilservices/study-kit/ias-pre-csat-paper-2
This test is mainly to judge a candidates ability to use his presence of mind to tackle a given situation, he may
come across anytime in life. For example, just think if you are going to appear in a test, and on the way you find
an injured person. I n this situation, what will you do?
So, at anytime we can face such problems. The candidate is, thus, expected to choose the best response which
shall present him/her as a good person or a sincere professional.
Types of Questions
Exampl e (Q. Nos. 1 to 5) Such questi ons consi sts four probabl e answers as an al ternati ves. You have to
sel ect most appropri ate al ternati ve as the answer.
Example 1
I n a bus you realize that someone has left his/ her wallet. You would
(a) give the money to the beggar
(b) hand it over to the bus conductor
(c) try to find his/her contact number and inform the owner
(d) leave the wallet as it is
Sol uti on. I f you are an honest man then, you will try to find his/her contact number and inform the owner
of the wallet. I n this way, that wallet will be reached in right hands. So option (c) is our answer.
Example 2
Do you think that one should change his job often and face new situation?
(a) No, unless compelled one should not leave his old job
(b) Yes, every new job is challenging and one should accept the challenge
(c) No, as it takes time to get adjusted
(d) No, as the new situation may not suit you
Sol uti on. Most appropriate answer will be option (b). Because every new job is challenging and one should
accept the challenge. Unless you accept the challenges you can not get different views of life.
Example 3
While travelling in a train, you observe some college students pulling the alarm chain simply to get down at
their desired point. You would
Situation Reaction Tests
CHAPTER 9
WWW.UPSCPORTAL.COM
For Any Help Call Our Course Director:
at +91 8800734161, 011- 65023618
Click Here to Order
http://www.upscportal.com/civilservices/study-kit/ias-pre-csat-paper-2
(a) with the help of some passengers, check them from doing so
(b) let them pull the chain but check them from detraining
(c) inform the guard of the train as soon as it stops
(d) keep quiet and do nothing
Sol uti on. Most appropriate option in this situation will be as in option (a). I n this case, train will not be late.
And others will not have to bear problems. As a good citizen, it is your duty to check such situations.
Example 4
You want to get married to a person of your choice, but your family members give their own reasons, why you
should not marry that person which you do not find very convincing. What would you do?
(a) Go by what your family says
(b) Become throughly confused and still remain undecided
(c) Marry the person of your choice
(d) Try to convince your family about your choice
Sol uti on. (d) Since, the person concerned does not find the views of the family members convincing, he
should try to convince them and mould their views to match his own. J ust following his own choice shall hurt the
familys sentiments and obeying the family members blindly shall be disloyalty towards the person he loves.
I n this way the best way to settle a conflict is always to arrive at a consensus through peaceful talks and
mutual discussions rather than stick to any one side and ignore the other.
Example 5
You are a team leader and two of your colleagues are having a strained relationship with each other. As a
result, they are not contributing well in group activities. How will you handle such a situation?
(a) How can I be bothered with such petty issues? At least the task is being done by others; so it is fine
(b) You will make an explicit effort to help them shake hands
(c) You will give them complementary tasks in which both have to work together
(d) You will punish them for not contributing by keeping them out of the team
Sol uti on. (b) As both your colleagues are having a strained relationship with each other, they are not
contributing well in group activities. But you are a team leader. So, it is your responsibility that work should be
done in good manner and in time. I t can be possible if both your colleagues do works together. I n this case, you
should make an explicit effort to help them shake hands.
EXERCISE
Di r ecti ons (Q. Nos. 1 to 5): I n the fol l owi ng
questi ons consi st four por tabl e answer s as an
al ternati ves. You have to sel ect most appropri ate
al ternati ve as the answer.
1. No risk no gain, you
(a) feel that risk means no gain
(b) believe that this slogan is correct
(c) feel it is foolish to accept unnecessary risk
(d) feel that risk may be taken only after judging
the situation throughly.
2. Your friends like smoking and influence you to do
the same. You will
(a) smoke only because your friends are smoking
(b) refuse to smoke
(c) smoke but only in their presence
(d) refuse and lie to them that you have asthma
3. You have a new boss. You
(a) will be in different
(b) welcome him warmly
(c) go and flatter him as will help out in future
(d) have problems adjusting as you are still loyal
WWW.UPSCPORTAL.COM
For Any Help Call Our Course Director:
at +91 8800734161, 011- 65023618
Click Here to Order
http://www.upscportal.com/civilservices/study-kit/ias-pre-csat-paper-2
to your old boss
4. You are getting late for your college and bus is
not available. I n such a situation
(a) you start walking
(b) you drop the idea of going to college that day
and return home
(c) you think about other possible conveyance
(d) you wait patiently for the bus though you are
late for the class
5. I f you are HR Manager of XYZ company and there
is only one vacancy. There are three candidates
to the post, one out of them is your nephew. You
would
(a) select your nephew though he is not eligible
for the post
(b) select the candidate who is eligible for the post
(c) consult with senior what needs to be done
(d) tell the candidate that vacancy is already
fulfilled
ANSWERS
1. (d) 2. (b) 3. (b) 4. (c) 5. (b)
EXPLANATIONS
1. I n this case you feel that risk may be taken only
after judging the situation thoroughly.
2. Clearly, one should have a strong will power so as
not to yield to easy temptations and indulge in
any activity that shall later prove harmful.
3. I n such si tuati on, you shoul d wel come hi m
warmly as it is not your decision. This decision is
taken by superiors.
4. As you are late for college, in such situation, you
should think about other possible conveyance. As
the correct time of your bus is not known, it may
be further late.
5. You are HR Manager and it is your responsibility
to hi re a qual i ty person, you shoul d gi ve
preference for the one who is eligible for the post.
WWW.UPSCPORTAL.COM
For Any Help Call Our Course Director:
at +91 8800734161, 011- 65023618
Click Here to Order
http://www.upscportal.com/civilservices/study-kit/ias-pre-csat-paper-2
WWW.UPSCPORTAL.COM
For Any Help Call Our Course Director:
at +91 8800734161, 011- 65023618
Click Here to Order
http://www.upscportal.com/civilservices/study-kit/ias-pre-csat-paper-2
When we see clouds in the sky. We take preassumption that it may be rain. Each action has an assumption
one in positive and another in negative. Let us suppose a baby born it may be a boy or a girl. So, here we take two
assumptions.
The candidate will be required to assess the given statement and decide which of the given assumptions is
implicit in the Statement. To understand the pattern of these questions, it is very essential to know what the
terms Statement and Assumptions do stand for.
Hence, An assumption is something that can be supposed or assumed on the basis of a given statement.
Types of Questions Based on Assumption
Type 1 Two-Assumptions Based
I n this type of questions a statement: is given, followed by two assumptions. The candidate is required to
assess the given statement: and then decide which of the given assumptions to implicit in the Statement: .
Di recti ons (Exampl es 1 to 3) I n each of the i l l ustrati ons bel ow i s gi ven a Statement: fol l owed by
Assumptions numbered I and I I . Consi der the Statement: and deci de whi ch of the gi ven Assumptions
i s i mpl i ci t? Gi ve answer as
(a) if only assumption I is implicit
(b) if only assumption I I is implicit
(c) if neither I nor I I is implicit
(d) if both I and I I are implicit
1. Statement: The Union Government has decided to withdraw existing tax relief on various small savings
schemes in a phased manner to augment its tax collection.
Assumptions:
I . People may still continue to keep money in small savings schemes and also pay taxes.
I I . The total tax collection may increase substantially.
Sol uti on. (d) Any measure is taken assuming that it would be accepted by the people. Therefore, both the
assumptions are implicit in the statement .
2. Statement: The Government has decided to levy 2 per cent on the tax amount payable for funding drought
relief programmes.
Statement and Assumptions
CHAPTER 3
WWW.UPSCPORTAL.COM
For Any Help Call Our Course Director:
at +91 8800734161, 011- 65023618
Click Here to Order
http://www.upscportal.com/civilservices/study-kit/ias-pre-csat-paper-2
WWW.UPSCPORTAL.COM
For Any Help Call Our Course Director:
at +91 8800734161, 011- 65023618
Click Here to Order
http://www.upscportal.com/civilservices/study-kit/ias-pre-csat-paper-2
Assumptions:
I . The Government does not have sufficient money to fund drought relief programmes.
I I . The amount collected by way of surcharge may be adequate to fund these drought relief programmes.
Sol uti on. (d) I f there were sufficient money to fund drought relief programmes, why this measure should
be taken. Therefore, both the assumptions are implicit in the statement.
3. Statement: The Government has decided to allow the shopping complexes to remain open till mid night to
reduce crowding of shoppers in these complexes during the weekends.
Assumptions:
I . People may still prefer to shop during the weekends.
I I . Many people may now do their shopping illusively during the late evening hour.
Sol uti on. (b) Only assumption I I is implicit because the people may shopping illusively during the late
evening hours thinking, so the Government has decided to allow the shopping complexes to remain open till
mid night.
Type 2 Three-Assumptions Based
This type also consists of similar type of questions as in Type I , with the difference that here three assumptions
are given and the candidate is required to choose that group which is implicit in context of the given statement.
Di recti ons: I n each questi ons bel ow i s gi ven a statement fol l owed by three assumpti ons numbered I , I I
and I I I . An assumpti on i s somethi ng supposed or token for granted. You hove to consi der the statement
and the fol l owi ng assumpti ons and deci de whi ch of the assumpti ons i s i mpl i ci t i n the statement.
4. Statement: I f you desire to enjoy the best holidays to Europe join our exclusive EURO package, An
advertisement of a travel agency in an I ndian newspaper.
Assumptions:
I . Many I ndians travel of European countries to spend their holidays.
I I . These are other travel agencies in I ndia which organize holiday tour to Europe.
I I I . Many people may still travel to Europe through other travel agencies.
(a) All are implicit (b) Only I and I I are implicit
(c) Only I is implicit (d) Only I and I I I are implicit
Sol uti on. (a) All are implicit because on the basis of all the three assumptions this advertise is given.
5. Statement: Graduate with first class are eligible to apply for the admission to MBA courses in our institute.
An advertisement by a management institute.
Assumptions:
I . Only those who are first class graduates can be up with the studies for MBA courses.
I I . There are plenty of first class graduates who are likely to apply for admission to MBA.
I I I . Reputation of the institute may get affected, if students having less than first class are admitted.
(a) Only I is implicit
(b) Only I I is implicit
(c) Only I I I is implicit
(d) Only I and I I are implicit
Sol uti on. (b) The advertisement was issued assuming that sufficient number of first class graduates are
likely to apply for admission to MBA.
WWW.UPSCPORTAL.COM
For Any Help Call Our Course Director:
at +91 8800734161, 011- 65023618
Click Here to Order
http://www.upscportal.com/civilservices/study-kit/ias-pre-csat-paper-2
Di r ecti ons: I n each questi ons bel ow i s gi ven a
Statement: f ol l owed by two Assumptions
numbered I and I I . An assumpti on i s somethi ng
supposed or taken for gr anted. You have to
consi der the Statements and the fol l owi ng
Assumptions and deci de whi ch of the
Assumptions i s i mpl i ci t i n the Statements?
Gi ve answer
(a) I f only assumption I is implicit
(b) I f only assumption I I is implicit
(c) I f neither I nor I I is implicit
(d) I f both I and I I are implicit
1. Statement: I nvest in our regular scheme and
earn an interest of at least 10%. Advertisement
by a financial institution.
Assumptions:
I . 10% interest may attract a good number of
investors.
I I . No other scheme offers interest as high as
10%.
2. Statement: For easi er and faster
communication, use our broadcast connections.
An advertisement.
Assumptions:
I . Many peopl e are l ooki ng up for best
communication facilities.
I I . Broad band connections are preferred mode
of communication.
3. Statement: You must refer to learn to dictionary,
if you want to become a good writer-A advises B.
Assumptions:
I . Only writers refer to the dictionary.
I I . Al l wri ters, good or bad, refer to the
dictionary.
4. Statement: I f it does not rain throughout this
month, most farmers would be in trouble this year.
Assumptions:
I . Timely rain is essential for farming.
I I . Most of the farmers are generally dependent
on rains.
5. Statement: A advises B- I f you want to study
English, joint I nstitute Y.
Assumptions:
I . B listens to As advice.
I I . I nstitute Y provi des good coachi ng for
English.
ANSWERS
1. (a) 2. (a) 3. (c) 4. (d) 5. (d)
EXEPLANATIONS
1. How can the advertisement be meaningful without
I ? Hence, I is implicit. I I may or may not be an
assumption. Hence, I I is not implicit.
2. People are looking for best communication so this
advertisement is introduced.
3. None of the Assumptions is implicit as nothing can
be said definitely.
4. (d) As the given Statement tells of the essentiality
and requirement or rain for farmers, hence both
the Assumptions are implicit in the Statement .
5. Since, A advises B to join institute Y to study
English, hence it is assumed that institute Y
provides good coaching for English. Secondly, it
is also assumed that B listens to As advice.
EXERCISE

Sample Chapters
of
Basic Numeracy
WWW.UPSCPORTAL.COM
For Any Help Call Our Course Director:
at +91 8800734161, 011- 65023618
Click Here to Order
http://www.upscportal.com/civilservices/study-kit/ias-pre-csat-paper-2
A Civil Servant should be well-versed in basics of Number System. I n the Civil Services Aptitude Test Paper
2, in Basic Numeracy, certainly there will be asked some questions based on types of, and operations on numbers.
I n I ndian system, numbers are expressed by means of symbols 0, 1, 2, 3, 4, 5, 6, 7, 8, 9, called digits. Here, 0
is called insignificant digit whereas 1, 2, 3, 4, 5, 6, 7, 8, 9 are called significant digits. We can express a number in
two ways.
Notation: Representing a number in figures is known as notation as 350.
Numeration: Representing a number in words is known as numeration as Five hundred and forty five.
Place Value (Indian)
Crore Lakh Thousand Unit
Ten Crore Crore Ten Lakhs Lakh Ten Thousands Thousand Hundred Tens One
100000000 10000000 1000000 100000 10000 1000 100 10 1
10
8
10
7
10
6
10
5
10
4
10
3
10
2
10
1
10
0
Place Value (International)
Million Thousand Unit
Hundred Ten Millions One Million Hundred Ten Thousand Hundred Tens One
Millions Thousands Thousands
100000000 10000000 1000000 100000 10000 1000 100 10 1
10
8
10
2
10
6
10
5
10
4
10
3
10
2
10
1
10
0
Face Value and Place Value of a Digit
Face Value: I t is the value of the digit itself eg, in 3452, face value of 4 is four, face value of 2 is two.
Place Value: I t is the face value of the digit multiplied by the place value at which it is situated eg, in 2586,
place value of 5 is 5 10
2
=500.
Number Categories
Natural Numbers (N): I f N is the set of natural numbers, then we write N ={1, 2, 3, 4, 5, 6,}
The smallest natural number is 1.
Whole Numbers (W): I f W is the set of whole numbers, then we write W ={0, 1, 2, 3, 4, 5,}
The smallest whole number is 0.
Number System
CHAPTER 1
WWW.UPSCPORTAL.COM
For Any Help Call Our Course Director:
at +91 8800734161, 011- 65023618
Click Here to Order
http://www.upscportal.com/civilservices/study-kit/ias-pre-csat-paper-2
Integers (I): I f I is the set of integers, then we write I ={ 3, 2, 1, 0, 1, 2, 3, }
Rational Numbers: Any number which can be expressed in the form of p/q, where p and q are both integers
and q #0 are called rational numbers.
eg,
3 7
, ,5, 2
2 9


There exists infinite number of rational numbers between any two rational numbers.
Irrational Numbers Non-recurring and non-terminating decimals are called irrational numbers. These
numbers cannot be expressed in the form of
p
q
.
eg, 3, 5, 29,
Real Numbers: Real number includes both rational and irrational numbers.
Basic Rules on Natural Numbers
1. One digit numbers are from 1 to 9. There are 9 one digit numbers. ie, 9 10
0
.
2. Two digit numbers are from 10 to 99. There, are 90 two digit numbers. ie, 9 10.
3. Three digit numbers are from 100 to 199. There are 900 three digit numbers ie, 9 10
2
.
I n general the number of n digit numbers are 9 10
(n1)
4. Sum of the first n, natural numbers ie, 1 +2 +3 +4 + +n =
( ) 1
2
n n +
5. Sum of the squares of the first n natural numbers ie. 1
2
+2
3
+3
2
+4
2
++n
2
=
( )( ) 1 2 1
6
n n n + +
6. Sum of the cubes of the first n natural ie, 1
2
+2
3
+3
2
++n
3
=
( )
2
1
2
n n

+



Example: What is the value of 51 +52 +53 + +100 ?
Sol uti on. 51 +52 + 33 +... +100 =(1 +3 ++100) (1 +2 +3 +... +50)
=
100 101 50 51
2 2

=5050 1275 =3775


Different Types of Numbers
Even Numbers: Numbers which are exactly divisible by 2 are called even numbers.
eg, 4, 2, 0, 2, 4
Sum of first n even numbers =n (n +1)
Odd Numbers: Numbers which are not exactly divisible by 2 are called odd numbers.
eg, 5, 3, 1, 0, 1, 3, 5
Sum of first n odd numbers =n
2
Prime Numbers: Numbers which are divisible by one and itself only are called prime numbers.
eg, 2, 3, 5, 7, 11
2 is the only even prime number.
WWW.UPSCPORTAL.COM
For Any Help Call Our Course Director:
at +91 8800734161, 011- 65023618
Click Here to Order
http://www.upscportal.com/civilservices/study-kit/ias-pre-csat-paper-2
1 is not a prime number because it has two equal factors.
Every prime number greater than 3 can be written in the form of (6K +1) or (6K 1) where K is an integer.
There are 15 prime numbers between 1 and 50 and l0 prime numbers between 50 and 100.
Relative Prime Numbers: Two numbers are said to be relatively prime if they do not have any common
factor other than 1.
eg, (3, 5), (4, 7), (11, 15), (15, 4)
Twin Primes: Two prime numbers which differ by 2 are called twin primes.
eg, (3, 5), (5, 7), (11, 13),
Composite Numbers Numbers which are not prime arc called composite numbers
eg, 4, 6, 9, 15,
1 is neither prime nor composite.
Perfect Number: A number is said to be a perfect number, if the sum of all its factors excluding itself is
equal to the number itself. eg, Factors of 6 are 1, 2, 3 and 6.
Sum of factors excluding 6 =1 +2 +3 =6.
6 is a perfect number.
Other examples of perfect numbers are 28, 496, 8128 etc.
Rules for Divisibility
Divisibility by 2: A number is divisible by 2 when the digit at ones place is 0, 2, 4, 6 or 8.
eg, 3582, 460, 28, 352, ....
Divisibility by 3: A number is divisible by 3 when sum of all digits of a number is a multiple of 3.
eg, 453 =4 +5 +3 =12.
12 is divisible by 3 so, 453 is also divisible by 3.
Divisibility by 4: A number is divisible by 4, if the number formed with its last two digits is divisible by 4. eg,
if we take the number 45024, the last two digits form 24. Since, the number 24 is divisible by 4, the number 45024
is also divisible by 4.
Divisibility by 5: A number is divisible by 5 if its last digit is 0 or 5.
eg, 10, 25, 60
Divisibility by 6: A number is divisible by 6, if it is divisible both by 2 and 3.
eg, 48, 24, 108
Divisibility by 7: A number is divisible by 7 when the difference between twice the digit at ones place and
the number formed by other digits is either zero or a multiple of 7.
eg, 658
65 2 8 =65 16 =49
As 49 is divisible by 7 the number 658 is also divisible by 7.
Divisibility by 8: A number is divisible by 8, if the number formed by the last 3 digits of the number is
divisible by 8. eg, if we take the number 57832, the last three digits form 832. Since, the number 832 is divisible
by 8, the number 57832 is also divisible by 8..
Divisibility by 9: A number is divisible by 9, if the sum of all the digits of a number is a multiple of 9.
eg, 684 =6 +8 +4 =18.
18 is divisible by 9 so, 684 is also divisible by 9.
Divisibility by 10: A number is divisible by 10, if its last digit is 0. eg, 20, 180, 350,.
Divisibility by 11: When the difference between the sum of its digits in odd places and in even places is
either 0 or a multiple of 11.
WWW.UPSCPORTAL.COM
For Any Help Call Our Course Director:
at +91 8800734161, 011- 65023618
Click Here to Order
http://www.upscportal.com/civilservices/study-kit/ias-pre-csat-paper-2
eg, 30426
3 +4 +6 =13
0 +2 =2
13 2 =11
As the difference is a multiple of 11 the number 30426 is also divisible by 11.
Smart Facts
If p and q are co-primes and both are factors of a number K, then their product p x q will also be a factor of r. eg,
Factors of 24 are 1, 2, 3, 4, 6, 8, 12 and 24 prime factors of 24 are 2 and 3, which are co-prime also. Product of
2 3 = 6, 6 is also a factor of 24.
If p divides q and r, then p also divides their sum or difference. eg, 4 divides 12 and 20. Sum of 12 and 20 is 32
which is divisible by 4. Difference of 20 and 12 is 8 which is divisible by 4.
If a number is divisible by another number, then it must be divisible by each of the factors of that number. 48 is
divisible by 12. Factors of 12 are 1, 2, 3, 4, 6, 12. So, 48 is divisible by 2, 3, 4 and 6 also.
Division on Numbers
I n a sum of division, we have four quantities.
They are (i) Dividend, (ii) Divisor, (iii) Quotient and (iv) Remainder. These quantities are connected by a
relation.
(a) Dividend =Divisor Quotient +Remainder.
(b) Divisor =(Dividend Remainder) Quotient.
(c) Quotient =(Dividend Remainder) Divisor.
Example 2: I n a sum of division, the quotient is 110, the remainder is 250, the divisor is equal to the sum of
the quotient and remainder. What is the dividend ?
Sol uti on. Divisor =(110 +250) =360
Dividend =(360 110) +250 =39850
Hence, the dividend is 39850.
Example 3: Find the number of numbers upto 600 which are divisible by 14.
Sol uti on. Divide 600 by 13, the quotient obtained is 46. Thus, there are 46 numbers less than 600 which are
divisible by 14.
Factors and Multiples
Factor: A number which divides a given number exactly is called a factor of the given number,
eg, 24 =1 24, 2 12, 3 8, 4 6
Thus, 1, 2, 3, 4, 6, 8, 12 and 24 are factors of 24.
1 is a factor of every number
A number is a factor of itself
The smallest factor of a given number is 1 and the greatest factor is the number itself.
I f a number is divided by any of its factors, the remainder is always zero.
Every factor of a number is either less than or at the most equal to the given number.
Number of factors of a number are finite.
Number of Factors of a Number: I f N is a composite number such that N =a
m
b
n
c... where a, b, c ... are
prime factors of N and m, n, o ... are positive integers, then the number of factors of N is given by the expression
(m +1) (n +1) (o +1)
WWW.UPSCPORTAL.COM
For Any Help Call Our Course Director:
at +91 8800734161, 011- 65023618
Click Here to Order
http://www.upscportal.com/civilservices/study-kit/ias-pre-csat-paper-2
1. Evaluate:
( )
93 5 54 10
3 5 242


(a) 9/10 (b) 8/17
(c) 16/19 (d) 4/7
2. The sum of three consecutive natural numbers
each divisible by 3 is 72. What is the largest
among them?
(a) 25 (b) 26
(c) 27 (d) 30
3. 55% of a number is more than one-third of that
number by 52. What is two-fifth of that number?
(a) 96 (b) 240
(c) 144 (d) 142
4. The digits of a two-digit number are in the ratio
of 2 : 3 and the number obtained by interchanging
the digits is bigger than the original number by
27. What is the original number?
(a) 63 (b) 48
(c) 96 (d) 69
5. What least number would be subtracted from
427398 so that the remaining number is divisible
by 15?
(a) 13 (b) 3
(c) 16 (d) 11
ANSWER
1. (c) 2. (c) 3. (a) 4. (d) 5. (b)
EXPLANATIONS
1.
( )


9 3 5 5 4 10
3 5 2 4 2
=
( ) ( )
( )


9 2 5 4 10
3 5 8 2
=


18 2
15 4
=

16
19
2. 3x +(3x +3) +(3x +6) = 72
9x +9 =72 =9x =72 9
or x =
63
9
= 7
The largest of them = 27.
3. Let the number be x.
55
100
x
=
1
52
3
x +

13
60
x
=52 x =240

2
5
x
=
2
240
5
=96
4. Let the number be 10x y
x : y =2 : 3
(10y + x) (10x + y) = 27 ...(i)
9y 9x =27 y x =3 y = x + 3
Putting this value of y, in (i)
3
x
x +
=
2
3
x =6 y = 9
Hence the number is 69.
5. Apply the divisibility tears of 3 and 5.
Example 4: Find the number of factors that 224 has.
Sol uti on. 224 =2
5
7
1
Hence, 224 has (5 +1) (1 +1) =6 2 =12 factors.
Multiple: A multiple of a number is a number obtained by multiplying it by a natural number eg,
Multiples of 5 are 5, 10, 15, 20
Multiples of 12 are 12, 24, 36, 48
Every number is a multiple of 1.
The smallest multiple of a number is the number itself.
We cannot find the greatest multiple of a number.
Number of multiples of a number are infinite.
EXERCISE
I n simplification of an expression there are certain laws which should be strictly adhered to. These laws are
as follows:
VBODMAS Rule
This rule gives the correct sequence in which the mathematical operation are to be executed so as to find out
the value of a given expression.
Here, V stands for Vicnaculum (or Bar), B stands for Brcket, O stands for Of, D stands for Division, M
stands for Multiplication, A stands for Addition and S stand for Subtraction.
(a) Here, VBODMAS gives the order of simplification. Thus, the order of performing the mathematical
operations in a given expression are
First : Virnaculum or line bracket or bar
Second: Bracket
Third: Of
Fourth: Division
Fifth: Multiplication
Sixth: Addition &
Seventh: Subtraction
The above order should strictly be followed.
(b) There are four types of brackets.
(i) Square brackets [ ]
(ii) Curly brackets {}
(iii) Circular brackets ( )
(iv) Bar or Virnaculum
Thus, in simplifying an expression all the brackets must be removed in the order , ( ), {} and [ ].
Modulus of a Real Number
The modulus of a real number x is defined as
=x, if a >0
| x|
=x, if a <0
Simplification
CHAPTER 5
WWW.UPSCPORTAL.COM
For Any Help Call Our Course Director:
at +91 8800734161, 011- 65023618
Click Here to Order
http://www.upscportal.com/civilservices/study-kit/ias-pre-csat-paper-2
Basic Formulae
(i) (a + b)
2
= a
2
+2ab + b
2
(ii) (a b)
2
=a
2
2ab + b
2
(iii) (a + b)
2
(a b)
2
=4ab
(iv) (a + b)
2
+(a b)
2
=2(a
2
+ b
2
)
(v) (a
2
b
2
) = (a + b) (a b)
(vi) (a + b + c)
2
= a
2
+ b
2
+ c
2
+2(ab + bc + ca)
(vii) (a
3
+ b
3
) = (a + b) (a
2
ab + b
2
)
(viii) (a
3
b
3
) = (a b) (a
2
+ ab +b
2
)
(ix) (a
3
+ b
3
+ c
3
3abc) = (a + b + c), (a
2
+ b
2
+ c
2
ab bc ca)
(x) a
3
+ b
3
+ c
3
=3abc, if a + b + c = 0
Example 1: Simplify 1005 +500 10 80.
Sol uti on. 1005 +500 10 80 =1005 +490 80 =1495 80 =1415
Example 2: I f a * b =2(a +b), then what is the value of 5 * 2?
Sol uti on. 5 * 2 =2(5 +2) =2 7 =14
Example 3: I f x =
1 1
3 3
3 3

+ , then what will be the value of 3x


3
9x?
Sol uti on. 3x
3
9x =3[3
1/3
+3
1/3
]
=3[3 +3
1
+3(3
1/3
+3
1/3
)] 9[3
1/3
+3
1/3
]
=
1/3 1/3 1/3 1/3
1
3 3 9(3 3 ) 9(3 3 )
3


+ + + +


=9 +1
=10
Example 4: 2/5 part of the students in a class are the girls and remaining are the boys. I f 2/9 part of the girls
and 1/4 part of the boys are absent, then what part of total students is present.
Sol uti on. Let total number of students =x
Number of girls =
3
5
x
Number of boys =
2
5
x
Number of absent students =
2 3 1 2
of of
9 5 4 5
x x

+


=
2 1
15 10
x x +
=
7
30
x
Present students =
7
1
30
x



=
23
30
x
Example 5: Simplify (2
10
2
9
) (2
8
2
7
).
Sol uti on. (2
10
2
9
) (2
8
2
7
) =2
9
(2 1).2
7
(2 1) =2
9+7
=2
16
Example 6: I f
x
y
=
3
2
find the value of
2 2
2 2
x y
x y
+

.
WWW.UPSCPORTAL.COM
For Any Help Call Our Course Director:
at +91 8800734161, 011- 65023618
Click Here to Order
http://www.upscportal.com/civilservices/study-kit/ias-pre-csat-paper-2
1. What is
1
th
6
of 3?
(a) 6 (b) 3 (c) 1/2 (d) 1/3
2. What is 10 _ _ 8 equal to?
(a) 2 (b) 18 (c) 10 (d) 8
3. Multiply 0.932 by 100.
(a) 932 (b) 9.32 (c) 93.2 (d) 9320
4. Divide 0.045 by 100.
(a) 0.0045 (b) 0.00045
(c) 0.000045 (d) 0.45
5. I f 2x =5 and 3y =8, then
4
9
x
y
is equal to
(a)
5
18
(b)
5
16
(c)
5
12
(d)
5
8
Sol uti on.
2 2
2 2
x y
x y
+

=
2
2
2
2
1
1
x
y
x
y
+

(dividing both numerator and denominator by y


2
)
=
2
2
1
1
x
y
x
y

+




=
2
2
3
1
2
3
1
2

+




=
9
1
4
9
1
4
+

=
13
4
5
4
=
13
5
=
3
2
5
Example 7: Simplify
0.3 0.3 0.03 0.03 0.6 0.03
0.54
+
Sol uti on. 0.3 0.3 +0.03 0.03 0.6 0.03 =0.3 0.3 +0.03 0.03 0.03 is of the form (a
2
+b
2
2ab) =
(a b)
2
a =0.3 and b =0.03 =(0.3 0.03)
2
=(0.27)
2
Given expression
0.27 0.27
0.54

=0.135
EXERCISE
ANSWERS
1. (a) 2. (b) 3. (c) 4. (b) 5. (c)
EXPLANATIONS
1.
1
th
6
of 3 =
1
3
6

=
1
2
2. 10 _ _ 8 =10 +8 = 18
3. 0.932 100 =93.2
4.
0.045
100
=0.00045
5. 2x =5, 3y = 8

4
9
x
y
=
2(2 )
3(3 )
x
y
=
2 5
3 8

=
5
12
WWW.UPSCPORTAL.COM
For Any Help Call Our Course Director:
at +91 8800734161, 011- 65023618
Click Here to Order
http://www.upscportal.com/civilservices/study-kit/ias-pre-csat-paper-2
WWW.UPSCPORTAL.COM
For Any Help Call Our Course Director:
at +91 8800734161, 011- 65023618
Click Here to Order
http://www.upscportal.com/civilservices/study-kit/ias-pre-csat-paper-2
Average
The average of a given number of quantities of the same kind is expressed as
Average =
Sumof thequantities
Number of thequantities
Average is also called the Arithmetic Mean.
Also, Sum of the quantities =Average Number of the quantities
Number of quantities =
Sumof thequantities
Average
I f all the given quantities have the same value, then the number itself is the average.
I f all the given quantities are not all the same, then the average of the given quantities is always
greater, then the smallest number and always less than the largest number. Equivalently, atleast one
of the numbers is less than the average and atleast one is greater then the average.
I f each of the given quantities is increased by a constant p, then their average is also increased by p.
I f each of the given quantities is decreased by a constant p, then their average is also decreased by p.
I f each of the given quantities is multiplied by a constant p, then their average is also multiplied by p.
Whenever the given quantities form an arithmetic sequence and if the given quantities has odd terms,
then the average is the middle term in the sequence and if the given quantities has even terms, then
the average of the sequence is the average of the middle two terms.
I n order to calculate the weighted average of a set of numbers, multiply each number in the set by the
number of times it appears, add all the products and divide by the total number of numbers in the set.
I f the speed of an object from A to B is x km/h and from B to A is y km/h,then the average speed during
the whol e journey i s
2
km/h
xy
x y +
.
I f the average of N
1
quantities is x and N
2
quantities is y then the average of total (N
1
+ N
2
) quantities
is given by
1 2
1 2
( ) N x N y
N N
+
+
Average
CHAPTER 9
WWW.UPSCPORTAL.COM
For Any Help Call Our Course Director:
at +91 8800734161, 011- 65023618
Click Here to Order
http://www.upscportal.com/civilservices/study-kit/ias-pre-csat-paper-2
Example 1: What is the average of first five even numbers.
Sol uti on. The first prime even numbers are 2, 4, 6, 8, 10
Average =
2 4 6 8 10
5
+ + + +
=
30
5
=6
Example 2: The average of five consecutive even numbers is 50. What is the largest of these numbers?
Sol uti on. Let the numbers be x 4, x 2, x, x +2, x +4.
Average =
Sumof thequantities
Number of thequantities
=
4 2 2 4
5
x x x x x + + + + + +
=50

5
5
x
=50
x =50
So, the numbers are 46, 48, 50, 52, 54.
The largest of these numbers is 54.
Example 3: Average weight of 32 students of a class is 30.5 kg. I f weight of a teacher is also included then
average weight is increased by 500 g. What is the weight of the teacher?
Sol uti on. Total weight of 32 students =30.5 32 =976 kg
Average weigth of (32 students +1 teacher) =(30.5 +0.5) =31 kg
Total weight of (32 students +1 teacher) =31 33 =1023 kg
Weight of teacher =(1023 976) kg =47 kg
Example 4: The average salary per head of all the employees of an institution is Rs 60. The average salary
per head of 12 officers is ` 400 and average salary per head of the rest is ` 56. Find the total number of employees
in the institution.
Sol uti on. Let the total number of employees be x.
Then, 60 =
Total salary of all employees
x
60 =
12 400 ( 12) 56 x
x
+
60x =12 400 +(x 12) 56 =4800 +56x 672
60x 56x =4800 672
4x =4128 x =1032
Hence, the total number of employees is 1032.
Example 5: I f the average of p and q is 58 and the average of q and 5 is 64, what is the value of s p?
Sol uti on. Given,
2
p q +
=58 p +q =116 ...(i)
Also,
2
q s +
=64 q +s =128 ...(ii)
Subtracting Eq. (i) from (ii), we get
(q +s) (p +q) =128 116 s p =12
WWW.UPSCPORTAL.COM
For Any Help Call Our Course Director:
at +91 8800734161, 011- 65023618
Click Here to Order
http://www.upscportal.com/civilservices/study-kit/ias-pre-csat-paper-2
1. The average weight of 8 persons increases by 2.5
kg when a new person comes in place of one of
them weighing 65 kg. What might be the weight
of the new person?
(a) 82 kg (b) 85 kg
(c) 76.5 kg (d) 80 kg
2. Sumitra has an average of 56% on her first 7
examinations. How much she should make on her
eighth examination to obtain an average of 60%
on 8 examinations?
(a) 88% (b) 78%
(c) 92% (d) 68%
3. Ages of A and B are i n the rati o of 2 : 3
respectively. Six years hence the ratio of their ages
will become 8 : 11 respectively. What is Bs present
age?
(a) 18 years (b) 28 years
(c) 27 years (d) 25 years
4. The total age of A and B is 12 years more than
that of total age of B and C. C is how many years
younger than A?
(a) C is elder than A (b) 26
(c) 12 (d) 25
5. The average marks fetched by Mohan in History,
Geography, Science and Mathematics is 10 more
than the marks fetched in Mathematics. I f he has
got 110 marks aggregate i n Hi story and
Geography, what will be the aggregate marks
fetched in Science and Mathematics?
(a) 90 (b) 70
(c) 75 (d) 85
ANSWERS
1. (b) 2. (a) 3. (c) 4. (c) 5. (d)
EXPLANATIONS
1. By short cut: The weight of the new person =8
2.5 +65 =85 kg
2. By given condition,
56 7 +x =60 8
x =480 392 =88%
4. By given condition, A + B
=12 +B +C A C =12
C is 12 years younger than A.
5. By givern condition,
History + Geography + Science + Maths
4
=10 +Maths
(History +Geography) +Science 3 = 40
110 +Science 3 Maths = 40
3 Maths Science = 70
Since there is no relation given between Maths
and Science therefore the question cannot be
solved.
Example 6: 12 men went to a restaurant. 11 of them spent ` 5 each and the 12th person spent ` 11 more
than the average expenditure of all. Find the total money spent by them?
Sol uti on. Let the average money spent by the 12 men =` x
Money spent by the 12th man =` (x +11)
Money spent by the other 11 men =` (11 5) =` 55
Total money spent by the 12 men =` (55 +x +11) =` (x +66)
x =
66
12
x+
12x =x +66 11x =66 x =` 6
Total money spent by the 12 men =6 12 =` 72.
EXERCISE
Percentage
Per cent means per hundred. I t is given by % symbol. Here x% means x per hundred or
.
100
x
Thus, any
percentage can be converted into an equivalent fraction by dividing it by 100.
eg 20% =
20 1
;
100 5

150% =
150 3
100 2

Also, any fraction or decimal can be converted into its equivalent percentage by multiplying with 100.
eg
1 1
100
5 5

=20%;
3 3
100
2 2

=150%.
Important Formulae
1. Percentage increase =
I ncrease
100
Original value

2. Percentage decrease =
Decrease
100
Original value

3. I f the price of the commodity increases by r% then the reduction in consumption so as not to increase the
expenditure is
100 %
100
r
r
1

1
+
]
4. I f the price of the commodity decreases by r% then the reduction in consumption so as not to increase the
expenditure is
100 %
100
r
r
1

]
5. I f As income is r% more than Bs income then Bs income is less than As income by
100 %
100
r
r
1

1
+
]
.
6. I f As income is r% less than Bs income then Bs income is more than As income by
100 %
100
r
r
1

]
.
Percentage
CHAPTER 10
WWW.UPSCPORTAL.COM
For Any Help Call Our Course Director:
at +91 8800734161, 011- 65023618
Click Here to Order
http://www.upscportal.com/civilservices/study-kit/ias-pre-csat-paper-2
7. Let the population of a town be P and it increases at the rate of r% per annum, then
(a) Population after n years =
1
100
n
r
P
_
+

,
(b) Population n years ago =
1
100
n
P
r _
+

,
8. Let the present value of the machine be P and if it depreciates at the rate of r% per annum.
(a) Value of machine after n years =
1
100
n
r
P
_


,
(b) Value of machine n years ago =
1
100
n
n
r
P
_


,
Example 1: Express 3/2 as rate per cent.
Sol uti on.
3
2
=
3
100 %
2
_


,
=150%
Example 2: Find 25% of 1000.
Sol uti on. 25% of 1000 =
25
1000
100

=250
Example 3. What per cent of 6 is 144?
Sol uti on. Required percentage =
144
100 %
6
_


,
=2400%
Example 4: What per cent of 2.5 kg is 15 g?
Sol uti on. Required percentage =
15
100 %
2.5 1000
_

,
=0.6%
Example 5. I f the price of tea falls by 12%, by how much pr cent must a house holder increase its consumption,
so as not to decrease its expenditure on tea?
Sol uti on. (Short cut method)
I ncrease % in consumption =
100 %
100
r
r

' ;

)
=
12
100 %
100 12

' ;

)
=
12
100 %
88
_


,
=
150
%
11
=
7
13 %
11
Example 6: The value of a machine depreciates at the rate of 10% per annum. I f its present value is
` 162000, what was the value of the machine 2 year ago?
Sol uti on. Value of machine 2 year ago =`
2
162000
.
10
1
100
Rs
1
1
1
1
_
1


1
, ]
=`
10 10
. 162000
9 9
Rs
_


,
=` 200000
WWW.UPSCPORTAL.COM
For Any Help Call Our Course Director:
at +91 8800734161, 011- 65023618
Click Here to Order
http://www.upscportal.com/civilservices/study-kit/ias-pre-csat-paper-2
Example 7: Due to a reduction of 5% in prices of sugar, a man is able to buy 1 kg more for ` 95. Find the
original and reduced rate of sugar.
Sol uti on. Let the original rate be ` x per kg.
Reduced rate =`
1
(100 5)
100
x
=`
95
100
x

95 95
1
95
100
x
x


5
1
x

x =5
Original rate = ` 5 per kg
Reduced rate =`
19 5
.
20 1
Rs
_


,
per kg =`
19
.
4
Rs
=4.75 er kg
Example 8: I f the price of 1 kg cornflakes is increased by 25%, the increase is ` 10. Find the new price of
cornflakes per kg.
Sol uti on. Original price =
Differencein price
100
Differencein per cent

=
10
100
25

=400
New price =
125
40
100

=` 50
EXERCISE
1. The difference of two numbers is 20% of the larger
number. I f the smaller number is 20, then the
larger number is:
(a) 25 (b) 46
(c) 27 (d) 82
2. When any number is divided by 12, then dividend
becomes 1/4th of the other number. By how much
percent first number is greater than the second
number?
(a) 165 (b) 200
(c) 300 (d) 400
3. I f one number is 80% of the other and 4 times the
sum of their squares is 656, then the numbers
are:
(a) 6,8 (b) 8, 10
(c) 16, 20 (d) 10, 15
4. Two numbers A and B are such that the sum of
5% of A and 4% of B is two-third of the sum of 6%
of A and 8% of B. Find the ratio of A : B.
(a) 1 : 2 (b) 3 : 1
(c) 3 : 4 (d) 4 : 3
5. Three candi dates contested an el ection and
received 1136, 7636 and 11628 votes respectively.
What percentage of the total votes did the winning
candidate get?
(a) 57% (b) 77% (c) 80% (d) 90%
ANSWERS
1. (a) 2. (b) 3. (b) 4. (d) 5. (a)
EXPLANATIONS
1. Let the larger number be x.
Then, x 20 =
20 1
20
100 5
x x x

4
5
x
=20 x =
5
20
4

' ;
)
=25%
2. Let the numbers be x and y. Then,
3 .
12 4
x y
x y
Required percentage =
100 %
x y
y
_


,
=
2
100 %
y
y
_


,
=200%
3. Let one number =x. Then, other number = 80%
WWW.UPSCPORTAL.COM
For Any Help Call Our Course Director:
at +91 8800734161, 011- 65023618
Click Here to Order
http://www.upscportal.com/civilservices/study-kit/ias-pre-csat-paper-2
of x =
4
5

2
2
4
4
5
x x
1
_
+
1

,
1
]
=656
x =
2 2
16
25
x x +
=164

2
41
25
x
=164
x
2
=
164 25
41
_

,
=100
x =100
So, the numbers are 10 and 8.
4. 5% of A +4% of B =
2
3
(6% of A +8% of B)
5 4 2 6 8
100 100 3 100 100
A B A B
_
+ +

,
1 1 1 4
20 25 25 75
A B A B + +
1 1 100 4
100 75 75 3
A
A B
B
+
5. Total number of votes polled
=(1136 +7636 + 11628) =20400.
Required percentage =
11628
100 %
20400
_


,
=57%.
WWW.UPSCPORTAL.COM
For Any Help Call Our Course Director:
at +91 8800734161, 011- 65023618
Click Here to Order
http://www.upscportal.com/civilservices/study-kit/ias-pre-csat-paper-2
WWW.UPSCPORTAL.COM
For Any Help Call Our Course Director:
at +91 8800734161, 011- 65023618
Click Here to Order
http://www.upscportal.com/civilservices/study-kit/ias-pre-csat-paper-2
Relation between Time, Speed and Distance
Distance covered, time and speed are related by
Time =
Distance
Speed
...(i)
Speed =
Distance
Time
...(ii)
Distance =Speed Time ...(iii)
Distance is measured in metres, kilometres and miles.
Time in hours, minutes and seconds.
Speed in km/h, miles/h and m/s.
1. To convert speed of an object from km/h to m/s multiply the speed by
5
18
.
2. To convert speed of an object from m/s to km/h, multiply the speed by
18
5
.
Average Speed
I t is the ratio of total distance covered to total time of journey.
Average speed =
Total distancecovered
Total timeof journey
General Rules for Solving Time & Distance Problems
Rul e 1
I f a certain distance is covered with a speed of x km/h and another equal distance with a speed of y km/h,
then the average speed for the whole journey is the harmonic mean of the two speeds.
Average speed =
2
km/h
1 1
x y




+


=
2
km/h
xy
x y


+

Time and Distance
CHAPTER 16
WWW.UPSCPORTAL.COM
For Any Help Call Our Course Director:
at +91 8800734161, 011- 65023618
Click Here to Order
http://www.upscportal.com/civilservices/study-kit/ias-pre-csat-paper-2
Rul e 2
I f three equal distances are covered by three different speeds x, y and z km/h, then average speed for the
whole journey is given by
Average speed =
3
km/h
1 1 1
x y z




+ +


=
3
km/h
xy yz zx


+ +

Rul e 3
I f a certain distance is covered with a speed of x km/h and another distance with a speed of y km/h but time
interval for both journeys being same, then average speed for the whole journey is given by
Average Speed =
km/h
2
x y +


Rul e 4
I f a certain distance is covered with a speed of x, y and z km/h, but time inverval for the three journey being
equal, then average speed is given by
Average speed =
km/h
3
x y z + +


Rul e 5
I f the ratio of speeds A and B is x : y, then the ratio of times taken by them to cover the same distance is
1 1
: .
x y
Relative Speed
(i) I f two bodies are moving in the same direction at x km/h and y km/h, where (x > y), then their relative speed
is given by (x y) km/h.
(ii) I f two bodies are moving in opposite direction at x km/h and y km/h, then the their relative speed is given
by (x + y) km/h.
General Rules for Solving Train Problems
Rul e 1 Trai n Vs Stati onary Obj ect of no Length
Time taken by a train of length l metre to pass a stationary object such as a pole, standing man or a building
is equal to the time taken by the train to cover l metre.
Speed of the train =
Length of thetrain
Timetaken to cross thestationary object
Rul e 2 Trai n Vs Stati onary Obj ect of Certai n Length
Time taken by a train of length l metre to pass a stationary object of length a metre such as another
standing train, bridge or railway platform is equal to the time taken by the train to cover (l +a) metre.
Speed of the train =
Length of thetrain+Lengthof thestationary object
Timetaken tocross thestationary object
WWW.UPSCPORTAL.COM
For Any Help Call Our Course Director:
at +91 8800734161, 011- 65023618
Click Here to Order
http://www.upscportal.com/civilservices/study-kit/ias-pre-csat-paper-2
Rul e 3 Trai n Vs Movi ng Obj ect of no Length
Time taken by the train of length l metre to pass a man moving is equal to the time taken by the train to
cover l metre
(i) When the train and man move in the same direction with speeds of x m/s and y m/s. Then,
(x y) =
Length of thetrain
Timetaken to cross each other
(ii) When the train and man move in opposite directions with speeds of x m/s and y m/s. Then,
(x +y) =
Length of thetrain
Timetaken to cross each other
Rul e 4 Trai n Vs Movi ng Obj ect of Certai n Length
Time taken by the train of length l metre to pass a moving object of length a metre such as another moving
train is equal to the time taken by the train to cover (l +a) metre.
(i) When the two trains move in the same direction with speeds of x m/s and y m/s, (x > y), then
(x y) =
Length of thetrain +Length of train two
Timetaken to cross each other
(ii) When the two tains move in opposite directions with speeds of x m/s and y m/s. Then,
(x +y) =
Length of thetrain one+Length of train two
Timetaken to cross each other
Rul e 5 Two Movi ng Trai n
I f two trains start at the same time from points A and B towards each other and after crossing they take a and
b second in reaching B and A respectively. Then, (As speed) : (Bs speed) =
: b a
.
General Rules for Solving Boats and Streams Problem
Downstream Motion
When an object is moving against (opposite) direction in which the water in the stream is flowing, then the
bject is said to be moving upstream.
Upstream Motion
When an object is moving against (opposite) direction in which the water in the stream is flowing, then the
object is said to be moving upstream.
Motion in Still Water
When an object is moving in water where there is no motion in water, the object can move in any direction
with a uniform speed, then the object is said to be moving in still water.
Rul e 1 Downstream and Upstream Speed
Let the speed of the boat in still water =x km/h and speed of the stream be y km/h, then
Speed of the boat with stream downstream speed =(x +y) km/h
Speed of the boat against stream = upstream speed =(x y) km/h
As, when the boat is moving downstream, the speed of the water aids the speed of the boat and when the boat
is moving upstream, the speed of the water reduces the speed of the boat.
WWW.UPSCPORTAL.COM
For Any Help Call Our Course Director:
at +91 8800734161, 011- 65023618
Click Here to Order
http://www.upscportal.com/civilservices/study-kit/ias-pre-csat-paper-2
Rul e 2 Speed of Boat i n Sti l l Water & Speed of Stream
I f the downstream speed of boat is a km/h and the upstream speed of boat is b km/h, then
Speed of boat in still water =
1
( )km/h
2
a b +
Speed of stream =
1
( )km/h
2
a b
General Rules for Solving Circular Tracks
Rul e 1
When two people are running around a Circular Track starting at the same point and at the same time, then
whenever the two people meet the person moving with a greater speed covers one round more than the person
moving with lesser speed.
Rul e 2
When two people with speeds of x km/h and y km/h start at the same time and from the same point in the
same direction around a circular track of circumference c km, then
The time taken to meet for the first time anywhere on the track =
h
c
x y
The time taken to meet for the first time at the starting point =LCM of
,
c c
h
x y



Rul e 3
When two people with speeds of x km/h and y km/h respectively start at the same time and from the same
point but in opposite direction around a circular track of circumference c km, then
The time taken to meet for the first time anywhere on the track =
h
c
x y +
The time taken to meet for the first time at the starting point =LCM of
, h
c c
x y



Example 1: Convert 90 km/h into m/s.
Sol uti on. 90 km/h =
5
90 m/s
15



=25 m/s
Example 2: Convert 10 m/s into km/h.
Sol uti on. 10 m/s =
18
10 m/s
5



=36 km
Example 3: A man can cover a certain distance in 1 h 30 min by covering one-third of the distance at 6 km/
h and the rest at 15 km/h. Find the total distance.
Sol uti on. Let the total distance be x km. Then,
2
3 3
6 15
x x
+
=
3
2

2
18 45
x x
+
=
3
2

9
90
x
=
3
2

10
x
=
3
2
WWW.UPSCPORTAL.COM
For Any Help Call Our Course Director:
at +91 8800734161, 011- 65023618
Click Here to Order
http://www.upscportal.com/civilservices/study-kit/ias-pre-csat-paper-2
x =
(3 10)
2

=15
Total distance =15 km
Example 4: An aeroplane started one hour later than the scheduled departure from a place 1200 km away
from its destination. To reach the distination on time, the pilot had to increase its speed by 200 km/h. What was
the normal speed of the aeroplane?
Solution. Let the time taken by the aeroplane in second case be x hour. Then,
1200
x
=
1200
200
1 x
+
+

6
x
=
6
1
1 x
+
+
6x +6 =6x +x
2
+x x
2
+x 6 =0
(x +3) (x 2) =x x =2h (
Q
=3 is not possible)
Time taken in second case =2 h
So, Speed =
1200
2
=600 km/h
Hence, normal speed =600 200 =400 km/h
Example 5: The current of a stream runs at 1 km/h. A motor boat goes 35 km upstream and back again at
the starting point in 12 h. What is the speed of motor boat in still water?
Sol uti on. Let speed of boat in still water be x km/h. Then,
35 35
1 1 x x
+
+
=12 35 (x +1 +x 1) =12 (x
2
1) 6x
2
35x 6 =0
(x 6) (6x +1) =0 x =6 (Q x =1/6 is not possible)
So, speed of boat in still water =6 km/h
Example 6: A train of length 100 m crosses a man who is coming to the train from opposite direction, in 6 s.
What is the speed of train?
Sol uti on. Let speed of train =x km/h
Then, speed of tain relative to man =(x +5) km/h
=
5
( 5) m/s
18
x+

100
5
( 5)
18
x+
=6
1800
5( 5) x+
=6
x +5 =60 x =55 km/h
Example 7: Speed of three cars are in the ratio 2 : 3 : 4. What is the ratio of time taken by them in covering
the same distance.
Sol uti on. Let speed of three cars be 2x, 3x and 4x km/h. I f covered distance be d, then ratio of time taken by
them
=
; ;
2 3 4
d d d
x x x
=
1 1 1
: :
2 3 4
=6 : 4 : 3
Example 8: A man can row 6 km/h in still waer. When the river is running at 4 km/h, it takes him 2 h 15 min
to row to a place and back. How far is the place?
Sol uti on. Speed downstream =(6 +4) km/h =10 km/h
WWW.UPSCPORTAL.COM
For Any Help Call Our Course Director:
at +91 8800734161, 011- 65023618
Click Here to Order
http://www.upscportal.com/civilservices/study-kit/ias-pre-csat-paper-2
Speed upstream =(6 4) km/h =2 km/h
Let the required distance be x km.
Then,
10 2
x x
+
=
9
4
x =
9 10
4 6


=3.75 km
Example 9: Two men A and B start together from the same point to walk round a circular path 8 km long. A
walks 2 km and B walks 4 km an hour. When will they next meet at the starting point, if they walks in the same
direction?
Solution. Time to complete one revolution by A and B is
8
h
2



and
8
h
4



or 4 h and 2h.
The required time is the LCM of 4 and 2 which is 4 h.
Thus, they will next time at the starting point after 4 h.
EXERCISE
5. A, B and C are on a trip by a car. A drives during
the first hour at an average speed of 50 km/hr. B
drives during the next 2 hours at an average
speed of 48 km/hr. C drives for the next 3 hours
at an average speed of 52 km/hr. They reached
their destination after exactly 6 hours. Their mean
speed was:
(a) 50 km/hr (b)
1
50
3
km/hr
(c) 51 km/hr (d) 52 km/hr
ANSWERS
1. (d) 2. (c) 3. (d) 4. (a) 5. (b)
EXPLANATIONS
1. Speed from A to B =
2
250
11



mph =
500
11



mph.
Speed from B to A =
2
250
9



mph =
500
9



mph.
Average speed =
500 500
2
11 9
500 500
11 9




+

mph
=
500000
4500 5500



+
mph =50 mph.
1. Mac travels from A to B a distance of 250 miles in
5 hours. He returns to A in 4 hours 30 minutes.
His average speed is:
(a) 42 mph (b) 49 mph
(c) 48 mph (d) 50 mph
2. A boy goes to his school from his house at a speed
of 3 km/hr and returns at a speed of 2 km/hr. I f
he takes 5 hours in going and coming, the distance
between his house and school is:
(a) 8.5 km (b) 5.5 km
(c) 6 km (d) 9 km
3. The average speed of a trai n i n the onward
journey is 25% more than that in the return
journey. The train halts for one hour on reaching
the destination. The total time taken for the
complete to and for journey is 17 hours, covering
a distance of 800 km. The speed of the train in
the onward journey is:
(a) 50 km/hr (b) 53 km/hr
(c) 52 km/hr (d) 56.25 km/hr
4. I started on my bicycle at 7 a.m. to reach a certain
place. After going a certain distance, my bicycle
went out of order. Consequently, I rested for 35
minutes and came back to my house walking all
the way. I reached my house at 1 p.m. I f my
cycling speed is 10 kmph and my walking speed
is 1 kmph, then on my bicycle I covered a distance
of:
(a)
61
4
66
km (b)
4
13
3
km
(c)
3
5
6
km (d)
7
15
8
km
WWW.UPSCPORTAL.COM
For Any Help Call Our Course Director:
at +91 8800734161, 011- 65023618
Click Here to Order
http://www.upscportal.com/civilservices/study-kit/ias-pre-csat-paper-2
2. Average speed =
2 3 2
3 2



+
km/hr =
12
5
km/hr.
Distance travelled =
12
5
5



km =12 km.
Distance between house and school
=
12
2



=6 km.
3. Let the speed in return journey be x km/hr.
Then, speed in onward journey =
125
100
x
=
5
4
x



km/hr.
Average speed =

5
2
4
5
4
x x
x x




+


km/hr
=
10
9
x
km/hr.

9
800
10x



=16
x =
800 9
16 10


=
7200
160
=45.
So, speed in onward journey
=
5
45
4



km/hr.
=
225
4
=56.25 km/hr.
4. Time taken =5 hrs 25 min =
65
12
hrs.
Let the required distance be x km.
Then,
10 1
x x
+
=
65
12
11x=
650
12
x =
325
66
=
61
4
66
km.
5. Total distance traveled
=(50 1 +48 2 +52 3) km
=(50 +96 +156) km =302 km.
Total time taken =6 hrs.
Mean speed =
302
6



km/hr
=50
1
3
km/hr.
WWW.UPSCPORTAL.COM
For Any Help Call Our Course Director:
at +91 8800734161, 011- 65023618
Click Here to Order
http://www.upscportal.com/civilservices/study-kit/ias-pre-csat-paper-2
Probability
Probability is used to indicate a possibility of an event to occur. I t is often used synonymously with chance.
(i) I n any experiment if the result of an experiment is unique or certain, then the experiment is said to be
deterministic in nature.
(ii) I f the result of the experiment is not unique and can be one of the several possible outcomes then the
experiment is said to be probabilistic in nature.
Various Terms Used in Defining Probability
(i) Random Experiment: Whenever an experiment is conducted any number of times under identical
conditions and if the result is not certain and is any one of the several possible outcomes, the experiment is
called a trial or a random experiment, the outcomes are known as events.
eg, When a die is thrown is a trial, getting a number 1 or 2 or 3 or 4 or 5 or 6 is an event.
(ii) Equally Likely Events: Events are said to be equally likely when there is no reason to expect any one of
them rather than any one of the others.
eg, When a die is thrown any number 1 or 2 or 3 or 4 or 5 or 6 may occur. I n this trial, the six events are
equally likely.
(iii) Exhaustive Events: All the possible events in any trial are known as exhaustive events. eg, When a die
is thrown, there are six exhaustive events.
(iv) Mutually Exclusive Events: I f the occurrence of any one of the events in a trial prevents the occurrence
of any one of the others, then the events are said to be mutually exclusive events. eg, When a die is thrown
the event of getting faces numbered 1 to 6 are mutually exclusive.
Classical Definition of Probability
I f in a random experiment, there are n mutually exclusive and equally likely elementary events in which n
elementary events are favourable to a particular event E, then the probability of the event E is defined as P (E)
P(E) =
Favourabel Events
Total number of Events
=
( )
( )
n E
m
n S n
=
I f the probability of occurrence of an event E is P(E) and the probability of non-occurrence is P ( ) E , then,
Probability
CHAPTER 25
WWW.UPSCPORTAL.COM
For Any Help Call Our Course Director:
at +91 8800734161, 011- 65023618
Click Here to Order
http://www.upscportal.com/civilservices/study-kit/ias-pre-csat-paper-2
P(E) + P ( ) E =1. Hence, P ( ) E =
1 ,
m n m
ie
n n

=
the sum of the probabilities of success and failure is 1.
Also, 0 P(E) 1 and 0 P ( ) E 1.
I f P(E) = 1, the event E is called a certain event and if P(E) = 0, the event E is called an impossible event.
I f E is an event, then the odds in favour of E are defined as P(E) : P(E) and the odds against E are defined
as P(E): P ( ) E . Hence, the odds in favour of E are
( )
( ) : : ,
n m
m
m n m
n n

= the odds against E are


( )
:
n m
m
n n

=(n m):m
Addition Theorem on Probability
I f E
l
and E
2
are two events in a sample space S, then P (E
l
E
2
) =P (E
l
) +P (E
2
) P (E
l
E
2
). I f E
1
and E
2
are
mutually exclusive events (disjoint), then P(E
l
E
2
) =P (E
l
) +P (E
2
) . (
Q
P(E
l
E
2
) =)
Independent and Dependent Events
Two or more events are said to be independent if the happening or non-happening of any one does not
depend (or not affected) by the happening or non-happening of any other. Otherwise they are called dependent
events.
eg, Suppose a card is drawn from a pack of cards and replaced before a second card is drawn. The result of the
second drawn is independent of the first drawn. I f the first card drawn is not replaced, then the second drawn is
dependent on the first drawn.
I f E
l
and E
2
are independent events, then
P(E
l
E
2
) =P(E
l
) P(E
2
)
Simple Event
An event which cannot be further split is called a simple event. The set of all simple events in a trial is called
a sample space.
Compound Event
When two or more events occur in relation with each other, they are called compound events.
Conditional Event
I f E
l
and E
2
are events of a sample space S and if E
2
occurs after the occurrence of E
l
, then the event of
occurrence of E
2
after the event E
l
is called conditional event of E
2
given E
l
. I t is denoted by E
2
/E
l
.
Conditional Probability
I f E
l
and E
2
are the events in a sample space S and P(E
l
) 0, then the probability of E
2
after the event E
l
has
occurred is called conditional probability of E
2
given E
l
. I t is denoted by
2
1
E
P
E



and we define,
2 2 2 1 2
1 1 1
( ) ( )
( ) ( )
E P E E n E E
P
E P E n E

= =


WWW.UPSCPORTAL.COM
For Any Help Call Our Course Director:
at +91 8800734161, 011- 65023618
Click Here to Order
http://www.upscportal.com/civilservices/study-kit/ias-pre-csat-paper-2
Smart Facts
When a die is rolled six events occur. They are {1, 2, 3, 4, 5 and 6}
When two dice are rolled 36 events occur. They are [(1,1), (1,2), (1,3), (1,4), (1,5), (1,6), (2,1), (2,2),
(2,3), (2,4), (2,5), (2,6), (3,1), (3,2), (3,3), (3,4), (3,5), (3,6), (4,1), (4,2), (4,3), (4,4), (4,5), (4,6), (5,1),
(5,2), (5,3), (5,4), (5,5), (5,6), (6,1), (6,2), (6,3), (6,4), (6,5), (6,6)]
When a coin is tossed 2 events occur. They are {H, T}
When two coins are tossed 4 events occur. They are {HH, HT, TH, T T}
When three coins are tossed 8 events occur. They are {HHH HHT, HTH, HT T, T HH, THT, T TH, T
T T}
I n a pack of 52 cards there are 26 red cards and 26 black cards. The 26 red cards are divided into 13
heart cards and 13 diamond cards. The 26 black cards are divided into 13 club cards and 13 spade
card. Each of the colours, hearts, diamonds, clubs and spades is called a suit. I n a suit, we have 13
cards (ie, A, K, Q, J , 10, 9, 8, 7, 6, 5, 4, 3 and 2)
Example 1: I n a toss of a coin, find the probability of getting a tail.
Solution. Here, S =[H, T] and E =[T] P(E) =
( )
( )
n E
n S
=
1
2
Example 2: Two unbiased coins are tossed, what is the probability of getting both heads.
Sol uti on. Here, S =[HH, HT, TH, TT] and E =[HH]
P(E) =
( )
( )
n E
n S
=
1
4
Example 3: I n a simultaneous throw of a pair of dice, find the probability of getting a total more than 9.
Sol uti on. Here n(S) =6 6 =36
Let E =Event of getting a total more than 9 =[(4, 6), (5, 5), (5, 6), (6, 5), (6, 4), (6, 6)]
P(E) =
( )
( )
n E
n S
=
6
36
=
1
6
Example 4: I n a simultaneous throw of a pair of dice, find the probability that the sum of numbers shown on
the two faces is divisible by 5 or 6.
Sol uti on. Here, n(S) =6 6 =36
Let E =Event of getting a sum of numbers shown on the two faces divisible by 5 or 6.
=[(1, 4), (1, 5), (2, 3), (2, 4), (3, 2), (3, 3), (4, 1), (4, 2), (5, 1), (5, 5), (6, 4), (6, 6)]
N(E) =12
P(E) =
( )
( )
n E
n S
=
12
36
=
1
3
Example 5: A card is drawn from a well shuffled pack of cards. Find the probability that it is a (i) queen (ii)
a red card (iii) a space.
(i) Let E be event of drawing a queen card. Then, one queen card can be drawn from 4 queens in
4
C
1
ways.
P(E) =
( )
( )
n E
n S
=
4
1
52
1
C
C
=
4 1
52 13
=
(ii) Let E be the event of drawing a spade card. Then, one spade card can be drawn from 13 spade cards in
13
C
1
ways.
WWW.UPSCPORTAL.COM
For Any Help Call Our Course Director:
at +91 8800734161, 011- 65023618
Click Here to Order
http://www.upscportal.com/civilservices/study-kit/ias-pre-csat-paper-2
P(E) =
( )
( )
n E
n S
=
26
1
52
1
C
C
=
13 1
52 4
=
(iii) Let E be the event of drawing a spade card. Then, one spade card can be drawn from 13 spade cards in
13
C
1
ways.
P(E) =
( )
( )
n E
n S
=
13
1
52
1
C
C
=
13 1
52 4
=
Example 6: I f three cards are drawn simultaneously from a pack of well shuffled cards, then find the
probability of them being (i) all queens (ii) all red cards (iii) all spades.
Sol uti on. The total number of ways of selecting three cards from 52 cards is
53
C
3
ways.
(i) Let E be event of drawing the queen cards. Then, three queen cards can be drawn from 4 queen cards in
4
C
3
ways.
P(E) =
( )
( )
n E
n S
=
4
3
52
3
C
C
=
4 1
22100 5525
=
(ii) Let E be event of drawing the red cards. Then, three red cards can be drawn from 26 red cards i
26
C
3
ways.
P(E) =
( )
( )
n E
n S
=
26
3
52
3
C
C
=
2600 26
22100 221
=
(iii) Let E be event of drawing spade cards. Then, three spade cards can be drawn from 13 spade cards in
13C3 ways.
P(E) =
( )
( )
n E
n S
=
13
3
52
3
C
C
=
286
22100
=
143
11050
Example 7: A card is drawn at random from a normal pack of cards. What is the probability that it is either
a diamond or a king?
Sol uti on. Out of 52 cards one card can be drawn in
52
C
1
ways.
Let E
1
be the event that the card drawn is a diamond, E
2
be the event that the card drawn is king and (E
1

E
2
) be the event that the card drawn is both diamond the king and S be the sample space.
P(E
1
) =
1
( )
( )
n E
n S
=
13
1
52
1
13
52
C
C
=
P(E
2
) =(
2
( )
( )
n E
n S
=
4
1
52
1
4
52
C
C
=
P (E
1
E
2
) =
1
1
52
1
1
52
C
C
=
P (E
1
E
2
) =P(E
1
) +P(E
2
) P (E
1
E
2
)=
13 4 1
52 52 52
+
=
16 4
52 13
=
Example 8: A bag contains 4 red balls and 5 white balls. a ball is drawn at random. Find the probability that
it is a red ball or a white ball.
WWW.UPSCPORTAL.COM
For Any Help Call Our Course Director:
at +91 8800734161, 011- 65023618
Click Here to Order
http://www.upscportal.com/civilservices/study-kit/ias-pre-csat-paper-2
P(E
1
) =
1
( )
( )
n E
n S
=
4
1
9
1
C
C
=
4
9
P(E
2
) =
2
( )
( )
n E
n S
=
5
1
9
1
C
C
=
5
9
P(E
1
) = 0
P(E
1
E
2
) =P(E
1
) +P(E
2
) =
4 5
9 9
+
=
9
1
9
=
Example 9: A bag contains 4 red balls and 4 white balls. Two balls are drawn in succession from the bag
with replacement, what is the probability that the two balls are of different colours?
Sol uti on. Let E
1
and E
2
denote the events of drawing balls of different colours in the first and the second
draw with replacement and S be the sample space.
P(E
1
) =
1
( )
( )
n E
n S
=
4
1
8
1
C
C
=
4 1
8 2
=
P(E
2
) =
2
( )
( )
n E
n S
=
4
1
8
1
C
C
=
4 1
8 2
=
P(E
1
E
2
) =P(E
1
) P(E
2
) =
1 1 1
2 2 4
=
Example 10: A family has two children. What is the probability that both the children are boys given that at
least one of them is a boy?
Sol uti on. Let b stand for boy and g for girl. The sample space of the experiement is
S ={(b, b), (g, b), (b, g), (g, g)}
Let E and F denote the following events:
E : both the children are boys
F : at least one of the child is a boy
Then, E ={(b, b)}and F ={(b, b), (g, b), (b, g)}
Now, E F ={(b, b)}
Thus, P(F) =
3
and ( )
4
P E F
=
1
4
Therefore, P(E| F) =
( )
( )
P E F
P F

=
1
4
3
4
=
1
3
Example 11: I n a school, there are 1000 students, out of which 430 are girls. I t is known that out of 430,
10% of the girls study in class XI I . What is the probability that a students choosen randomly studies in Class XI I
given that the choosen student is a girl?
Sol uti on. Let E denote the event that a student choosen randomly studies in Class XI I and F be the event
that the randomly choosen student is a girl. We have to find P(E| F).
WWW.UPSCPORTAL.COM
For Any Help Call Our Course Director:
at +91 8800734161, 011- 65023618
Click Here to Order
http://www.upscportal.com/civilservices/study-kit/ias-pre-csat-paper-2
Now, P(F) =
430
1000
=0.43 and P(E F) =
43
0.043
1000
=
[
Q
no. of girls studying in XI I 10% of 430 =43]
Then, P(E| F) =
( )
( )
P E F
P F

=
0.043
0.43
=0.1
Example 12: A person has undertaken a construction job. The probabilities are 0.65 that there will be
strike, 0.80 that the construction job will be completed on time if there is no strike, and 0.32 that the construction
job will be completed on time if there is no strike, and 0.32 that the construction job will be completed on time if
there is a strike. Determine that probability that the construction job will be completed on time.
Sol uti on. Let a be the event that the construction job will be completed on time, and B be the event that
there will be a strike. We have to find P(A).
We have P(B) =0.65, P(no strike) =P(B) =1 P(B) =1 0.65 =0.35
P(A| B) =0.32, P(A| B) =0.80
Since events B and B form a partition of the sample space S therefore, by theorem on total probability, we
have
P(A) =P(B) P(A| B) +P(B) P(A| B)
=0.65 0.32 +0.35 0.8 =0.208 +0.28 =0.488
Thus, the probability that the construction job will be completed in time is 0.488.
EXERCISE
1. Which of the following cannot be the probability
of an event ?
(a) 1/4 (b) 20% (c) 1.2 (d) 0.3
2. I f P(E) =0.03, what is the probability of not E?
(a) 0.90 (b) 0.97 (c) 0.07 (d) 0.70
3. A bag contains orange flavoured candies only. A
girl takes out one candy without looking into the
bag. What is the probability that she takes out
an orange flavoured candy ?
(a) 1 (b) 0
(c) 1/2 (d) 1/4
4. A bag contains orange flavoured candies only. A
girl takes out one candy at random from the bag.
What is the probability that she takes out a
strawberry flavoured candy ?
(a)
1
3
(b) 1 (c) 0 (d)
1
2
5. An unbiased die is thrown once. What is the
probability of getting a prime number ?
(a)
1
2
(b)
1
4
(c)
2
3
(d)
1
3
ANSWERS
1. (c) 2. (b) 3. (a) 4. (c) 5. (a)
EXPLANATIONS
1. 1.2 cannot be the probability of an event because
0 P (E) 1
2. Probability of not E
P (not E) =
( ) p E
=1 P(E) =1 0.03 =0.97
3. Probability that the girl takes out an orange
flavoured candy is 1 because the bag contains
orange flavoured candies only.
4. Probabi l ity that she takes out a strawberry
flavoured candy is 0 because the bag contains only
orange flavoured candies.
5. Here, n(S) = {1, 2, 3, 4, 5, 6}and E =Event of
getting a prime number ={2, 3, 5)
P(E) =
( )
( )
n E
n S
=
3
6
=
1
2
WWW.UPSCPORTAL.COM
For Any Help Call Our Course Director:
at +91 8800734161, 011- 65023618
Click Here to Order
http://www.upscportal.com/civilservices/study-kit/ias-pre-csat-paper-2
Sets
A set is a collection of well defined objects.
The objects of the sets are called elements.
(i) Sets are usually denoted by capital letters A, B, C,..., X, Y, Z.
(ii) The elements of the sets are denoted by small letters like a, b, c,..., x, y, z etc.
Representation of Sets
Sets are usually described into two ways.
(i) Tabular form or roster form, in this form, all the elements of the set are separated by commas and enclosed
between the bracket {}.
For example
(a) The set of vowels of English Alphabet as
A ={a, e, i, o, u)
(b) The set of numbers on a clock face is written as
B =(1, 2, 3, 4, 5, 6, 7, 8, 9, 10, 11, 12}
(ii) Set builder from: We define a set by stating properties which its elements must satisfy. For example the
set of all even integers. Then, we use the letters usually x, and we write
A ={x | x is an even integer}
This is to be read as A is a set of numbers x such that x is an even integer. The vertical line | to be read as
such that some times we use x in place of vertical line.
A ={x : x is an even integer}
eg, C ={1,,
2
}= {x | x
3
1 =0}
I f an object x is an element of a set A, we write x A which is read as x belong to A and if an object x is not
a member of A we write x A and read as x does not belong to A.
Some Important Terms
(i) Empty or Null set The set which contains no elements is called the empty set or the null set. The empty
set is written as .
Thus, ={}as there is no element in the empty set.
For example; the set of odd numbers divisible by 2 is the null set.
Set Theory,
Venn Diagrams,
Functions & Relations
CHAPTER 27
WWW.UPSCPORTAL.COM
For Any Help Call Our Course Director:
at +91 8800734161, 011- 65023618
Click Here to Order
http://www.upscportal.com/civilservices/study-kit/ias-pre-csat-paper-2
(ii) Singleton set A set containing only one element is called a singleton for example, {1}, {4}are singleton
sets.
(iii) Equality of sets. The sets A and B are equal if they have same members that is if every elements of A is
an element of B and every element of B is an element of A, then A = B
eg, if A ={l, 3,5,7}and B ={7, 3, 1, 5}, then A = B
I f the two sets are not equal we write A B
Important Formulae
1. A set does not change if its elements, are repeated.
2. A set does not change even if the order of its elements is different.
(iv) Finite and Infinite set. The set which contains a definite number of elements is called a finite set. The set
which contains an infinite number of elements is called an infinite set.
eg, (I ) The set of days in a week.
eg, (I I ) The set of natural numbers.
(v) Disjoint set. Two sets A and B are said to be disjoint, if they do not have any element in common.
eg, A ={1, 2, 3}, B ={4, 5, 6}are disjoint sets.
(vi) Subset. I f every element in set A is also an element of another set B. Then A is called a subset of B. Also B
is said to be super set of A.
Symbolically, we write
A B (ie, A contained in B)
B A (ie, B contains A)
More specifically A B if x A x B
eg, (I ) Let A ={2, 4, 7}, B ={1, 2, 3, 4, 7}
Then, A B since every element of A is in B.
eg, (I I ) A ={x | x a real number}and B ={x | x is an integer}Then, A B
1. I f there is at least one element of A which is not in B, then A is not a subset of B written as A B.
2. Every set is a subset of itself ie, A A.
3. I f A B and B c A, then A =B.
(vii) The Null set is a subset of every set A.
(viii) Proper Subset: A is a proper subset of B. if A B and A B and is written as A B ie, if B contains at least
one element more than A, then A is a proper subset of B
(ix) Power set: Set of all the subsets of a set is called the power set
eg, A ={a, b, c}subsets of A are , {a}, {b}, {c}, {a, b}, {b, c}, {c,a}, {a, b, c}
Hence, P(A) = [, {a}, {b}, {c}, {a, b}, {b, c}, {c, a}, {a, b, c}]
I f n is the number of elements of a set A, then the number of subset of A ie, the number of elements of P (A)
=2
n
.
(x) Universal set: I f all the sets under consideration are the subsets of a fixed set U, then U is called the
Universal set.
Union of sets
Union of two sets A and B is the set of all elements which belongs to A or B (or to both) and is written as
A B (ie, A union B)
The same is defined in set builder form as
A B ={x| x A or x B}
WWW.UPSCPORTAL.COM
For Any Help Call Our Course Director:
at +91 8800734161, 011- 65023618
Click Here to Order
http://www.upscportal.com/civilservices/study-kit/ias-pre-csat-paper-2
I f A ={1, 3, 5, 7, 9}and B ={2, 4, 5, 6, 9}
Then, A B ={1, 2, 3, 4, 5, 6, 7, 9}
1. From the definition of Union of sets A u B =B u A (Commutative Law)
I f A is any set, then A A = A and A =A
2. I f A and B are any two sets, then A (A B) and B (A B)
I f x A B,then x A or x B and if x A B,then x A and x B.
3. I f A, B, C are three sets, then A (B C) = (A B) C
Intersection of Sets
I f A and B are any two sets, then intersection of A and B is the set of all elements which are in A and also in
B. I t is written as A B and is read as A intersection B
I f A ={2, 4, 6, 8}and B ={4, 5, 6, 9}
Then A B ={4, 6}
1. From the definition of the intersection, it follows A B = B A (Commutative Law)
2. I f A is any set, then A A =A and A f ) = f
3. For any two sets A and B.
A B =A and A B B
4 I f A and B have no elements in common ie, A and B are disjoint, then A B =
I f x A B =x A and x B
eg, (I ) I f A ={2, 3, 6, 8, 9}and B =(1, 3, 5, 6, 7, 9}, then A B ={3, 6, 9}
eg,(I I ) I f A ={x1| <x <4 }and B ={x| 2 <x <5}, then A B ={x| 2 <x <4}
I f A, B, C are three sets, then
(i) (A B) C = A (B C) Associative Law
(ii) A (B C) = (A B) (A C) Distributive Law
Difference of Sets
The difference of two sets A and B is set of elements which belongs to A but do not belong to B. This is written
as A B
A B ={x| x A and x B}
1. Set A B subset of A ie, A B A
2. Set (A B) and B are disjoint ie, (A B) B =
3. A B =(A B) (A B)
Symmetric Difference of Sets
The symmetric difference of two sets A and B is (A B) (B A) and is written as A B
Thus, A B =(A B) (B A)
I n the set builder form A B ={x | x A or x B, but x A B}
Demorgan Laws
I f A, B, C are three sets, then
(i) A (B C) = (A B) (A C)
(ii) A (B C) = (A B) (A C)
WWW.UPSCPORTAL.COM
For Any Help Call Our Course Director:
at +91 8800734161, 011- 65023618
Click Here to Order
http://www.upscportal.com/civilservices/study-kit/ias-pre-csat-paper-2
Complement of a Set
Let A be a subset of universal set U, then the complement of A is denoted by A
C
is defined by
A
C
={x U, x A} x A
C
x A
eg, (I ) I f U ={1, 2, 3, 4, 5, 6}and A ={1, 3, 5}, then A
C
={2, 4, 6}
eg, (I I ) U be the set of all letters in English alphabet and A is a set of all vowels, then A
C
is the set of all
consonants.
1. (A B)
C
= A
C
B
C
2. (A B)
C
= A
c
B
C
Venn Diagrams
A simple way of explaining the relation between sets is by a diagram which is called Venn diagram. I n this a
set is generally represented by a circle and its elements by points in the circle.
CaseI: A U and B U and A B
A B
U
Here A and B are represented by a circle.
A B is the lined region
B A is dotted region and A B is plane region.
Case II: A ={a, c, e}, B ={b, d}
f
g
a b
c
d
e
A B
A B = and A B =A and B A =B
Case III: When A B U
I n adjoining figure, in Venn diagram
A
B
U
A B =B, A B =A and A B =
Some results from the Venn diagram
(i) n(A B) =n (a) +n(B) n(A B)
(ii) n(A B) = n (a) +n(B), when A B =
(iii) n(A B) + n (A B) = n(A)
(iv) n(B A) + n (A B) = n(A)
(v) n(A B) + n (A B) + n (B A) = n (A B)
WWW.UPSCPORTAL.COM
For Any Help Call Our Course Director:
at +91 8800734161, 011- 65023618
Click Here to Order
http://www.upscportal.com/civilservices/study-kit/ias-pre-csat-paper-2
Example 1: I f in a factory of 30 workers, 10 take tea but not coffee and 14 take tea. Then how many take
only coffee ?
Sol uti on. Total number of workers =n (T C) =30
Number of workers who take tea n(T) =14
n(T)
10 4 x
n
U = 30
( ) c
Who take tea but not coffee =n(T C) =10
Who drinks both coffee and tea =n (T) n (T C) =14 10 =4
Who takes only coffee =n (C T) =x
From the figure =x +4 +10 =30 =x =30 14 =16
The worker who drinks only coffee =16
Example 2: An elocution competition was held in English and Hindi. Out of 80 students, 45 took part in
English, 35 in Hindi, 15 in both English and Hindi, then for the number of students.
(a) Who took part in English but not in Hindi.
n(E) = 45
n(H) = 35
only E only H
(b) Who took part in Hindi but not in English.
(c) Who took part in either English or Hindi.
(d) Who took part in neither.
Sol uti on. Suppose E is the set of students who took part in English, His the set of students who took part in
Hindi, then E n H gives the set of students who took part in both English and Hindi.
(a) The number of students who took part in English but not in Hindi
=n(E) =n(E H) = 45 15 = 30
(b) The number of students who took part in Hindi but not in English
=n(H) n(E H) = 35 15 = 20
(c) The number of students who took part either in English or in Hindi is
n(E H) = n(E) + n(H) n(E H) = 45 + 35 15 = 65
(d) The number of students who took part neither in English nor in Hindi
=n (S) n (T H) = 80 65 = 15
Ordered Pair
I f a, b be any two objects, then the pair (a, b) is called the ordered pair. The object a is called the first
coordinate (or first number) and b is called the second coordinate (or second number) of the ordered pair (a, b).
1. The ordered pair (a, b) (b, a)
Two ordered pairs (a, b) and (c, d) are said to be equal, if and only if a = c and b = d.
WWW.UPSCPORTAL.COM
For Any Help Call Our Course Director:
at +91 8800734161, 011- 65023618
Click Here to Order
http://www.upscportal.com/civilservices/study-kit/ias-pre-csat-paper-2
Cartesian Product of Sets
I f A and B be any two sets, then cartesian product of A and B is the set of all ordered pair (a, b), where a
and b B
Cartesian product of A and B is written as A B (ie, A cross B)
ie A B ={(a, b) | a A and b B}
eg, I f A ={a, b, c}and B ={1, 2}, then
A B ={(a, 1), (a, 2),(b, 1),(b, 2),(c, 1),(c, 2)}
B A ={(1, a),(1, b),(1, c),(2, a),(2, b),(2, c)}
Thus, A B B A
A A ={(a, a), (a, b), (a, c),(b, a),(b, b),(b, c) (c, a),(c, b),(c, c)}
B B ={(1, 1), (1, 2),(2, 1),(2, 2))
1. A (B C) = (A B) (A C)
2. A (B C) = (A B) (A C)
Relations
A relation is a set of ordered pairs. I f (x, y) is a member of a relation R, we write it as x R y (ie relation R to y).
eg, I f R is the ordered pairs of positive integers where R ={(x, y)| x
2
=y}The relation is y is a square of x and
the set is {(1, 1), (2, 4),(3, 9),(4, 16),...}
Types of Relations
(i) Reflexive: A relation R on a set A is said to be reflexive for every x A
(x, x) R
(ii) Symmetric Relation: A relation R on a set A is said to be symmetric if x R y y R x
(x, y) R =(y,x) R
eg, Let A ={1, 2, 3}and R ={(1, 1), (2, 2),(1, 3),(3,1)}
Clearly, R is a symmetric relation.
(iii) Transitive Relation: A relation R in a set A is called transitive if x R y and y R z x R z
eg, Let R be a relation in the real number defined by x less than y then
x <y and y <z =x <z
(iv) Equivalence Relation A relation which is reflexive, symmetric and transitive is a equivalence relation.
Functions
I f each element of a set A is associated with exactly one element in the set B, then this association is called a
function from A to B.
The set A is called the domain and the set B is called the co-domain of the function.
Consider : A ={1, 2}, and B =(3, 4, 5, 6), then {(1, 4),(2, 5)) is a function
{(1, 4),(2, 5),(2, 6)}is not a function since element 2 in the set A have two images 5 and 6 in the set B
1. Each element of A must be associated with exactly one element in the set B.
2. All the elements of the set B need not have the association.
3. The set of elements of B which are associated with the elements of the set A is called the range of the
function.
4. The range is the subset of the co-domain.
WWW.UPSCPORTAL.COM
For Any Help Call Our Course Director:
at +91 8800734161, 011- 65023618
Click Here to Order
http://www.upscportal.com/civilservices/study-kit/ias-pre-csat-paper-2
Types of Functions
(i) One-one Function (injection): A function f : A B is said to be a one-one function elements of A have
different images in B ie,
f(x) =zx, x {1, 2, 3}
f ={(1, 2),(2, 4),(3, 6)}
(ii) Many-one Function: A function f : A B is said to be a many-one function if two are n of A have the
same images in B.
(iii) Onto Function: A function f : A B is called an onto function if every element of B is an image of some
elements of A ie, if co-domain =range.
eg, Let A ={a, b, c, d}and B ={1, 2, 3}
f ={(a, 3),(b, 2),(c, 2),(d, 1))
(iv) Into Function: A function f: A B is called an into function if co-domain range.
Example 3: A is set of prime numbers less than 20, write A in Roster form.
Sol uti on. Prime numbers Less than 20 are 2, 3, 5, 7, 11; 13, 17, 19 set A in Roster form.
A ={2, 3, 5, 7, 11, 13, 17, 19}
Example 4: Let A ={4, 5, 6, 7}and B ={6, 4, 7, 5}, then
Sol uti on. {4, 5, 6, 7}={6, 4, 7, 5}, since each of the elements 4 , 5, 6, 7 belongs to Band each of the elements
6, 4, 7, 5 belongs to A, then A =B.
The set does not change if its elements are rearranged.
Example 5: A ={x
2
=16, x is odd}, then
Sol uti on. A is a empty set.
x
2
=16 x =+4 or x =4, but x is not odd
A does not contain any element, A =
Example 6: Rewrite the following statements using set notations.
(a) x does not belongs to A (b) A is not a subset of B
(c) H does not include D (d) d is a member of E.
Sol uti on.
(a) x A (b) A B
(c) H D (d) d E
Example 7: Let A ={a, b, c}; ie, A contains the elements a, b, c, state whether each of the four statements is
correct or incorrect tell why.
(a) a A (b) a A
(c) {a} A (d) {a} A
Sol uti on.
(a) a A, correct.
(b) I ncorrect. The symbol must connect two set it indicates that one set a subset of other. Therefore,
a A is incorrect since a is a member of A, not a subset.
(c) I ncorrect. The symbol a connects an objects to a set. I t indicates that object is a member of the set.
Therefore, {a} A is incorrect since {a}is a subset of A.
(d) Correct.
Example 8: I f S be the universal set of English alphabet and let A ={a, b, c}, then complement of A is
Sol uti on. A
C
={d, e, f x, y, z}
Example 9: I f A ={1, 2, 3, 4}and B ={2, 4, 6, 8}, find A B, B A and A B.
Sol uti on. A B ={1, 3},
WWW.UPSCPORTAL.COM
For Any Help Call Our Course Director:
at +91 8800734161, 011- 65023618
Click Here to Order
http://www.upscportal.com/civilservices/study-kit/ias-pre-csat-paper-2
(A B) contains the elements of A but not the elements of B. Similarly, B A =16,81
(B A) contains the elements of B but not the elements of A.
A B =(A B) (B A) ={1, 3, 6, 8}
Example 10: I f S ={1, 2, 3, 4, 5}, A={1, 2, 4}, B={2, 4, 5}
Find
(a) A B (b) A B (c) B
C
(d) B A
(e) A
C
B (f) A B
C
(g) A
C
B
C
(h) B
C
A
C
(i) (A B)
C
(j) (A B)
C
Sol uti on.
(a) A B ={1, 2, 4, 5}
(b) A B ={2, 4}
(c) The complement of B consists of letters which are in S but not in B, therefore B
C
={1, 3}
(d) B A consisted of elements in B which are not in A ie, B A ={5}
(e) A
C
={3, 5}and B ={2, 4, 5}, therefore, A
C
B ={2, 3, 4, 5}
(f) A ={1, 2,4) and B
C
={1, 3}, therefore, A B
C
={1, 2,3,4}
(g) A
C
={3, 5}, and B
C
={1, 3}; therefore, A
C
B
C
={3}
(h) B
C
={1, 3}, and A
C
=(3,5); therefore, B
C
A
C
={1}
(i) A B ={2, 4}, therefore, (A B)
C
={1, 3,5}
(j) A B ={1, 2, 4, 5}; therefore, (A B)
C
={3}
Example 11: A ={1, 2, 3}and B ={a, b}, then find A B and B A
Sol uti on. A B ={{1, a}, {1, b}, {2, a}, {2, b), {3, a}, {3, b}}and B A ={{a,1}, {a, 2}, {a, 3}, {b, l}, {b, 2}, {b, 3}}
A B B A since the ordered pair (1, a) (a, 1)
Example 12: I f the set A contains 4 elements and set B contains 3 elements, then A B contains
Sol uti on. The set A B contains 12 elements.
EXERCISE
1. The set in set builder form of vowels of English
alphabet is
(a) {a, e, i, o, u}
(b) A ={x | x is a vowel in English alphabet)
(c)
(d) {a, b, c, d, e, f, g}
2. I f A ={5}which of the following statement is
correct?
(a) A =5 (b) 5 A
(c) {5} A (d) 5 A
3. I f A ={a, (b, c), d)}, which of the following is a
subset of A.
(a) {a, b} (b) {b, c}
(c) {c, d} (d) {a, d}
4. I f A B = nd A and B are two sets, then
(a) A B (b) B A
(c) A B (d) A and B are disjoint
5. I f n(A B) =13, n(A) =20, n(B) =44, then n(A
B) = ?
(a) 27 (b) 13 (c) 75 (d) 51
ANSWERS
1. (b) 2. (d) 3. (d) 4. (d) 5. (d)
EXPLANATIONS
1. Set builder form of vowels of English alphabet is
A = {x | x is a vowel in English alphabet}
2. The set A contains an element 5, therefore 5 A
3. Clearly, {a, d}is a subset of {a, (b, c), d}
4. I f A and B are disjoint sets, then A and B have no
common elements A B =f
5. n (A B)=n (A) + n (B) n (A B)
=20 +44 13 = 51
Arithmetic Progression (AP)
An arithmetic progression is a sequence in which terms increase or decrease by a constant number called the
common difference.
(i) The sequence 2, 6, 10, 14, 18, 22 is an arithmetic progression whose first term is 2 and common difference
4.
(ii) The sequence
5 7
2, ,3, ,4
2 2
is an arithmetic progression whose first term is 2 and common
difference .
An arithmetic progression is represented by a,(a +d), (a +2d), (a +3d) a +(n 1)d
Here, a =first term
d =common difference
n =number of terms in the progression
The general term of an arithmetic progression is given by T
n
=a +(n - 1) d.
The sum of n terms of an arithmetic progression is given by S, =
2
n
[2a +(n 1) d] or S
n
=2 [a +l]
where l is the last term of arithmetic progression.
I f three numbers are in arithmetic progression, the middle number is called the arithmetic mean of the
other two terms.
I f a, b, c are in arithmetic progression, then b =
2
a c +
where b is the arithmetic mean.
Similarly, if n terms a
l
, a
2
, a
3
a
n
are in AP, then the arithmetic mean of these n terms is given by
AM =
1 2 3
.
n
a a a a
n
+ + ++
I f the same quantity is added or multiplied to each term of an AP, then the resulting series is also an AP.
I f three terms are in AP, then they can be taken as (a d), a, (a + d).
I f four terms are in AP, then they can be taken as (a 3d), (a d), (a + d), (a +3d).
I f five terms are in AP, then they can be taken as (a 2d), (a d), a, (a + d), (a +2d).
Sequences & Series
CHAPTER 29
WWW.UPSCPORTAL.COM
For Any Help Call Our Course Director:
at +91 8800734161, 011- 65023618
Click Here to Order
http://www.upscportal.com/civilservices/study-kit/ias-pre-csat-paper-2
Geometric Progression (GP)
A geometric progression is a sequence in which terms increase or decrease by a constant ratio called the
common ratio.
(i) The sequence 1, 3, 9, 27, 81is a geometric progression whose first term is 1 and common ratio 3.
(ii) The sequence
1 1 1 1
1, , , , ,
3 9 27 81

is a geometric progression whose first term is 1 and common ratio


1
.
3
A geometric progression is represented by a, ar, ar
2
ar
n1
.
Here, a =first term
r =common ratio
n =number of terms in the progression.
The general term of a geometric progression is given by T
n
=a
n1
The sum to n terms of a geometric progression is given by
( )
1
,
1
n
n
a r
S
r

when r < 1
( )
1
,
1
n
a r
r

when r > 1
I f three numbers are in geometric progression, the middle number is called the geometric mean of the other
two terms.
I f a, b, c are in geometric progression, then b ac where b is the geometric mean.
Similarly, if n terms a
1
, a
2
, a
3
, a
4
,a
n
are in geometric progression, then the geometric mean of 1 these n
terms is given by GM =( )
1
1 2 3
. n
n
a a a a
For a decreasing geometric progression the sum to infinite number of terms is
,
1
a
S
r

where a = first term and r <1.


I f every term of a GP is multiplied by a fixed real number, then the resulting series is also a GP.
I f every term of a GP is raised to the same power, then the resulting series is also a GP.
The reciprocals of the terms of a GP is also a GP.
I f three numbers are in GP, then they can be taken as
,
a
r
a, ar.
I f four numbers are in GP, then they can be taken as
3
3
, , , .
a a
ar ar
r
r
I f five numbers are in GP, then they can be taken as
2
2
, , , , .
a a
a ar ar
r
r
Harmonic Progression (HP)
I f the reciprocals of the terms of a series form an arithmetic progression, then the series is called a harmonic
progression.
(i) The sequence
4 3 12
, , ,
3 2 7

is a harmonic progression as
3 2 7
, ,
4 3 12
is in arithmetic progression.
I f a, b, c are in harmonic progression, then b =
2ac
a c +
where b is the harmonic mean.
WWW.UPSCPORTAL.COM
For Any Help Call Our Course Director:
at +91 8800734161, 011- 65023618
Click Here to Order
http://www.upscportal.com/civilservices/study-kit/ias-pre-csat-paper-2
Sum of Natural Series
The sum of the first n natural numbers =
( ) 1
2
n n +
The sum of the square of the first n natural numbers =
( )( ) 1 2 1
6
n n n + +
The sum of the cubes of the first n natural numbers =
( )
2
2
1
4
n n+
The sum of first n even numbers =n(n +1)
The sum of first n odd numbers =n
2
Example 1: Find the nth term and the fifteenth term of the arithmetic progression 3, 9, 15, 21
Sol uti on. I n the given AP we have a =3, d =(9 3) =6
T
n
=a +(n 1)d =3 +(n 1)6 =6n 3
T
15
=(6 15 3) =87
Example 2: Find the 10th term of the AP 13, 8, 3, 2,
Sol uti on. I n the given AP, we have a =13, d =(8 13) =5
T
n
=a +(n 1)d =13 +(n 1)(5) =18 5n
T
10
=18 5 (10) =32
Example 3: The first term of an AP is -1 and the common difference is -3, the 12th term is
Sol uti on. T
1
=a =1, d =3
T
n
=a +(n 1)d =1 +(n 1)(3) =2 3n
T
12
=2 3 12 =34
Example 4: Which term of the AP 10, 8, 6, 4 is 28?
Sol uti on. We have, a =10,d =(8 10) =2, T
n
=28
T
n
=a +(n 1)d 28 =10 +(n 1)(2) =n =20
Example 5: The 8th term of an AP is 17 and the 19th term is 39. Find the 20th term.
Sol uti on. T
8
=a +7d =17 ...(i)
T
19
=a +18d =39 ... (ii)
On subtracting Eq. (i) from Eq. (ii), we get 11d =22 d =2
Putting d =2 in Eq. (i), we get a +7(2) =17 =a =(17 14) =3
First term =3, Common difference =2
T
20
=a +19d =3 +19(2) =41
Example 6 Find the sum of the first 20 terms of the AP, 2,1, 4,7,
Sol uti on. Here, a =5, d =(12) =3 and n =20
S
n
= ( ) 2 1
2
n
a n d 1 +
]
( )( )
20
20
20 5 20 1 3 470
2
S 1 +
]
Example 7: Find the sum of the series 5, 10, 15, 20,125.
Sol uti on. Here, a =5, T
n
=125, d =(10 5) =5
T
n
=a +(n 1)d 125 =5 +(n 1) 5 n =25
S
n
= [ ] [ ]
25
5 125 1625
2 2
n
a l + +
WWW.UPSCPORTAL.COM
For Any Help Call Our Course Director:
at +91 8800734161, 011- 65023618
Click Here to Order
http://www.upscportal.com/civilservices/study-kit/ias-pre-csat-paper-2
Example 8: Find three numbers in AP whose sum is 36 and product is 1620.
Sol uti on. Let the numbers be (a d), a, (a +d). Then, (a d) +a +(a +d) =36 3a =36 a =12
(a d) a (a +d) =1620 (12 d) l2 (12 +d) =1620
(144 d
2
) 135 d
2
9 d =3
Numbers are 9, 12, 15 or 15, 12, 9.
Example 9: Find the nth term and 8th term of the GP 3, 6, 12, 24,48,
Sol uti on. I n the given GP, we have a =3, r =
6
2
3

T
n
=ar
n1
=(3)(2)
n1
T
8
=(3)(2)
81
=(3)(2)
7
=384
Example 10: The n
th
term of GP is 3/2
n
. Find the ratio of 5th to 10th term.
Sol uti on. I n the given GP, we have
3
2
n
n
T
5
5 10 5 10
5 10 5 10 5
3 3 3 3 1
, , : : 1: 2 :1 32:1
2 2 2 2 2
T T T T
Example 11: Determine the 9th term of GP whose 8th term is 192 and common ratio 2.
Sol uti on. I n the given GP, we have r =2, T
8
=ar
7
=192
a(2)
7
=192 a =
192 3
128 2

T
9
=ar
8
= ( )
( )
8
7
3
2 3 2 384
2

Example 12: The first term of a GP is 50 and the 4th term is 1350. Determine the 6th term.
Sol uti on. Let a be first term and r be the common ratio.
Then, a =50 ...(i)
T
4
=ar
3
=1350 ...(ii)
On dividing Eq. (ii) by Eq. (i), we get r
3
=27 r =3
Example 13: Find the sum to infinity for the GP
1 1 1
, , ,
4 16 64

Sol uti on. I n the given GP, we have a =
1 1/16 1
, ,
4 1/ 4 4
r


1 1
1 4 1
4 4
5 1 4 5 5 1
1
4
4
a
S
r



1 _

1
, ]
Example 14: I n a certain colony of cancevous cells, each cell divides into two every minute. How many cells
will be produced from a single cell, if the rate of division continues for 12 min?
Sol uti on. Total number of cells =2 +(2
2
+2
3
+2
4
+...+2
12
)
=2
1
+2
2
+2
3
+....+2
12
=
( )
12
2 2 1
8190
2 1

WWW.UPSCPORTAL.COM
For Any Help Call Our Course Director:
at +91 8800734161, 011- 65023618
Click Here to Order
http://www.upscportal.com/civilservices/study-kit/ias-pre-csat-paper-2
1. I f five times the fifth term of an AP is equal to
seven times, the seventh term of the AP, then
what is the twelfth term?
(a) 1 (b) 0
(c) 1 (d) 2
2. Three terms are in AP such that their sum is 18.
The sum of the first two terms is six more than
the sum of the last two terms. Find the last term.
(a) 6 (b) 9
(c) 3 (d) 2
3. Determine k, so that (k +2), (4k 6) and (3k 2)
are three consecutive terms of an AP.
(a) 3 (b) 2
(c) 4 (d) 6
4. I n an AP, the first term is 2 and the sum of the
first five terms is one-fourth the sum of the next
five terms. Find the second term.
(a) 4 (b) 10
(c) 16 (d) 12
5. The sum of four terms in an AP is 64. The product
of the extreme terms is 220. Find the first and
fourth term.
(a) 14, 28 (b) 10, 22
(c) 28, 14 (d) 6, 30
ANSWERS
1. (b) 2. (c) 3. (a) 4. (a) 5. (b)
EXPLANATIONS
1. T
5
= a +4d, t
7
= a +6d
5 (a +4d) =7 (a +6d) 5a +20d =7a +42d
a =11d
T
12
= a +11d = 11d +11d = 0
The twelfth term is 0.
2. Let the three terms be a d, a, a + d.
(a d) + a +(a + d) = 18
EXERCISE
3a =18
a =6
Also, [(a d) + a] [a +(a + d)] = 6 d = 3
So, the three terms are 9, 6, 3 respectively, last
term =3.
3. Since, (k +2), (4k 6) and (3k 2) are in AP.
(4k 6) (k +2) =(3k 2) (4k 6)
4k 6 k 2 =3k 2 4k +6 k = 3
4. Given, a =2, S
n
=
2
n
[2a +(n 1) d]
S
5
=
5
2
[2 2 +(5 1) d] =10 (d +1) ...(i)
S
10
=
10
2
[2 2 +(10 1) d] =5 (9d +4) ...(ii)
Sum of the sixth to tenth term is
S
10
S
5
=(35d +10) ...(iii)
Now, given S
5
=
1
4
(S
10
S
5
)
10(d +1) =
1
4
(35d +10)
d = 6
T
2
= a + d =2 +( 6) = 4
5. Let the four terms be
(a 3d), (a d), (a + d), (a +3d)
(a 3d) + (a d) + (a + d) + (a +3d) = 64
4a =64 a =16 ...(i)
Also, (a 3d) (a +3d) =220
a
2
9d
2
=220
(16)
2
9d
2
=220 d = 2
First term =a 3d =(16 3 2) =10
Fourth term =a +3d =(16 +3 2) =22
WWW.UPSCPORTAL.COM
For Any Help Call Our Course Director:
at +91 8800734161, 011- 65023618
Click Here to Order
http://www.upscportal.com/civilservices/study-kit/ias-pre-csat-paper-2

Sample Chapters
of
Data Interpretation
&
Data Sufficiency
WWW.UPSCPORTAL.COM
For Any Help Call Our Course Director:
at +91 8800734161, 011- 65023618
Click Here to Order
http://www.upscportal.com/civilservices/study-kit/ias-pre-csat-paper-2
Data Interpretation
Bureaucrats and Government executives must face lots of facts and figures in everyday business. Normally
these facts are presented in more compact and precise forms such as
1. Tables (known as data tables)
2. Charts (Pie, Bar, Pert, etc.)
3. Graphs (2D and 3D)
4. Diagrams (Geometric or Venn diagram)
An Administrator must possess basic skills on deciphering the data from the above mentioned precise forms of
tables, charts etc. which enhances his/her administrative efficiency. This ability is known as data interpretation.
So I AS Prelim/CSAT exam test the candidates ability of data interpretation.
Exampl e 1
Questi on: These questions are based on following pie chart, which gives the details of percentage of energy
source of I ndia.
Nuclear, 2%
Hydro, 2%
Gas, 9%
Coal, 51%
Oil, 36%
Q. The second largest energy source of I ndia is
(a) Coal (b) Oil
(c) Gas (d) Hydro and Nuclear Ans: b
Note: Question may not be always direct. You may need to compute answers before arriving conclusion. Look
the example below.
Exampl e 2
Questi on: These questions are based on following table, which gives the details of Distribution of paper
industry in I ndia during 1997 especially state wise no of mills, its production capacity in thousand metric ton and
Percentage of mills of all I ndia it possess.
Introduction to
Data Interpretation
CHAPTER 1
WWW.UPSCPORTAL.COM
For Any Help Call Our Course Director:
at +91 8800734161, 011- 65023618
Click Here to Order
http://www.upscportal.com/civilservices/study-kit/ias-pre-csat-paper-2
State No. of Mills Production Capacity Percentage of
(in 000 MT) all India
Maharashtra 63 624 16.52
Andhra Pradesh 19 427 11.3
Gujarat 55 369 9.77
Uttar Pradesh 68 336 8.89
All India (Total) 379 3778 100
Q. Which of the state has lowest productivity with respect to number of mills it possesses?
(a) Maharashtra (10.19) (b) Andhra Pradesh (22.47)
(c) Gujarat (6.71) (d) Uttar Pradesh (4.94) Ans: d
Note: Also there may be few questions unable to interpret using available chart or tables. I t needs careful
attention however it is very easy.
Question Patterns of Data Interpretation
There are a multiple ways in which data can be shown to you. They are various forms of tables, charts, bars,
diagrams. For UPSC-CSAT, 2011, you can expect Data I nterpretation questions on singular bar, multiple bars,
tables, pie chart, line diagram etc. There you will be provided certain data and instructions. On the basis of the
instructions you have, you have to answer the questions asked to you, along with the data.
Ways to Prepare for Data Interpretation
For interpreting data, you should read a lot. Read lot of business reports and the analysis given with the
presentation of data. This will give you a fair idea on how data are presented and how to interpret or analyze
them. You must read articles, especially on business which are heavy on data and analyze them. This will keep
your DI cells on toes. I n a similar note, Refer to the graphs and data provided in various business papers and
magazines. You need to identify techniques to solve questions which require big and time consuming calculations.
At this stage, when there are few months left for UPSC-CSAT, 2011 you should solve at least three to four
Data I nterpretation sets every day. Also, you need to be good with quick calculation of ratios, percentages, etc.
With consistent practice, you will be able to achieve the same.
While Solving Data Interpretation Questions
While solving the Data I nterpretation questions, you must keep some points in mind. Experts suggest the
following points which you should keep in mind while you start attempting the Data I nterpretation questions.
Read the title carefully
Take your time and read both the questions and data set carefully. Understand what you are being asked to
do before you begin figuring out the information.
Check the data and types of information required. Be sure that you are looking at the right part of graph or
chart, i.e., on paper columns or rows or proper lines.
Read the parameters listed along with axes and the scale. This is the most important points as generally one
assumes standard scale as 1 unit =100 or 1 unit =10. Also the scales of two axes might be different.
The footnotes give the additional information for particular data. This also explains the jargons and lists the
formulae that might be needed to solve the questions asked.
Check the units required. Be sure that your answer is in thousand, millions or whatever it is that the
question specifies.
WWW.UPSCPORTAL.COM
For Any Help Call Our Course Director:
at +91 8800734161, 011- 65023618
Click Here to Order
http://www.upscportal.com/civilservices/study-kit/ias-pre-csat-paper-2
Circle graphs are used to show how various sectors share in the whole. Circle graphs are sometimes called pie
charts. Circle graphs usually give the percentage that each sector receives.
Expenditure of Major Industries
2000 ($3,087 million)
Legal
affairs, 12%
Research,
15%
Miscellaneous,
5%
Materials,
18%
Advertising,
12%
Construction,
2%
Taxes, 10%
Compensation,
26%
2010 ($4,851 million)
Research,
14%
Materials, 22%
Construction,
5%
Advertising,
15%
Legal
affairs, 4%
Taxes, 2%
Compensation,
38%
Q. The amount spent on materials in 2000 was 120% of the amount spent on
(a) research in 2000 (b) compensation in 2000
Pie Charts
CHAPTER 4
WWW.UPSCPORTAL.COM
For Any Help Call Our Course Director:
at +91 8800734161, 011- 65023618
Click Here to Order
http://www.upscportal.com/civilservices/study-kit/ias-pre-csat-paper-2
(c) advertising in 2010 (d) materials in 2010
I n 2000, 18% of the expenditures were for materials. We want x where 120% of x =18%; so x =15%. Any
category that received 15% of 2000 expenditures gives the correct answer, but only one of the five choices is
correct. Here, the answer is (a) since research received 15% of the expenditure in 2000.
Exampl e
Di recti ons : Study the fol l owi ng i nformati on to answer the gi ven questi ons.
Percentage of students in various courses (A, B, C, D, E, F) and Percentage of girals out of these.
Total Student: 1200 (800 girls + 400 boys) Percentage in Various Courses
A
20%
B
15%
C
5%
D
35%
E
12%
F
13%
Total Girls : 800
Percentage of Girls in Couress
A, 30%
B, 10%
C, 2%
D, 30%
E, 14%
F, 14%
1. For which course is the number of boys the minimum?
(a) E (b) F
(c) C (d) A
2. How many girls are there in course C?
(a) 44 (b) 16
(c) 40 (d) 160
3. For course D what is the respective ratio of boys and girls?
(a) 3 : 4 (b) 4 : 5
(c) 3 : 5 (d) 5 : 6
4. For which pair of courses is the number of boys the same?
(a) E and F (b) A and D
(c) C and F (d) B and D
5. For course E, the number of girls is how much percent more than the boys for course E?
(a) 250 (b) 350
(c) 150 (d) 80
WWW.UPSCPORTAL.COM
For Any Help Call Our Course Director:
at +91 8800734161, 011- 65023618
Click Here to Order
http://www.upscportal.com/civilservices/study-kit/ias-pre-csat-paper-2
Sol uti on
Calculations for questions (15)
Courses Boys Girls Total
A 0 240 240
B 100 80 180
C 44 16 60
D 180 240 420
E 32 112 144
F 44 112 156
1. (d) Number of boys in course A is minimum i.e., zero.
2. (b) Number of girls in course C =16
3. (a) Required ratio =180 : 240 =3 : 4
4. (c) Number of boys in each of courses C and F =44
5. (a) Required percentage =
112 32
32
100
=
18
32
100
=250
EXERCISE
Di r ecti ons: Study the fol l owi ng pi e-di agr ams
careful l y to answer these questi ons:
Number of students studying
in different faculties in the years 2001 and 2002 from
State X
Year-2001
Total Students - 35000
Arts, 12%
Commerce,
22%
Science, 24%
Agriculture, 7%
Engmeering,
18%
Pharmacy, 6%
Medicine, 11%
Year - 2002
Total Students - 40000
WWW.UPSCPORTAL.COM
For Any Help Call Our Course Director:
at +91 8800734161, 011- 65023618
Click Here to Order
http://www.upscportal.com/civilservices/study-kit/ias-pre-csat-paper-2
Arts, 11%
Commerce, 24%
Science, 22%
Agriculture, 5%
Engmeering, 19%
Pharmacy, 9%
Medicine, 10%
1. I n which faculty there was decrease in the number
of students from 2001 to 2002?
(a) None
(b) Arts
(c) Agriculture (d) Pharmacy
2. What is the ratio between the number of students
studying pharmacy in the years 2001 and 2002
respectively?
(a) 4 : 3 (b) 3 : 2
(c) 2 : 3 (d) 7 : 12
3. What was the approximate percentage increase
in the number of students of Engineering from
the year 2001 to 2002?
(a) 17 (b) 15
(c) 25 (d) 20
4. I n the year 2001, the number of students studying
Arts and Commerce together is what percent of
the number of students studying these subjects
together in 2002?
(a) 76 (b) 85
(c) 82 (d) 79
5. I n which of the following faculties the percent
increase in the number of students was minimum
from 2001 to 2002?
(l) Arts
(b) Science
(c) Commerce
(d) Medicine
ANSWERS
1. (c) 2. (d) 3. (d) 4. (b) 5. (d)
EXPLANATIONS
Calculation for questions (15):
Number of students in different faculties
Subj ect Year 2001 Year 2002
Arts 4200 4400
Commerce 7700 9600
Science 8400 8800
Agriculture 2450 2000
Engineering 6300 7600
Pharmacy 2100 3600
Medicine 3850 4000
1. The number of students decreased from 2450 to
2000 in Agriculture
2. Number of students studying Pharmacy in 2001
=2100
Number of students studying Pharmacy in 2002
=3600
Required ratio = 2100 : 3600 =7 : 12
3. Difference =7600 6300 =1300
Required percentage increase
=
1300
6300
100 =20.63 =20
4. Total number of students in Arts and Commerce
faculties in 2001
=4200 +7700 =11900
Total number of students in Arts and Commerce
faculties in 2002 =4400 +9600 =1400
Required percentage =
11900
14000
100
=85%
5. Only on the basi s of table, we can say, the
percentage increase in Medicine was minimum.
WWW.UPSCPORTAL.COM
For Any Help Call Our Course Director:
at +91 8800734161, 011- 65023618
Click Here to Order
http://www.upscportal.com/civilservices/study-kit/ias-pre-csat-paper-2
You can compare several categories by a graph of the cumulative type. These are usually bar or line graphs
where the height of the bar or line is divided up proportionately among different quantities.
Q. I n 1986, roughly what percent of the federal prisoners received from the courts were guilty in other offenses?
(a) 10 (b) 15
(c) 25 (d) 30
The total number of prisoners in 1986 was about 20,000, and guilty in other offences was about 5,000.
Since = =
5, 000 1
25%,
20, 000 4
the correct answer is (c).
I n DI section of the question paper, the intention should be to attempt all questions as skipping them would
amount to losing precious scoring opportunities. There are two approaches to arrive at the solution. One is to work
on the data to arrive at the correct answer. The other one is the Elimination method which requires working
backwards by eliminating the wrong choices. Though the elimation method is more time consuming, it may still be
preferred where direct solution involves enormous calculation of data.
At times, examiners pose rather difficult data sets at the beginning of the sections. These are intended to be
SPEED BRAKERS which take away much of precious time. Therefore, as a rule, scan the whole section quickly
before actually attempting the questions and start with easier part of the section.
I n some exams, data are presented in more than one table or graph. The objective is to test not only quantitative
skills but also relational and analytical ability. Recently, in some exams the questions in this section are being
Mix Diagrams
CHAPTER 7
WWW.UPSCPORTAL.COM
For Any Help Call Our Course Director:
at +91 8800734161, 011- 65023618
Click Here to Order
http://www.upscportal.com/civilservices/study-kit/ias-pre-csat-paper-2
framed in caselet (paragraph) form, beginning with probability and reasoning questions. I t is left to the reader to
study the case, call out requisite data and arrange it in a suitable form for meaningful interpretation. I t is best to
arrange data with rough sketch to hasten comprehension.
I mportant Ti ps: These will help in saving time, reducing mistakes and finding solution easily.
1. Get a general picture of the information by looking at the entire table or graph.
2. Read the table title, nomenclatures of columns and rows.
3. Simplify the questions being asked. Break down lengthy questions into smaller parts.
4. Use only the information given for finding solutions. Select the appropriate data for answering a question.
5. Eliminate impossible choices.
6. Avoid lengthy calculations.
7. Try to interpret through trends of the data in the graph. Whenever possible, try to answer the questions by
visualizing rather than by computing.
8. Where calculation is required prefer approximate values at the first stage. Go for exact calculation where
values are close and require exact answer.
9. Be careful to use proper units.
10. Make correct use of your knowledge of basic mathematical rules, principles and formulae.
11. Dont confuse in decimals and percent ages. For example, 0.5% =0.005.
12. Use pencil or straight edge of the answer sheet to read the graph and find approximate values.
13. Focus your answer on the question actually asked and not on what the question should be in your opinion.
14. Never do anything that is unnecessary.
15. Last, but not the least, make sure that the answer is sensible and reasonable.
EXERCISE
Di recti ons: Study the pi e-chart gi ven to answer
these questi ons.
Composition of Exports and Imports (per cent)
Exports (19981999)
AAgriculture and Allied Products
BOres and Minerals
CManufactured Goods
DCrude and Petroleum Products
ECapital Goods
Imports (19981999)
AFood and Allied Products
BFuel
CFertilisers
DPaper Board Manuf.and Newsprint
EOthers
FOther Bulk I tems
GOthers
1. I ndias trade balance during the year 199899
was:
(a) favourable (b) unfavourable
(c) neutral (d) none of these
2. Maximum export contribution has been due to :
(a) agriculture and allied products
(b) ores and minerals
(c) crude and petroleum products
(d) manufactured goods.
3. The import of capital goods and other bulk items
incurred more thanof total import expenditure
(a) 50% (b) 60%
(c) 35% (d) 70%
4. The imports exceeded by aboutmillions.
(a) US$ 8000 (b) US$ 8200
(c) US$ 8199.4 (d) US$ 8400
WWW.UPSCPORTAL.COM
For Any Help Call Our Course Director:
at +91 8800734161, 011- 65023618
Click Here to Order
http://www.upscportal.com/civilservices/study-kit/ias-pre-csat-paper-2
5. The export of agriculture and allied products was
to the tune of US$ :
(a) 5700 millions (b) 6700 millions
(c) 4700 millions (d) 1730 millions
ANSWERS
1. (b) 2. (d) 3. (a) 4. (c) 5. (a)
EXPLANATIONS
4. I mport Export
=(41857.9 33658.5) =8199.4 million
5. % of Agriculture and allied products
=17.3%
=33658.5 17.3% =5822 million
5700 million
WWW.UPSCPORTAL.COM
For Any Help Call Our Course Director:
at +91 8800734161, 011- 65023618
Click Here to Order
http://www.upscportal.com/civilservices/study-kit/ias-pre-csat-paper-2
I n this section the UPSC tests the ability of the candidate to identify whether the data given are sufficient to
answer the question or not. Although data sufficiency was given as a topic under Data I nterpretation its scope
also extends from mathematical based quantitative ability questions to non-mathematical based simple reasoning
under Analytical Section.
Since these questions require candidates to identify the sufficiency of data, you must stop at the stage of
determining the sufficiency of data and you are not expected to solve the problem completely.
Like Assertion and Reason which all Civil service aspirants have prior knowledge from the old question
pattern, this data sufficiency questions also have set of directions?
For exampl e:
Raju is the tallest boy in the school. I s he the tallest student in the class?
1. Priyanka is the tallest girl in the class
2. Priyanka is shorter than Raju
Choose A: I f statement 1 alone is sufficient and statement 2 alone is sufficient to answer
Choose B: I f statement 1 alone is sufficient and statement 2 alone is not sufficient to answer
Choose C: I f statement 1 alone is not sufficient and statement 2 alone is sufficient to answer
Choose D: I f both the statements are sufficient to answer
Ans: D
However there are 3 different sets of Directions as they are appearing in different examinations.
Note: I f the UPSC did not mention the set of direction earlier in the oncoming notification, then we must
prepare for all sets of directions.
Some Typical Cases: Data Sufficiency
The questions can be broadly divided into the following categories:
1. Relationships
2. Dates
3. Comparison
4. Critical Analysis
5. Age
6. Miscellaneous
Data Sufficiency
CHAPTER 8
WWW.UPSCPORTAL.COM
For Any Help Call Our Course Director:
at +91 8800734161, 011- 65023618
Click Here to Order
http://www.upscportal.com/civilservices/study-kit/ias-pre-csat-paper-2
1. Relationships
The wording goes like this: How is P related to Q?. I n these questions one very prominent source of confusion
is the fact that students tend to conclude about relationship without knowing about the sex of the person involved.
2. Dates
The wording of the question goes like this: On which day of the week did P reach (say) Delhi? or What is
the date of birth of P?. I n such questions, you have to determine a day or a date. Usually, the data will provide
you with some knowledge of some previous day or date and from this you can work out the required day or date.
3. Comparison
The wording of the question is: Who is the tallest among P, Q and R? or Who is the heaviest among X, Y
and Z? or Who is sitting to the left of P when P, Q and R are sitting together? etc. Here you have to make
comparisons of the respective places of some persons in a fixed pattern or order.
4. Critical Analysis
Sometimes questions asked in the examination are qualitative in nature, wherein you have to apply your
value-judgement in order to reach a conclusion.
5. Age
I n some questions of data sufficiency, problems are concerned with age. Again, we must emphasis that the
question does not ask you to actually calculate the age of the person. I t only wants you to tell whether the given
statements are sufficient (or not) to calculate the age of that person.
6. Miscellaneous
Although the five categories mentioned above are standard categories and most of the questions asked will be
from these five categories, these are certain type which do not belong to any of them. We may sell them miscellaneous
questions.
EXERCISE
Di r ecti on: I s the data gi ven i n two statements,
l abel ed (1) and (2), suffi ci ent for answeri ng the
questi on? Al l numbers used are real numbers.
Choose (a): I f statement 1 alone is sufficient and
statement 2 alone is sufficient to answer
Choose (b): I f statement 1 alone is sufficient and
statement 2 alone is not sufficient to answer
Choose (c): I f statement 1 alone is not sufficient
and statement 2 alone is sufficient to answer
Choose (d): I f both the statements are sufficient
to answer
1. I f n is a positive integer greater than 1, is 3n(n
2
-
1) divisible by 12 ?
(1) n is odd. (2) n is a multiple of 3.
2. Boxes A, B and C together contain a total of 8
pencils. Each box contains at least 1 pencil. Box A
contains more than 1 pencil. How many pencils
clops each hox contain?
(1) The number of pencils in Box A is not a prime
number.
(2) Box B and Box C contain the same number of
pencils.
3. I f 16
x +y/2+1
=4
x +2y
what are the values of x and
y?
(1) 7
2x +3y
=49
38
(2) 5
4
=25
x y
4. How much did a family spend on their Alaskan
vacation?
(1) There were 4 members in the family.
(2) Each family member spent ` 1799.
5. What is the value of m?
(1) m =
81
(2) m
2
=81
6. Hemants flower-vase contains only yellow asters
and orange asters. What is the ratio of orange
asters to yellow asters in her flower-vase?
(1) The number of yellow asters is 2 less than the
number of orange asters.
WWW.UPSCPORTAL.COM
For Any Help Call Our Course Director:
at +91 8800734161, 011- 65023618
Click Here to Order
http://www.upscportal.com/civilservices/study-kit/ias-pre-csat-paper-2
(2) The number of yellow asters is 3/4 the number
of orange asters.
7. What is the area of rectangle PQRS?
(1) I ts diagonal is 13 feet.
(2) PS =5 feet
S R
P Q
8. A rectangular photograph is to be placed in a
wooden frame. What is the area of the photograph?
(1) The length of the photograph is 7 inches.
(2) The area of the wooden frame is 55 square
inches.
9. I s possitive integer a even?
(1) a is a multiple of 4.
(2) a is a factor of 4
4
2
2
.
10. What is the value of odd integer s?
(1) 1 <s <10.
(2)
s
is also an odd integer.
ANSWERS
1. (a) 2. (c) 3. (a) 4. (c) 5. (a)
6. (b) 7. (c) 8. (c) 9. (a) 10. (c)
EXPLANATIONS
1. Now, 3n (n
2
1) =3n(n 1)(n +1).
For 3n(n
2
1) to be divisible by 12, it is necessary
that n(n 1)(n +1) be divisible by 4.
Statement (1) specifies that n is odd, which implies
that (n - 1) and (n +1) are even numbers and are
each divisible by 2. So, (n - 1)(n +1) is divisible by
4. Thus, statement (1) ALONE is sufficient.
Statement (2) specifies that n is a multiple of 3.
Consider an even multiple of 3 [because an odd
multiple has been already considered in statement
(1)]. I f n is even, then (n 1) and (n +1) are odd
integers. Since odd integers are not divisible by 2,
n must be divisible by 4 for n(n 1)(n +1) to be
divisible by 4. However, not all even multiples of
3 are divisible by 4. Thus, statement (2) ALONE
is not sufficient.
Alternative Approach :
Since statement (1) specifies that n is odd, let
n =2m +1, where m is a positive integer. Then,
3n(n
2
1) =3(2m +1)(4m
2
+4m +1 1) =12(2m
+1)(m
2
+m), which is divisible by 12.
Thus, statement (1) ALONE is sufficient.
Since statement (2) specifies that n is a multiple
of 3, let n =3m, where m is a positive integer.
Then,
3n(n
2
1) =9m(9m
2
1), which is divisible by 9
but not necessarily 12.
Thus, statement (2) ALONE is not sufficient.
2. Let the pencil distribution be represented by (a,
b, c), where Box A has a pencils, Box B has b
pencils and Box C has c pencils.
Each box has at least I pencil. So, out of a total of
8 pencils, 5 pencils now need to be distributed
among 3 boxes.
Given that 1, 2, 3, 5 and 7 are the only prime
numbers less than 8, statement (1) gives rise to
four possibilities: (6, 1, 1), (4, 2, 2), (4, 3, 1) and
(4, 1, 3).Thus, statement (1) ALONE i s not
suffi cient. Statement (2) gi ves ri se to three
possibilities: (6, 1, 1), (4, 2, 2) and (2, 3, 3). Thus,
statement (2) ALONE is not sufficient.
When both statements are considered together,
two possibilities still exist: (6, 1, 1) and (4, 2, 2).
Thus, statements (1) and (2) TOGETHER are not
sufficient.
3. The questi on statement gi ves the fol l owi ng
equation, on converting to same base:
4
2x +y +2
=4
x +2y
2x +y +2 =x +2y or y x =2
Statement (1) gives the following equation, on
converting to same base:
7
2x +3y
=7
76
2x +3y=76
The two equations (y x =2 and 2x +3y =76) can
be simultaneously solved to obtain values of the
two unknowns x and y. Thus, statement (1)
ALONE is sufficient.
Statement (2) gives the following equation, on
converting to same base:
5
-4
=5
2x - 2y
2x 2y=4 or y x =2
Note that this equation is the same as that obtained
from the question statement. Thus, statement (2)
ALONE is not sufficient.
4. Statement (1) specifies the number of members,
but not how much each member spent. Thus,
statement (1) ALONE is not sufficient. Statement
(2) specifies how much each member spent, but
WWW.UPSCPORTAL.COM
For Any Help Call Our Course Director:
at +91 8800734161, 011- 65023618
Click Here to Order
http://www.upscportal.com/civilservices/study-kit/ias-pre-csat-paper-2
not the number of members. Thus, statement (2)
ALONE is not sufficient. Since the total expenses
are obtai ned on mul ti pl yi ng the number of
members by the amount each member spent,
BOTH statements TOGETHER are sufficient.
5. Statement (1) tells us that m =9. Thus, statement
(1) ALONE is sufficient.
Statement (2) tel ls us that m = 19. Thus,
statement (2) ALONE is not sufficient.
6. I f o and y are the number of orange asters and
yellow asters respectively, then o/y is the ratio that
is needed.
Statement (1) gives the following equation:
y =o 2, from which o/ y cannot be determined.
Thus, statement (1) ALONE is not sufficient.
Statement (2) gives the following equation:
y =3/4o, which can be solved to obtain o/ y =4/3.
Thus, statement (2) ALONE is sufficient.
7. To find the area of rectangle, its length and width
must be known because
Area of rectangle =Length Width.
Statement (1) specifies the diagonal. To find the
rectangles area using the Pythagorean theorem,
either its length or its width must be known.
Thus, statement (1) ALONE is not sufficient.
Statement (2) specifies the width. To find the
rectangles area, either its length or its diagonal
must be known. Thus, statement (2) ALONE is
not sufficient.
Knowing the width as well as the diagonal of the
rectangle allows us to calculate its length by the
Pythagorean theorem, and subsequently its area.
Thus, BOTH statements TOGETHER are
sufficient.
8. To find the area of the photograph, its length and
wi dth must be known or some addi ti onal
information about the wooden frame must be
given.
Statement (1) speci fi es the l ength of the
photograph, but not its width. Thus, statement
(1) ALONE is not sufficient.
Statement (2) specifies the area of the wooden
frame, but nothing about the photograph. Thus,
statement (2) ALONE is not sufficient. Combining
both statements gives the following equation
(7 +2wf)(wp +2wf) 7wp =55,
where w
p
is the width of the photograph and wf is
the width of the wooden frame.
Since there is one equation with two unknowns,
it cannot be solved for w
p
unless the width of the
frame is given. Thus, Statements (1) and (2)
TOGETHER are NOT sufficient.
9. Statement (1) specifies that a is a multiple of 4,
and all multiples of 4 are even. Thus, statement
(1) ALONE is sufficient.
Statement (2) specifies that a is a factor of 44 22,
but a could be odd (1) or even (2, 4, ...). Thus,
statement (2) ALONE is not sufficient.
10. Statement (1) tel l s us that s = 3, or
s =5, or s =7, or s =9. Thus, statement (1) ALONE
is not sufficient.
Statement (2) specifies that the square root of s is
al so an odd i nteger, but there are many
possibilities of s (1, 9, 25, 49, ...) that satisfy this
cri terion. Thus, statement (2) ALONE is not
sufficient.
On combining both statements, the value of s is
uniquely determined as 9 (since 9 =3), because
3 and 5 and 7 are not odd integers. Thus,
BOTH statements TOGETHER are sufficient.

También podría gustarte